Você está na página 1de 184

Geometria Basica

Bruno Holanda
12 de novembro de 2011

Resumo
Este trabalho representa um conjunto de notas de aulas de um curso inicial em Geometria
Euclidiana Plana para alunos do ensino fundamental. A principal tafera dos exerccios aqui
apresentados e a formacao do rigor matematico necessario em problemas de geometria, porem
sem grandes aprofundamentos teoricos. Portanto, nos focaremos em tres pontos principais:
Teorema de Pitagoras,
areas e
angulos.

Teorema de Pit
agoras

O Teorema de Pit
agoras e um dos mais antigos e usados teoremas da geometria plana. Tambem
e ele que forma a base da geometria analtica de Descartes. Apesar de toda a sua fama, muitos
estudiosos da Historia da Matem
atica afirmam que Pit
agoras nao foi o verdadeiro autor desse
teorema. E que, muito possvelmente, os alunos da escola pitagorica sejam os reais autores.
Existem muitas provas, a maioria delas usa algum argumento de area. A solucao a seguir e
uma das mais simples.

Figura 1: Teorema de Pit


agoras
Prova.
Na figura 1, temos um quadrado de lado (a + b) particionado em um quadrado de
lado c e quatro tri
angulos ret
angulos de area ab
, por uma equivalencia de areas, temos que
2 . Da

Outros materiais como este podem ser encontrados em http://brunolholanda.wordpress.com/

(a + b)2 = c2 + 2ab, ou seja:


a 2 + b2 = c 2 .
Problema 1. Prove o Teorema de Pit
agoras de duas novas maneiras:
(a) (George Airy) Mostre como cortar dois quadrados em tri
angulos e quadrilateros e usar os
pedacos para formar um u
nico quadrado maior.
(b) (Henry Perigal) Dados dois quadrados, mostre como cortar um deles em quatro partes iguais
que, juntas com o outro quadrado, formem um u
nico quadrado maior.
Problema 2. Determine x nas seguintes figuras:
x

2 5

6
3

4
9

10

2 13

10

17

Problema 3. Na figura abaixo ABCD e um quadrado de area 64cm2 e EF GH um quadrado de


area 36cm2 . Determine a
area do quadrado CM HN .
N
D

Problema 4. Na figura abaixo os dois crculos s


ao tem o mesmo centro e os cinco quadrilateros
s
ao quadrados. Se o crculo menor tem raio igual a 1cm determine o raio do crculo maior.

Problema 5. Seja ABCD um quadrado de lado 28. Seja P um ponto no seu interior e E um
ponto no lado CD de modo que CDP E e AP = BP = P E. Ache AP .
Problema 6. Suponha que ABC seja um tri
angulo ret
agulo escaleno, e P seja o ponto na hi
potenusa AC tal que ABP = 45 . Dado que AP = 1 e CP = 2, calcule a area do tri
angulo
ABC.
Problema 7. Na figura abaixo temos dois crculos de raios 3 e 2 e cuja dist
ancia centro a centro
e 10. Ache o comprimento da tangente comum P Q.

Q
P
Problema 8. (Cone Sul 1989 - adaptado) Na figura abaixo temos dois quadrados, um de lado
dois e outro de lado um. Determine o raio do crculo que e tangente aos lados do maior e passa
pelo vertice do menor.

Problema 9. Nos lados AB e DC do ret


angulo ABCD, os pontos F e E s
ao escolhidos de modo
que AF CE seja um losango. Se AB = 16 e BC = 12, ache EF .
F

C
E
Problema 10. P e um ponto no interior do ret
angulo ABCD. Se P A = 2; P B = 3, e P C = 10.
Ache P D.
Problema 11. (Maio 2006) Um ret
angulo de papel 3cm 9cm e dobrado ao longo de uma reta,
fazendo coincidir dois vertices opostos. Deste modo se forma um pentagono. Calcular sua area.
Soluc
ao. Seja ABCD o ret
angulo e ABEF D o pentagono formado ao dobrar o paple como e
mostrado na figura a seguir:
3

A
b

Trabalhando com Angulos

Ao lado da distancia, o
angulo e uma unidade de medida fundamental para o estudo da geometria plana. Explicando de uma maneira formal, o angulo mede a diferenca entre as inclinacoes
de duas retas.
Problema 12. (Torneio das Cidades 1994) No tri
angulo ABC, ret
angulo em C, os pontos M e
N s
ao escolhidos sobre a hipotenusa de modo que BN = BC e AM = AC. Ache a medida do
angulo N CM .
Problema 13. Seja ABCDEF um hex
agono com todos os angulos internos iguais a 120 . Mostre
que
AB DE = CD F A = EF BC
Soluc
ao.

Sejam AF BC = P, ED AF = Q, BC ED = R. Como todos os angulos internos s


ao
120 obtemos que todos os tri
angulos P AB, QEF, RCD, P QR s
ao equilateros. Como
P Q = P R temos: AB + AF + F E = P A + AF + F Q = P B + BC + CR = AB + BC + CD,
entao CD AF = F E BC. A outra igualdade e an
aloga.
Problema 14. O oct
ogono ABCDEF GH e equiangular. Sabendo que AB = 1, BC = 2, CD = 3,
DE = 4, e EF = F G = 2, calcule o perimetro do oct
ogono.
Problema 15. Seja ABCDEF um hex
agono com todos os angulos internos iguais a 120 . Mostre
que
AB DE = CD F A = EF BC
4

Problema 16. Seja RST U V pentagono regular. Construa um triangulo eq


uilatero P RS com P
no interior do pentagono. Ache a medida do angulo P T V .
Problema 17. Ache a soma dos
angulos internos de uma estrela

Problema 18. No tri


angulo isosceles ABC com AB = AC, P e o ponto medio do lado AB tal
que AP = P C. Se a bissetriz do
angulo ABC corta P C em O de modo que P O = BO, ache
os angulo do tri
angulo.
Problema 19. No trapesio ABCD, de bases AB e CD, temos AD = 39, CD = 14, ABC = 69
e CDA = 138 . Ache a medida de AB.
Problema 20. (OBM) No ret
angulo ABCD, E e o ponto medio do lado BC e F e o ponto medio
b mede 20 . Quanto vale o angulo
do lado CD. A intersecao de DE com F B e G. O angulo E AF
b
E GB?
F
D
C
G
E
A
B
Problema 21. DEF G e um quadrado no exterior do pentagono regular ABCDE. Quanto mede
b ?
o angulo E AF
Problema 22. No tri
angulo ABC, D e E s
ao pontos sobre os lados BC e AC respectivamente.
Determine CDE sabendo que AB = AC, AE = AD e BAD = 48 .
na figura abaixo sabendo que AB = AC e BC = CD = DE =
Problema 23. Determine B AC
EF = F A.
A
F
E
D
B

Problema 24. No tri


angulo ABC com AB = BC, ABC = 144 . Seja K um ponto em AB, L
um ponto em BC e M em AC de modo que KL k AC, KM k BC e KL = KM . A reta LM
corta o prolongamento de AB em P . Ache a medida do angulo BP L.
Problema 25. No tri
angulo ABC com AB = BC, P , Q e R s
ao pontos nos lados AC, BC e AB,
respectivamente tais que P Q k AB, RP k BC e RB = AP . Se AQB = 105 , ache as medidas
dos angulos do ABC.
Problema 26. BE e AD s
ao as alturas do tri
angulo ABC, H e o ortocentro e F , G, K s
ao os
pontos medios dos segmentos AH, AB, BC, respectivamente. Prove que F GK e reto.
= 37 e C = 38 . Sejam P e Q pontos sobre
Problema 27. Em um tri
angulo ABC temos que B
o lado BC tais que BAP = P AQ = QAC. Se traca por B uma paralela `a AP e por C uma
paralela `a AQ. O ponto de encontro destas duas retas e D. Calcule DBC.
Problema 28. Em um romboide ABCD (AB = BC e CD = DA) as diagonais se cortam em
um ponto F . Sobre o prolongamento do lado BC se marca um ponto E de modo que CF = CE
e F CED tambem seja um romboide. Se ABC = 122 , quanto mede ADE?
Problema 29. (Maio 1996) Seja ABCD um quadrado e F um ponto qualquer do lado BC.
Traca-se por B a perpendicular `
a reta DF que corta a reta DC em Q. Quanto mede o angulo
F QC?

Areas

Problema 30. (OBM 2006) ABC e um tri


angulo ret
angulo e M e N s
ao pontos que trisectam
a hipotenuza BC. Sejam X e Y os simetricos de N e M em relacao ao ponto A. Determine a
area do quadrilatro XY CB, sabendo que o tri
angulo ABC tem area 1 cm2 .
Soluc
ao.
C
N
M
A

X
Y
Observe que AXY AN M e Y XA = AM N . Assim, XY k M N e como XY = M N =
M C = N B, segue que os quadrilateros XY CM e XY N B s
ao paralelogramos, como A e ponto
medio de XM e N Y temos que [AY C] = [BAX] = 32 . Logo, [XY CB] = 83 .

Problema 31. Na figuras abaixo ABC e um tri


angulo de area 72cm2 e M, N, P s
ao pontos
medios. Determine a
area da regi
ao sombreada.
A

N
P
B

C
M
Problema 32. Na figura abaixo D, E, F, G s
ao pontos medios. Determine a area que esta faltando.
D
210

250
240

Problema 33. Na proxima figura ABCD e um quadrilatero de area 200cm2 e D, E, F, G s


ao
pontos medios. Determine a
area sombreada.
B
E
H

C
G

D
Problema 34. Na figura abaixo ABCD e um quadrado de lado 6cm e EF e um segmento paralelo
ao lado AD. Sabendo que a
area sombreada e um terco da area do quadrado determine a medida
do segmento EF .
B
C

F
E
A

Problema 35. No trapezio ABCD, AD k BC. A = D = 45 , enquanto B = C = 135 .


Se AB = 6 e a
area de ABCD e 30, ache BC.
Problema 36. Na figura abaixo ABCD e um quadrado de lado 4cm e O e o seu centro. Determine
a area marcada sabendo que o
angulo EOF e reto.
7

C
E
O

D
F
Problema 37. Na figura abaixo ABCD e um ret
angulo de area 11cm2 . Sabemos tambem que

A A = AD, BB = BA, CC = CB e DD = DC. Determine a area do quadrilatero A B C D


B

D
D

Problema 38. Na figura abaixo DEF G e um quadrado de lado 4cm e ABCD um ret
angulo cujos
lados tem medidas 1cm e 4cm. O encontro da reta AC com a reta F G e o ponto H. Determine
a area marcada.
E
F
B

G
A
D
Problema 39. O quadrado ABCD abaixo tem lado 10cm. Sabe-se que P C = QD e que a area
do tri
angulo ABP e 37 da
area do tri
angulo P CQ. Calcule o permetro do quadrilatero AP QD.
Q
D
C
P
A

Problema 40. Um quadrado de lado 5 e dividido em cinco partes de areas iguais usando cortes
paralelos `as suas diagonais. Ache o permetro do pentagono BEF GH.
I
D
C
F
J
G
E
A

B
H
Problema 41. (Teste Rioplatense 2005) Paladino dividiu uma folha de papel quadrada, com 20
cm de lado, em 5 pedacos de mesma
area. O primeiro corte teve incio no centro do quadrado e
prolongou-se ate a fronteira do papel a 7 cm de um canto, como indicado na figura seguinte.
8

Sabendo que o Jo
ao fez todos os cortes em linha recta a partir do centro do quadrado, de que
forma cortou o papel?
Problema 42. Na figuras abaixo ABC e um tri
angulo de area 72cm2 e M, N, P, Q s
ao pontos
medios. Determine a
area da regi
ao sombreada.
A

N
P

C
M
Problema 43. Sejam ABCD um quadrado de lado 12cm, E o ponto medio de DA e F o ponto
medio de BC. Tracamos os segmentos EF , AC e BE, que dividem o quadrado em seis regi
oes.
Calcular a area de cada uma dessas regi
oes.
Problema 44. Seja ABCD um ret
angulo com area 1, e E um ponto sobre CD. Qual e a area
do tri
angulo formado pelos baricentros dos tri
angulos ABE, BCE, e ADE?
Problema 45. Em um parapelogramo ABCD de area igual a 1, seja E o ponto medio do lado
DC, K o ponto de encontro das diagonais BD e AC e L o ponto de encontro de BD com AE.
Ache a area do quadrilatero ELKC.
Problema 46. No tri
angulo ABC sabe-se que C = 90 , AC = 20 e AB = 101. Seja D o ponto
medio de BC. Ache a
area do tri
angulo ADB.
Problema 47. Seja ABCDEF um hex
agono regular de area 1cm2 . Determine a area do tri
angulo
ABC.
Problema 48. Suponha que ABCDE seja um pentagono convexo (nao necessariamente regular)
tal que as areas dos tri
angulos ABC, BCD, CDE, DEA e EAB sa
o iguais a 1. Qual a area do
pentagono?
Problema 49. No tri
angulo ABC, D e o ponto medio de BC, E o ponto medio de AD, F o
ponto medio de BE e G o ponto medio de F C. Calcule a relacao entre as areas dos tri
angulo
ABC e EF G.
Problema 50. (Maio 1996) Um terreno (ABCD) tem forma de trapezio retangular. O angulo
mede 90 . AB mede 30m; AD mede 20m e DC mede 45m.
em A mede 90 e o
angulo em D
Este terreno tem que ser dividido em dois terrenos de area iguais tracando uma paralela ao lado
AD. A que dist
ancia de D deve-se tracar a paralela?

Problema 51. Na figura abaixo ABCD e DEF G s


ao paralelogramos. Alem disso, F , C e G s
ao
colineares. Prove que ambos tem a mesma area.
G

B
F

E
Problema 52. (Maio 2006) Seja ABCD um trapezio de bases AB e CD. Seja O o ponto de
intersecao de suas diagonais AC e BD. Se a area do tri
angulo ABC e 150 e a area do tri
angulo
ACD e 120, calcular a
area do tri
angulo BOC.
Problema 53. (Maio 2006) Um ret
angulo de papel 3cm 9cm e dobrado ao longo de uma reta,
fazendo coincidir dois vertices opostos. Deste modo se forma um pentagono. Calcular sua area.
Problema 54. (Torneio das Cidades 1981) O quadrilatero convexo ABCD esta inscrito em um
crculo de centro O e possui suas diagonais perpendiculares. Prove que a linha quebrada AOC
divide o quadrilatero em duas regi
oes de mesma area.
Problema 55. (Banco IMO) Sejam ABCD um quadrilatero convexo e M e N os pontos medios
dos lados BC e DA, respectivamente. Prove que [DF A] + [CN B] = [ABCD].

Este material faz parte de um conjunto de notas de aulas voltadas para o treinamento de alunos
permitida a copia apenas no caso de uso pessoal.
para competicoes de matem
atica. E
Pode conter falhas.
10

Programa Olmpico de Treinamento


Aula

Curso de Geometria - Nvel 2

Prof. Rodrigo Pinheiro

Introduc
ao
Nesta aula, aprenderemos conceitos iniciais de geometria e alguns teoremas b
asicos que
importante o aluno perceber que os exerccios
utilizaremos em todas as aulas seguintes. E
olmpicos de geometria exigem muita criatividade, mas sem o conhecimento do colegial,
n
ao h
a criatividade que resolva. Vamos assumir alguns conhecimentos b
asicos, que podem
ser encontrados em livros de geometria do colegial. Alguns teoremas enunciados abaixo
ser
ao demonstrados posteriormente, em aulas futuras.
Teorema 1. A soma dos
angulos internos de um tri
angulo e 180 .
Demonstrac
ao.
E

A D

Dado um tri
angulo ABC, tomamos a partir de A uma reta paralela a BC. Pelas
propriedades de paralelismo, temos que EAB = ABC e DAC = ACB. Como
EAD e um
angulo raso, temos que EAD = 180 , podemos concluir que:
ABC + BCA + CAB = 180 .
Teorema 2. A medida de um
angulo externo de um tri
angulo e igual a soma das medidas
dos angulos internos n
ao adjacentes a ele.
Demonstrac
ao.
A
b

POT 2012 - Geometria - Nvel 2 - Aula 1 - Prof. Rodrigo Pinheiro

Como a soma dos


angulos internos e 180 , entao ABC + BCA + CAB = 180 .
Mas na reta BD, temos que BCA + DCA = 180 . Assim,
ABC + BCA + CAB = BCA + DCA,
ABC + CAB = DCA.
Teorema 3. A soma de todos os
angulos internos de um polgono convexo de n lados e

180 (n 2)
Demonstrac
ao. A partir de um vertice do polgono, tracaremos todas as suas diagonais,
ou seja dividimos o polgono em n 2 tri
angulos, portanto, a soma de todos os angulos
internos do polgono e igual a soma de todos os angulos internos de todos os tri
angulos que
e 180 (n 2).
Teorema 4. Dois lados de um tri
angulo s
ao congruentes se, e somente se os angulos opostos
a estes lados s
ao congruentes.
Teorema 5. Em todo tri
angulo is
osceles, a altura, mediana e bissetrizes relativas `a base
s
ao coincidentes.
Teorema 6. Dados dois lados distintos de um tri
angulo, o maior angulo e oposto ao maior
lado.
Demonstrac
ao.
A
b

Suponhamos BC > AC. Seja D o ponto sobre o lado BC tal que AC = CD. Portanto, o tri
angulo ADC e is
osceles. Pelo teorema anterior temos que, CAD = CDA.
Pelo teorema do
angulo externo temos que CDA = ABD + DBA > ABC. Como
BAC > CAD = CDA > ABC, temos que BAC > ABC.
Teorema 7. A soma de dois lados quaisquer de um tri
angulo e maior que o terceiro lado.
Demonstrac
ao.

POT 2012 - Geometria - Nvel 2 - Aula 1 - Prof. Rodrigo Pinheiro

D
b

b
b

Seja D o ponto sobre o prolongamento BC, tal que BD = BA. Sendo assim, o tri
angulo
DBA e is
osceles, portanto, BAD = BDA. Pela figura, percebemos que CAD >
BAD = BDA. Pelo teorema anterior, temos que CD > CA. Como BD = BA e
CB + BD = CD, podemos concluir que CB + BA > AC. Analogamente provamos para
os outros lados.
Problema 1. Paladino, num belo domingo `a tarde decidiu se divertir com a bela geometria.
Ele pegou um tri
angulo, com tres pontos distintos em seu interior, e tracou alguns segmento
entre esses pontos e os vertices do tri
anguo. Ele notou que dividiu a figura toda em
tri
angulos como mostrada abaixo.
B
b

D
A

b
b

Em todos os desenhos onde os segmentos n


ao se cortavam e a figura foi dividida em
tri
angulos, sempre existiam 5 tri
angulos pequenos! Ele provou que em um tri
angulo, se
tomarmos n pontos em seu interior e triagularizarmos a figura unindo os pontos internos
sem cruzamento dos segmentos, sempre dividiremos a figura em 2n+1 tri
angulos pequenos.
Demonstre esta afirmac
ao.
Soluc
ao. Voce ja escutou falar em contagem dupla? Pois e! Voce escutara muito isso em
combinat
oria! Utilizaremos isso tambem em geometria.
3

POT 2012 - Geometria - Nvel 2 - Aula 1 - Prof. Rodrigo Pinheiro

Vamos calcular a soma de todos os


angulos internos de todos os tri
angulos pequenos de
duas formas, essas duas somas tem que ser a mesma. Na primeira forma, digamos que
existem T tri
angulos pequenos, portanto a soma que queremos ser
a 180 T . Na segunda
forma, basta perceber que cada
angulo vertice no interior do tri
angulo contribui para a
soma com 360 , enquanto todos os vertices do tri
angulo contribui com 180 . Temos entao
que 180 T = 360 n + 180 , simplificando, temos que T = 2n + 1.
Problema 2. Paladino ja estava na madrugada de segunda-feira, quando pensou na seguinte
hip
otese: Ser
a que dado um polgono convexo, se dividirmos o polgono em tri
angulos
tracando suas diagonais sem se interceptarem, o n
umero de tri
angulos e sempre o mesmo?
E a, o que voce acha?
Soluc
ao. Se dividirmos o polgono em T tri
angulos ligando suas diagonais sem se interceptarem, a soma de todos os
angulos internos do polgono ser
a 180 T , como a soma sempre

e 180 (n 2), teremos que T = n 2.


Problema 3. Demonstre que se em um polgono convexo de n lados, 4 desses angulos forem
retos, entao esse polgono e um ret
angulo.
Soluc
ao. Obviamente n 4. Suponhamos n > 4. Seja Sn4 a soma dos outros n 4
angulos. Por ser um polgono convexo, cada angulo e menor que 180 . Portanto, Sn4 <
180 (n 4). Sabendo que a soma de todos os angulos internos e
180 (n 2) = 90 + 90 + 90 + 90 + Sn4 < 360 + 180 (n 4)
chegamos que 180 (n 2) < 180 (n 2), que e um absurdo.
Problema 4. No tri
angulo ABC abaixo, BP e bissetriz do angulo B e M e o ponto medio
do lado AC. Se AB = 6 e BC = 10, calcule P M .

B
b

Soluc
ao. Veja a nova figura, onde prolongamos AP ate encontrar o lado BC em Z.

POT 2012 - Geometria - Nvel 2 - Aula 1 - Prof. Rodrigo Pinheiro

B
b

10

Note que no tri


angulo ABZ, o segmento AP e altura e bissetriz. Isso faz com que o
tri
angulo ABZ seja is
osceles! Logo BZ = AB = 6 e portanto:
ZC = BC BZ = 10 6 = 4.
Perceba ainda que como o tri
angulo ABZ e is
osceles, BP e altura, bissetriz e mediana.
Logo P e o ponto medio de AZ. Como M ja e o ponto medio de AC, vemos que P M e a
base media no tri
angulo AZC. Conclusao:
PM =

ZC
= 2.
2

Problema 5. Em um tri
angulo ABC (AB = AC, BAC = 30 ) marcamos um ponto Q no
lado AB e um ponto P na mediana AD, de modo que P C = P Q(Q 6= B). Ache P QC.
Soluc
ao.
A
b

Q
b

E
b

P
b

Como o ABC e is
osceles e BAC = 30 , temos que ABC = ACB = 75 . Chame
BP Q = e P BC = . Como P D e mediana e altura do BP C, entao BCP =
e P C = P B, pela propriedade de
angulo externo, conclumos que CP E = 2. Como
P C = P B = P Q, temos que P BQ e is
osceles, portanto P QB = QBP = . Pela
propriedade de
angulos externos QP E = 2. Da temos que QP C = 2.( + ) = 150 .
Como P Q = P C, temos que P QC e is
osceles, entao P QC = P CQ, concluindo que

P QC = 15 .

POT 2012 - Geometria - Nvel 2 - Aula 1 - Prof. Rodrigo Pinheiro

Problema 6. (OBM - 99) Na figura, os tri


angulos ABC e EGF s
ao equilateros. O permetro
do tri
angulo ABC e 132cm e, alem disso, AE = EC, BD = DC, EF = F C e DG = GE.
B
b

D
b

G
b

Qual o permetro da
area sombreada?
Soluc
ao. Como o ABC e equilatero, entao todos os seus lados s
ao iguais, assim como
seus angulos s
ao todos iguais a 60 . Portanto, AB = BC = CA = 132
3 = 44cm. Como
=
22,
analogamente
EC
=
22.
Dado
que o DEC
BD = DC, temos que BD = DC = 44
2

e is
osceles com um
angulo de 60 , entao ele e equilatero, consequentemente DE = 22.
Sabendo que DG = GE, obtemos que DG = GE = 11. Analogamente obtemos que
EF = F C = 11, assim o permetro da
area sombreada e AB +BD+DG+GF +F E +EA =
44 + 22 + 11 + 11 + 11 + 22 = 121.
Problema 7. Na figura abaixo, ABGH, BCF G e CDEF s
ao quadrados iguais. Determine
a soma ABH + ACH + ADH.
H
b

E
b

Soluc
ao. Observemos a figura abaixo.

POT 2012 - Geometria - Nvel 2 - Aula 1 - Prof. Rodrigo Pinheiro

A
b

C
b

90

D
b

90

I
b

J
b

K
b

Pela propriedade de
angulos alternos internos, temos que ACH = CHE = .
Pela simetria da figura, vemos que N HM = HDA = , da mesma forma vemos que
OCN = HCA = e CN = HC, como ACO = 90 , conclumos que HCN = 90 .
Como o HCN e is
osceles com um angulo de 90 , temos que CHN = 45 . Sabendo
que M HE = 90 , vemos que + + 45 = 90 , como ABH = 45 , temos que:
ADH + ACH + ABH = 90
Problema 8. Dados n pontos A1 , A2 , . . . , An e um crculo unit
ario, prove que e possvel
encontrar um ponto M sobre o crculo tal que M A1 + M A2 + + M An n.
Soluc
ao. Sejam M1 e M2 pontos diametralmente opostos no crculo. Entao M1 Ak +
M2 Ak M1 M2 = 2. Adcionando essas desigualdades para k = 1, 2, 3, . . . , n temos
(M1 A1 + + M1 An ) + (M2 A1 + + M2 An ) 2n.
Portanto, M1 A1 + + M1 An n ou M2 A1 + + M2 An n, entao basta tomar M = M1
ou M = M2 .
Problema 9. Os
angulos BAD e CBA do quadril
atero convexo ABCD s
ao iguais e
BC = 1, AD = 3. Prove que o comprimento de CD e maior que 2.
Soluc
ao. Tomemos um ponto E sobre AD tal que AE = BC. Seja F o ponto de encontro
das retas AD e BC, como BAD = CBA F AB = F BA. Portanto o tri
angulo
F AB e is
osceles, com F A = F B. Assim, F E = F C, pois AE = BC, podemos concluir
entao que o F EC tambem e is
osceles, consequentemente, F EC = F CA. A partir
disto, temos que DEC = GCE. Pela figura, percebemos que GCE > DCE, ou
seja, DEC > DCE, pelo Teorema 6, conclumos que CD > ED = 3 1 = 2.

POT 2012 - Geometria - Nvel 2 - Aula 1 - Prof. Rodrigo Pinheiro

Problemas Propostos
Problema 10. Dados os pontos colineares e consecutivos A, B, C, D e E, tal que AB +
CD = 3 BC e DE = AB. Sendo M o ponto medio de BE, onde M D = 2 e AE = 16,
calcule M C.
Problema 11. Em um reta, temos quatro pontos A, B, C e D que satisfazem as seguintes
relacoes 4 AB BD 2 CD = 4, AB = 3 e AC = 5, calcule AD.
Problema 12. (OBM-2011) Dois tri
angulos equilateros de permetro 36cm cada um s
ao
sobrepostos de modo que sua intersec
ao forme um hexagono com pares de lados paralelos,
conforme ilustrado no desenho. Qual e o permetro desse hexagono?
H
b

J
b

Problema 13. Um trapezio ABCD de bases BC e AD com BC < AD e tal que 2.AB = CD
e BAD + CDA = 120 . Determine os angulos do trapezio ABCD.
Problema 14. No ABC , E e D s
ao pontos interiores aos lados AC e BC, respectivamente. Se AF bissecta CAD e BF bissecta CBE. Prove que AEB +ADB = 2AF B.
C
F

Problema 15. No ABC, um ponto D est


a sobre AC tal que AB = AD. Se ABC
ACB = 30, encontre CBD.
Problema 16. A bissetriz interior de B, e a bissetriz exterior de C do tri
angulo ABC
encontram-se em D. Atraves de D, uma reta paralela a CB encontra AC em L e AB
em M . Se as medidas dos comprimentos de LC e M B do trapezio CLM B s
ao 5 e 7,
respectivamente, encontre a medida de LM . Prove seu resultado.
Problema 17. No ABC, CF e mediana relativa a hipotenusa AB, CE e bissetriz de
ACB, e CD e uma altura realativa `
a AB. Prove que DCE = ECF .
8

POT 2012 - Geometria - Nvel 2 - Aula 1 - Prof. Rodrigo Pinheiro

Problema 18. A medida do segmento de reta P C, perpendicular a hipotenusa AC do


tri
angulo ret
angulo ABC, e igual `
a medida do comprimento BC. Mostre que BP deve ser
perpendicular ou paralelo `
a bissetriz de A.
Problema 19. Prove que para quaisquer tres pontos A, B, C n
os temos AC |AB BC|.
Problema 20. O lado AC do tri
angulo ABC tem comprimento 3.8, e o lado AB tem
comprimento 0.6. Se o comprimento do lado BC e um inteiro, qual e o seu comprimento?
Problema 21. Prove que o comprimento de qualquer lado de um tri
angulo n
ao e maior que
metade do permetro.
Problema 22. A dist
ancia de Leningrado para Moscou e 660 quilometros. De Leningrado
para Likovo s
ao 310 quilometros, de Likovo para Klin s
ao 200 quilometros e de Klin para
Moscou s
ao 150 quilometros. Qual e a dist
ancia entre Likovo e Moscou?
Problema 23. Encontre um ponto dentro de um quadril
atero convexo tal que a soma das
dist
ancias do ponto aos vertices e mnima.
Problema 24. O ponto O e dado no plano do quadrado ABCD. Prove que a dist
ancia
de O ate um dos vertices do quadrado n
ao e maior que a soma das dist
ancias de O ate os
outros tres vertices.
Problema 25. Prove que a soma das diagonais de um quadril
atero convexo e menor que o
permetro mas e maior que o semipermetro.
Problema 26. Prove que a soma das diagonais de um pentagono convexo maior que o
permetro mas e menor que o dobro do permetro.
Problema 27. Um ponto A, dentro de um angulo acut
angulo, e refletido em cada lado do
angulo para obtermos os pontos B e C. O segmento de reta BC intersecta os lados do
angulo em D e E. Mostre que BC/2 > DE
B

D
A
E
C

Problema 28. Prove que a dist


ancia entre quaisquer dois pontos dentro de um tri
angulo
n
ao e maior que que metade do permetro do triangulo.
Problema 29. Se o ponto O est
a dentro do tri
angulo ABC, prove que AO+OC < AB+BC.
9

POT 2012 - Geometria - Nvel 2 - Aula 1 - Prof. Rodrigo Pinheiro

Problema 30. Prove que a soma das dist


ancias de O aos vertices de um dado tri
angulo e
menor que o permetro, se o ponto O est
a dentro do tri
angulo. O que acontece se o ponto
O estiver fora do tri
angulo?
Problema 31. O ponto C est
a dentro de um angulo reto, e os pontos A e B est
ao sobre seus
lados. Prove que o permetro do tri
angulo ABC n
ao e menor que duas vezes a dist
ancia
OC, onde O e o vertice do
angulo reto.

A
O

Problema 32. Prove que o comprimento da mediana AM em um tri


angulo ABC n
ao e
maior que a metade da soma dos lados AB e AC. Prove que a soma dos comprimentos das
tres medianas n
ao e maior que o permetro do tri
angulo.
Problema 33. Prove que um polgono convexo n
ao pode ter tres lados , cada um maior
que a maior diagonal.
Problema 34. Prove que o permetro de um tri
angulo n
ao e maior que 4/3 da soma das
medianas.
Problema 35. Prove que um pent
agono convexo tem tres diagonais que s
ao lados de um
tri
angulo.
Problema 36. Qual e o
angulo formado pelas agulhas do rel
ogio as 12:35?
Problema 37. Na figura, os vertices do ret
angulo P QRS pertencem aos lados do ret
angulo
ABCD. Sendo AP = 3cm, AS = 4cm, SC = 6cm e CR = 8cm, qual a area do ret
angulo
P QRS, em cm2 ?
A
b

b
b

10

POT 2012 - Geometria - Nvel 2 - Aula 1 - Prof. Rodrigo Pinheiro

Problema 38. A piscina do clube que Samuel frequenta tem a forma de um hexagono
(polgono com seis lados), com um
angulo intero de 270 , os demais angulos de 90 e os
quatro lados menores com 12m cada. Samuel costuma nadar pelo meio da piscina, a partir
do ponto A, descrevendo o trajeto representado, na figura, pelo angulo reto ABC, em que
AB = BC. Certo dia, ele nadou por esse trajeto 4 vezes, isto e, foi e voltou 2 vezes.
Quantos metros ele percorreu?
A
b

Problema 39. Uma folha de papel tem 20cm de comprimento por 15cm de largura. Dobramos essa folha ao meio, paralelamente `a sua largura. Em seguida, dobramos a folha
retangular dupla, de modo que dois vertices opostos coincidam. Ao desdobrar a folha, as
marcas da segunda dobra dividem a folha em duas partes, conforme mostrado na figura ao
lado. Qual e a
area da parte escura, em cm2 ?
b

b
b

dobra 1
dobra 2

dobra 2
b

F
b

11

POT 2012 - Geometria - Nvel 2 - Aula 1 - Prof. Rodrigo Pinheiro

Problema 40. Prove que e impossvel desenhar uma estrela (veja a figura abaixo) de modo
que AB < BC, CD < DE, EF < F G, GH < HI e IK < KA.
D

F
G

I
K

Problema 41. Seja ABCD um paralelogramo. O ponto E est


a sobre AD de modo que
AE = CD. Se ABE = 30 encontre o valor do angulo EBC.
Problema 42. Seja ABC um tri
angulo com A = 50 . O lado BC e prolongado em
ambas as direc
oes e sobre os prolongamentos s
ao marcados os pontos P e Q de modo que
P B = BA, CQ = CA e P B + BC + CQ = P Q. Calcule a medida do angulo P AQ.
Problema 43. Seja ABC um tri
angulo ret
angulo em C. Sejam M e N pontos sobre a
hipotenusa tais que BN = BC e AM = AC. Ache o valor do angulo N CM .

12

Programa Olmpico de Treinamento


Aula

Curso de Geometria - Nvel 2

Prof. Rodrigo Pinheiro

Congru
encia de tri
angulos
Defini
c
ao: Um tri
angulo e congruente a outro se, e somente se, e possvel estabelecer
uma correspondencia entre seus vertices de modo que:
1. Seus lados s
ao ordenadamente congruentes aos lados do outro e
2. Seus angulos s
ao ordenadamente congruentes aos angulos do outro.
b

B
b

b
b

Nota
c
ao:

A = D
a = n
ABC DF E B = F e b = p
C = E
c = m
Teorema 1. Dois lados de um tri
angulo s
ao congruentes se,e somente se os angulos opostos
a estes lados s
ao congruentes.
Demonstrac
ao.
A
b

POT 2012 - Geometria - Nvel 2 - Aula 1 - Prof. Rodrigo Pinheiro

() Seja ABC um tri


angulo com AB = AC e M o ponto medio do lado BC. Observe
que ABM AM C, pelo caso LLL, portanto ABC = ACB.
() Seja ABC um tri
angulo com ABC = ACB e M o pe da altura relativa a BC.
Como AM B = AM C = 90 e ABC = ACB, temos entao que M AB = M AC
pois a soma dos
angulos internos de um tri
angulo interno e sempre 180 . Conclumos assim
que AM B AM C pelo caso ALA, consequentemente, AB = AC.
Teorema 2. Em todo tri
angulo is
osceles, a altura, mediana e bissetrizes relativas `a base
s
ao coincidentes.
Demonstrac
ao.
A
b

Seja ABC um tri


angulo com AB = AC e M o ponto medio do lado BC. Observe que
ABM AM C, pelo caso LLL, portanto AM B = AM C e BAM = CAM , por
definicao temos ent
ao que AM tambem e bissetria. Como AM B + AM C = 180 e
AM B = AM C, ent
ao AM B = AM C = 90 , concluindo assim que AM tambem e
altura.
Teorema 3. Seja ABCD um polgono convexo, demonstre que se dois lados opostos s
ao
iguais e paralelos, ent
ao ABCD e um paralelogramo.
Demonstrac
ao. Suponhamos que AB = CD e AB\\CD. Pela propriedade de paralelismo,
temos que: ABD = BDC, concluindo assim que ABD CDB. Da, tiramos que
DBC = BDA, pela propriedade de paralelismo podemos concluir que BC \ \AD, isto
e, ABCD e um paralelogramo.
C

B
b

b
b

Problema 1. Dado um tri


angulo ABC, onde AB = BC. Tomam-se dois pontos M e N
em AB e BC, respectivamente. Demonstre que se M CA = N AC, entao AM = CN .
Soluc
ao.
2

POT 2012 - Geometria - Nvel 2 - Aula 1 - Prof. Rodrigo Pinheiro

B
b

b
b

Dado que o tri


angulo e is
osceles, temos que: M AC = N CA, portanto conclumos
que M AC N CA, da tiramos que M A = N C.
Problema 2. Mostre que se um tri
angulo possui duas altura iguais, entao o tri
angulo e
is
osceles.
Soluc
ao. Seja ABC um tri
angulo onde as alturas AD e CE tem o mesmo comprimento.
Pelo caso especial de congruencia temos que AEC ADC, portanto EAC = DCA,
isto e, ABC is
osceles.
B
b

E
b

b
b

Problema 3. Demonstre que se duas retas AB e AC s


ao duas retas tangentes a um
circunferencia de centro O /nos pontos B e C, respectivamente, entao AB = AC e
OAB = OAC.
Soluc
ao.

POT 2012 - Geometria - Nvel 2 - Aula 1 - Prof. Rodrigo Pinheiro

B
b

A
b

O
b

Observe que BO = OC e AO pertence ao dois tri


angulos ABO e ACO, entao pelo caso
especial de congruencia ABO ACO, assim AB = AC e OAB = OAC.
Problema 4. (Teorema de pitot) Mostre que um quadril
atero pode ser circunscrito a uma
circunferencia se, e somente se, a soma de dois lados opostos for igual `a soma dos outros
dois lados.
Soluc
ao. () Suponha que o quadril
atero ABCD seja circunscrito a uma circunferencia,
e os pontos de tangencia da circunferencia com os lados sejam E, F , G, H, como mostra a
figura abaixo.
b

F
B

b
b

b
b

Pelo problema anterior, vemos que: AH = AE; BE = BF ; CF = CG; GD = HD.


Portanto, AE + BE + CG + GD = BF + CF + +HD + AH, isto implica dizer que:
AB + CD = BC + AD.
() Suponha que ABCD seja um quadril
atero tal que AB + CD = BC + AD e n
ao
seja circunscritvel.

POT 2012 - Geometria - Nvel 2 - Aula 1 - Prof. Rodrigo Pinheiro

y
E

z+w

w-k
b

wI

Sejam AO e BO as bissetrizes internas dos angulos DAB e ABC. Tomamos E, F e


H como sendo os pes das alturas de O aos lados AB, BC e AD, respectivamente. Pelo caso
especial de congruencia temos que AOH AOE e BOE BOF , assim sendo,
AE = AH = x e BE = BF = y. Defina CF = z e HD = w. Pela hip
otese, temos que:
(x + y) + CD = (y + z) + (x + w) CD = z + w
Como CD n
ao e tangente `
a circunferencia pois, caso contr
ario, o quadril
atero seria circunscritvel. Tomamos G tal que CG seja tangente a circunferencia e defina CG AD = I,
perceba que pelo problema anterior, temos: CG = CF = z e GI = HI = k. Dessa maneira, ID = w k, mas analisando o tri
angulo CID isso e um absurdo pois CI + ID = CD
e CID e um tri
angulo. Ent
ao se a soma dos lados opostos de um quadril
atero forem iguais,
entao ele ser
a circunscritvel.
Problema 5. S
ao construdos exteriormente ao ABC, os tri
angulos equilateros ABM ,
BCN , ACP . Prove que N A = BP = CM .
Soluc
ao.
b

B
b

60

60

b
b

Primeiramente, construmos somente os tri


angulos equilateros ABM e BCN . Dessa
forma, temos que: AB = M B e BC = BN e M BA = CBN = 60 . Assim sendo,
5

POT 2012 - Geometria - Nvel 2 - Aula 1 - Prof. Rodrigo Pinheiro

temos que M BC ABN pelo caso LAL, pois AB = M B; BC = BN e


M BA = CBN , portanto AN = CM . Analogamente, provamos que AN = CM = BP .

Problemas Propostos
Problema 6. ABCD e um paralelogramo e ABF e ADE s
ao tri
angulos equilateros construdos exteriormente ao paralelogramo. Prove que F CE tambem e equilatero.
Problema 7. Quatro quadrados s
ao construdos exteriormente nos lados de um paralelogramo. Mostre que os centros destes quadrados tambem formam um quadrado
Problema 8. Um hexagono convexo ABCDEF est
a circunscrito a uma circunferencia.
Mostre que AB + CD + EF = BC + DE + F A.
Problema 9. Na figura, ABCD e AEF G s
ao quadrados. Mostre que BE = DG.
C
b

B
b

G
b

b
b

Problema 10. (R
ussia 1946) Dados tres pontos A, B, C sobre uma reta l, s
ao construdos
tri
angulos equilateros ABC1 e BCA1 em um mesmo semi-plano com respeito a l. Se M ,
N s
ao os pontos medios de AA1 , CC1 , respectivamente, mostre que o tri
angulo BM N e
equilatero.
Problema 11. (Inglaterra/95) Seja ABC um tri
angulo ret
angulo em C. As bissetrizes
internas de BAC e ABC encontram BC e CA em P e Q, respectivamente. Sejam M e N
os pes das perpendiculares a partir de P e Q ate AB, respectivamente. Encontre a medida
do angulo M CN
Problema 12. (Pol
onia/92) Os segmentos AC e BD intersectam-se no ponto P de modo
que P A = P D, P B = P C. Seja O o circucentro do tri
angulo P AB. Prove que as retas
OP e CD s
ao perpendiculares.
Problema 13. Prove que se em um tri
angulo ABC, a mediana AM e tal que BAC e
dividido na raz
ao 1 : 2, e D est
a sobre AM , com M entre A e D, tal que DBA = 90 ,
entao AC = AD
2 . Dica: Escolha P sobre AD tal que AM = M P .

POT 2012 - Geometria - Nvel 2 - Aula 1 - Prof. Rodrigo Pinheiro

Problema 14. Em um quadrado ABCD, M e o ponto medio de AB. Uma reta perpendicular a M C em M toca AD em K. Prove que BCM = KCM .
Problema 15. Dado um quadrado ABCD com EDC = ECD = 15 , prove que ABE
e equilatero.
Problema 16. Dado um tri
angulo qualquer ABC, D, E e F s
ao pontos medios dos lados
AC, AB e BC, respectivamente. Sendo BG a altura do tri
angulo ABC. Prove que
EGF = EDF .
Problema 17. (congruencia) No losango ABCD com BAD = 60 , tomamos pontos F ,
H e G nos lados AD, DC e na diagonal AC, respectivamente, de modo que DF GH seja
um paralelogramo. Prove que o tri
angulo BF H e equilatero.
Problema 18. (congruencia) Seja ABCD um paralelogramo. A bissetriz de BAD corta
BC em M e o prolongamento de CD em N . Se O e o circuncentro do tri
angulo M CN ,
mostre que B, O, C, D s
ao concclicos.
Problema 19. (congruencia) Sejam ABC um tri
angulo, D um ponto sobre o prolongamento da semi-reta BC a partir de B tal que BD = BA e M o ponto medio de AC. A
bissetriz do angulo ABC corta DM em P . Mostre que BAP = ACB.
Problema 20. (congruencia) Seja ABCDE um pentagono com AE = ED, AB+CD = BC
e BAE + CDE = 180 . Prove que AED = 2BEC.
Problema 21. (Congruencia) Sejam ABC um tri
angulo de circuncrculo 1 , O o circuncentro de ABC e 2 o ex-incrculo relativo ao lado BC. Se M, N, L s
ao os pontos de tangencia
de 2 com as retas BC, AC, AB e os raios de 1 e 2 s
ao iguais, mostre que O e o ortocentro
do tri
angulo M N L.

Programa Olmpico de Treinamento


Aula

Curso de Geometria - Nvel 2

Prof. Rodrigo Pinheiro

Teorema de Tales e Aplicac


oes

Divis
ao Harm
onica
Dizemos que os pontos M e N dividem harmonicamente o segmento AB quando
NA
NB .
N

M
b

MA
MB

B
b

MA
NA
Como M
ao. Estes
B = k = N B , os pontos M e N dividem o segmento AB na mesma raz
pontos s
ao chamados conjugados harmonicos de AB na razao k.

Problema 1. Prove que em uma divisao harmonica com k > 1, temos que:
1
1
2
=
+
AB
AM
AN
Soluc
ao.
A
b

M
b

B
b

N
b

MA
NA
AM
AN
=

=
MB
NB
AB AM
AN AB
AM (AN AB) = AN (AB AM ) AM.AN AM.AB = AN.AB AM.AN

1
1
2
=
+
AB
AM
AN
Problema 2. Prove que em uma divisao harmonica com k < 1, temos que:
2.AM.AN = AN.AB + AM.AB
1
1
2
=

AB
AM
AN

Problema 3. Sendo O o ponto medio de AB em uma divisao harmonica, prove que:


OA2 = OM.ON
Soluc
ao.

POT 2012 - Geometria - Nvel 2 - Aula 3 - Prof. Rodrigo Pinheiro

A
b

NA
OM + OA
ON + OA
MA
=

=
MB
NB
OB OM
ON OB
Como OB = OA, temos que:
(OM + OA)(ON OA) = (ON + OA)(OA OM )

OM.ON OM.OA + ON.OA OA2 = ON.OA OM.ON + OA2 OM.OA


OA2 = OM.ON

Problema 4. Sejam M e N conjugados harmonicos na razao k > 1 do segmento AB = l.


Qual e a dist
ancia entre os divisores harmonicos de AB?
Soluc
ao.

A
b

M
b

B
b

MN = x

AB = l M B = a

Portanto,

N
b

BN = b

1a
1
MA
=k
= k 1 a = a.k a =
MB
a
k+1
NA
1+b
1
=k
= k 1 + b = b.k a =
NB
b
k1
x=a+bx=

2k.l
k2 1

Problema 5. Sejam M e N conjugados harmonicos na razao k < 1 do segmento AB = l.


Qual e a dist
ancia entre os divisores harmonicos de AB?
Teorema de Tales
Teorema 1. Se um feixe de retas paralelas e cortado por duas retas transversais, r e s,
entao a razao entre quaisquer dois segmentos determinados em r e igual a `a razao entre os
segmentos correspondentes em s.

POT 2012 - Geometria - Nvel 2 - Aula 3 - Prof. Rodrigo Pinheiro

E
b
b

F
b
b

c
C

G
b
b

D
b
b

Se b, c, d e e s
ao retas paralelas cortadas pelas transversais r e s, entao:
BC
CD
AC
BD
AD
AB
=
=
=
=
=
EF
FG
GH
EG
FH
EH
Teorema da bissetriz interna
Teorema 2. A bissetriz interna de um angulo interno de um tri
angulo determina sobre o
lado oposto ao
angulo dois segmentos proporcionais aos lados adjacentes.
Assim, por exemplo, a bissetriz interna do angulo A do tri
angulo ABC divide o lado
BC em dois segmentos x e y tais que:
A
b

B
b

y
b

x
y
=
c
b
Demonstrac
ao. Tracamos por C um reta paralela a bissetriz interna AD, e seja E a intersecao dessa paralela com o prolongamento da reta AB. Pela propriedade de paralelismo,
temos que BAD = BEC e DAC = ACE, como AD e bissetriz, conclumos que
ACE = AEC, portanto ACE e isosceles, com AE = AC = b. Sendo assim, pelo
teorema de tales, temos que:
x
y
=
c
b

POT 2012 - Geometria - Nvel 2 - Aula 3 - Prof. Rodrigo Pinheiro

E
b

A
b

B
b

y
b

Teorema da bissetriz externa


Teorema 3. A bissetriz externa de um angulo de um tri
angulo determina sobre o lado
oposto ao angulo dois segmentos proporcionais aos lados adjacentes.
Assim, por exemplo, a bissetriz externa do angulo A do tri
angulo ABC determina sobre
o lado BC dois segmentos x e y tais que:
y
x
=
c
b
A
b

c
D
b

B
b

C
b

y
Demonstrac
ao. Analogo ao teorema da bissetriz interna.
Problema 6. Seja ABC um tri
angulo tal que AB = 6, AC = 7 e BC = 8. Tome S AC
onde BS e bissetriz do
angulo B e tome I BS tal que CI e bissetriz do angulo C,
determine a razao BI
IS .
Soluc
ao.

POT 2012 - Geometria - Nvel 2 - Aula 3 - Prof. Rodrigo Pinheiro

A
b

S
b

I
b

C
b

Seja SC = x. Temos entao que AS = 7 x. Pelo teorema da bissetriz interna no


tri
angulo ABC temos que:
6
AS
7x
=
=
6x = 56 8x x = 4
8
SC
x
Pelo teorema da bissetriz interna no tri
angulo BSC, temos que:
8
BI
= =2
IS
x
Problema 7. Seja ABC um tri
angulo ret
angulo em A, com hipotenusa BC = 30 e AC
AB = 6. Calcule o comprimento da bissetriz BS.
Soluc
ao. Seja AC = x e AB = y, entao temos que: x y = 6 e x2 + y 2 = 900 pelo teorema
de pit
agoras. Isolando x na primeira equacao e substituindo na segunda, teremos que:
(y + 6)2 + y 2 = 900 y 2 + 6y 432 = 0
onde teremos as razes 18 e 24, portanto, y = 18, assim x = 24, como BS e bissetriz, pelo
teorema da bissetriz interna, teremos que:
18
AS
=
AS = 9
30
24 AS

Pelo teorema de pit


agoras, teremos que: BS 2 = 182 + 92 BS = 9. 5.
A
b

S
b

Problema 8. Sendo AS e AP bissetrizes dos angulos internos e externos em A, determine


o valor de CP , sabendo que BS = 8 e CS = 6.

POT 2012 - Geometria - Nvel 2 - Aula 3 - Prof. Rodrigo Pinheiro

Problema 9. Seja ABC um tri


angulo de lados a, b, c opostos aos vertices A, B, C, respecab
ac
e CD = b+c
.
tivamente. Se D BC tal que AD e bissetriz interna, mostre que BD = b+c
Problema 10. O incentro do tri
angulo ABC divide a bissetriz interna do angulo A na razao
AI : ID = 2 : 1. Mostre que os lados do tri
angulo estao em progress
ao aritmetica.
Problema 11. (Crculo de Apolonius) Seja k um n
umero real positivo, k 6= 1. Mostre que
o lugar geometrico dos pontos P do plano tais que P A : P B = k e uma circunferencia cujo
centro pertence `
a reta AB.
Problema 12. Em um tri
angulo ABC, BC = 7,
ao lado a sabendo que ela e m
axima.

AB
BC

= 3. Calcule o valor da altura relativa

Problema 13. Em um tri


angulo ABC, BC = 16 e a altura relativa ao lado BC e 8. Calcule
sabendo
que
ela e m
axima.
a razao AB
AC
Problema 14. Os comprimentos dos lados de um tri
angulo s
ao os inteiros x 1, x e x + 1
e seu maior angulo e o dobro do menor. Determine o valor de x.
Problema 15. Em um tri
angulo ABC, de lados AB = 12, AC = 8 e BC = 10, encontre o
maior segmento que a bissetriz interna de A determina sobre BC.

Programa Olmpico de Treinamento


Curso de Geometria - Nvel 2

Aula

Prof. Rodrigo Pinheiro

Semelhan
ca de Tri
angulos
Dois tri
angulos s
ao semelhantes se, e somente se, possuem angulos ordenadamente congruentes e os lados homologos proporcionais. Sendo k a razao entre os lados homologos,
k e chamado de razao de semelhanca. Observe que se k = 1, entao os tri
angulos s
ao
congruentes. Igualmente a congruencia de tri
angulos, temos os casos de semelhanca.
1 Caso: Se dois tri
angulos tem congruentes dois a dois os tres angulos internos, entao
esses dois tri
angulos s
ao semelhantes.
2 Caso: Se dois tri
angulos tem dois pares de lados proporcionais e os angulos compreendidos entre eles congruentes, entao esses dois tri
angulos s
ao semelhantes.

3 Caso: Se dois tri


angulos tem os tres lados correspondentes proporcionais, entao
esses tri
angulos s
ao semelhantes.
Teorema 1. Se uma reta e paralela a um dos lados de um tri
angulo, entao o tri
angulo que
ele determina e semelhante ao primeiro.
Demonstrac
ao. Basta ver que eles tem os mesmo angulos por paralelismo.
Observa
c
ao 1: Se dois tri
angulos s
ao semelhantes na razao k, entao tambem e igual
a k:
a razao entre as alturas
a razao entre as medianas
a razao entre as bissetrizes, etc.
Observa
c
ao 2: A razao entre as
areas de dois tri
angulos semelhantes (na razao k) e
igual a k 2 .
Problema 1. As bases de um trapezio medem 12m e 18m e os lados oblquos `as bases
medem 5m e 7m. Determine o permetro do tri
angulo menor que obtemos ao prolongar os
lados oblquos `
as bases.
Soluc
ao.

POT 2012 - Geometria - Nvel 2 - Aula 4 - Prof. Rodrigo Pinheiro

12
7

5
b

18
Como as bases do trapezio s
ao paralelas, teremos que os dois tri
angulos s
ao semelhantes,
portanto:
x
12
y
=
=

x+5
18
7+y
18x = 12x + 90 e 18y = 12y + 84, entao: x = 15 e y = 14, assim, o permetro sera
15 + 12 + 14 = 41
Problema 2. Num tri
angulo ABC, os lados medem AB = 4cm, BC = 5cm e AC = 6cm.
Calcule os lados de um tri
angulo semelhante a ABC cujo permetro mede 20cm.
Soluc
ao. Sejam x, y e z os lados do tri
angulo. Como os dois tri
angulos s
ao semelhantes,
entao:
y
z
x+y+z
20
x
= = =
=

4
5
6
4+5+6
15
x = 16/3, y = 20/3 e z = 8.
Problema 3. Seja ABC um tri
angulo eq
uilatero de lado 20. Uma reta passando pelo ponto
medio M do lado AB corta o lado AC no ponto N e o prolongamento do lado BC no ponto
P , de tal modo que CP = 12. Determine o comprimento de CN e N A.
Soluc
ao.
A
b

M
b

10
B
b

P
b

10

12

Tomemos O como sendo o ponto medio de BC. Como M O e base media, temos que
M O = 10 e M O e paralelo a AC, assim o tri
angulo N CP e semelhante a M OP , entao:
12
60
x
=
x=
10
22
11
2

POT 2012 - Geometria - Nvel 2 - Aula 4 - Prof. Rodrigo Pinheiro

.
Problema 4. Sejam D e E pontos sobre os lados AB e AC do tri
angulo ABC. Sendo BC =
22cm, AD = 8cm, DB = 3cm, AE = 5cm e ABE = ACD, calcule o comprimento de
DE.
Problema 5. Considere a circunferencia circunscrita ao tri
angulo ABC. Seja AE um
di
ametro dessa circunferencia e AD a altura do tri
angulo. Sendo AB = 6cm, AC = 10cm
e AE = 30cm, calcule AD.
Problema 6. Calcule o raio da circunferencia circunscrita ao tri
angulo ABC sabendo que
AB = 4, AC = 6 e a altura AH relativa ao lado BC e igual a 3.
Problema 7. (Base media de um tri
angulo) Sejam M e N os pontos medios, respectivamente, dos lados AB e AC do tri
angulo ABC. O segmento M N e chamado de base media,
relativa ao lado BC. Mostre que M N e paralela a BC e que M N = BC
2 .
Problema 8. Sejam ABCD um trapezio com AB paralelo a CD, M e N os pontos medios
dos lados oblquos AD e BC. Use o exerccio anterior para concluir que M N = AB+CD
.
2
Problema 9. No tri
angulo ABC, a bissetriz interna do angulo A encontra BC em D. A
reta por B, perpendicular a AD, encontra AD em E. Seja M o ponto medio do lado BC.
Se AB = 26, BC = 28 e AC = 30, ache os comprimentos de DM e M E.
Problema 10. No tri
angulo ABC, Z e um ponto sobre o lado AB. Uma reta por A e
paralela a CZ, encontra BC em X; uma reta por B e paralela a CZ encontra AC em Y .
1
1
1
+ BY
= CZ
.
Mostre que AX
Problema 11. Seja P um ponto no interior do tri
angulo eq
uilatero ABC. Por P tracamos
tres retas paralelas aos lados de ABC, determinando tres tri
angulos menores, de areas 4,
9 e 49. Determine a
area do tri
angulo ABC.
Problema 12. Duas circunferencias c1 e c2 interceptam-se em dois pontos A e B. Construa
um segmento P Q pelo ponto B com uma extremidade sobre c1 e a outra sobre c2 de modo
que P Q seja o maior possvel.
Problema 13. Os lados de um tri
angulo ABC medem AB = 6, AC = 9 e BC = 11. Se
J e o ponto de tangencia do crculo ex-inscrito relativo ao lado AB. Sabendo que JL e
paralelo a BC (com L sobre o lado AC), determine o comprimento do segmento AL.
Problema 14. Seja C1 a circunferencia inscrita num tri
angulo ABC cujo permetro mede
18cm. Uma tangente a C1 e paralela a um dos lados do tri
angulo e mede 2cm. Quais os
possveis valores do lado ao qual esta tangente e paralela?

Polos Olmpicos de Treinamento


Aula

Curso de Geometria - Nvel 2

Prof. Ccero Thiago

Algumas propriedades importantes de tri


angulos
Propriedade 1. Num tri
angulo ret
angulo ABC, a mediana BM relativa `a hipotenusa
mede metade da hipotenusa AC.
A

D
M

Demonstrac
ao. Seja D o ponto sobre o prolongamento da mediana BM tal que BM =
M D. Os tri
angulos AM B e CM D s
ao congruentes, pelo caso LAL. Da, AB = CD e
BAM = DCM , ou seja, AB e CD s
ao segmentos iguais e paralelos e portanto
ABC = DCB = 90 .
Assim, os tri
angulos ABC e DCB s
ao congruentes, pelo caso LAL, e portanto
AC

BD = AC = 2 BM = AC = BM =
2
Afirmac
ao. Uma base media de um tri
angulo e um segmento que une os pontos medios de
dois de seus lados.
Assim, todo tri
angulo possui exatamente tres bases medias.
Propriedade 2. Sejam ABC um tri
angulo e M , N os pontos medios dos lados AB, AC,
respectivamente. Ent
ao
BC
M N k BC e M N =

2
A
M

POT 2012 - Geometria - Nvel 2 - Aula 5 - Prof. Ccero Thiago

Demonstrac
ao. Inicialmente, prolonguemos a base media M N ate um ponto P tal que
M N = N P . Em seguida, construmos o tri
angulo CN P . Note que os tri
angulos AN M e
CN P s
ao congruentes, pelo caso LAL. Da, CP = AM e M AN = P CN e portanto
CP k AM

CP k BM.

Assim, M BCP e um paralelogramo, pois CP e BM s


ao segmentos paralelos e iguais. Mas
entao M P k BC e
M P = BC

2M N = BC

MN =

BC

Afirmac
ao. A base media de um trapezio e o segmento que une os pontos medios de seus
lados n
ao paralelos.
Propriedade 3. Seja ABCD um trapezio de bases AB e CD, e sejam M e N os pontos
medios dos lados BC e AD, respectivamente. Entao,
M N k AB, M N k CD

N
b

MN =

AB + CD
.
2

B
b

C
b

facil
Demonstrac
ao. Inicialmente, prolonguemos AM ate encontrar DC no ponto E. E
ver que
ABM CM E (ALA) AB = CE.
Portanto, M N e base media do tri
angulo ADE. Assim,
M N k BE M N k DC M N =
Finalmente, M N =

DE
.
2

DC + CE
DC + AB
=
.
2
2

Problema 1. (OBM) Considere um tri


angulo acut
angulo ABC com BAC = 30 . Sejam
B1 , C1 os pes das alturas relativas aos lados AC, AB, respectivamente, e B2 , C2 os pontos
medios dos lados AC, AB, respectivamente. Mostre que os segmentos B1 C2 e B2 C1 s
ao
perpendiculares.

POT 2012 - Geometria - Nvel 2 - Aula 5 - Prof. Ccero Thiago


A

C2

B2

B1

C1
B

Soluc
ao.
Seja O a intersec
ao entre B1 C2 e B2 C1 . O segmento B1 C2 e uma mediana do tri
angulo
ret
angulo AB1 B e portanto
AC2 = B1 C2

C2 B1 A = BAB1 = 30 .

Analogamente, AC1 B2 = 30 . Da,


BC2 B1 = C2 B1 A + BAB1 = 60
e portanto
C1 OC2 = 180 BC2 B1 AC1 B2 = 90 .
Problema 2. Sejam ABC um tri
angulo e M o ponto medio do lado BC. Se D, E s
ao os
pes das alturas relativas aos lados AC, AB, respectivamente, prove que M E = M D.
Soluc
ao.
A
D

Note que M E e mediana relativa `


a hipotenusa do tri
angulo BEC. Da,
M E = BM = CM
e, analogamente,
M D = BM = CM.
Assim, M E = M D.

POT 2012 - Geometria - Nvel 2 - Aula 5 - Prof. Ccero Thiago

Problema 3. Dado um quadril


atero ABCD, prove que os pontos medios M, N, P, Q dos
lados AB, BC, CD, DA formam um paralelogramo.
Soluc
ao.
A
M

D
N

P
C

Temos
Tri
angulo ABC: M N k AC e M N = AC/2.
Tri
angulo DAC: P Q k AC e P Q = AC/2.
Assim, M N k P Q e M N = P Q, isto e, M N P Q e paralelogramo.
Problema 4. Sejam ABC um tri
angulo e M o ponto medio de BC. Se AM = BM = CM ,
prove que BAC = 90 .
Problema 5. (Torneio das Cidades) Sejam ABCD um paralelogramo, M o ponto medio
de CD e H o pe da perpendicular baixada de B a AM . Prove que BCH e um tri
angulo
is
osceles.
Problema 6. Em um tri
angulo ABC, ret
angulo em A e is
osceles, sejam D um ponto no
lado AC (A 6= D 6= C) e E o ponto no prolongamento de BA tal que o tri
angulo ADE e
is
osceles. Se P e o ponto medio de BD, R o ponto medio de CE e Q a intersecao entre
ED e BC, prove que o quadril
atero ARQP e um quadrado.
Problema 7. Seja ABC um tri
angulo acut
angulo tal que B = 2C, AD e perpendicular
a BC, com D sobre BC, e E o ponto medio de BC. Prove que AB = 2DE.
Problema 8. (China) Seja ABCD um trapezio, AD//BC, B = 30o , C = 60o , E, M, F, N
os pontos medios de AB, BC, CD, DA respectivamente. Se BC = 7, M N = 3, determine
a medida de EF .

POT 2012 - Geometria - Nvel 2 - Aula 5 - Prof. Ccero Thiago

Problema 9. (China) Seja ABCD um trapezio, AB//CD, DAB = ADC = 90o , e o


3
tri
angulo ABC e equilatero. Se a base media do trapezio EF = a, determine o compri4
mento da menor base AB, em func
ao de a.
Problema 10. (Moscou) Seja ABCD um quadril
atero convexo e O um ponto em seu inteo
rior tal que AOB = COD = 120 , AO = OB, CO = OD. Sejam K, L, M os pontos
medios de AB, BC, CD respectivamente, prove que KLM e equilatero.
Problema 11. (OBM) Num quadril
atero convexo, a reta que passa pelos pontos medios
de dois lados opostos forma
angulos iguais com ambas as diagonais. Mostre que as duas
diagonais tem o mesmo comprimento.
Problema 12. Se um segmento paralelo a um lado de um tri
angulo tem uma extremidade
no ponto medio de um lado e a outra extremidade no terceiro lado, prove que esta extremidade e ponto medio do terceiro lado.
Problema 13. (OBM) No tri
angulo ABC, D e ponto medio de AB e E ponto sobre o lado
BC tal que BE = 2 EC. Sabendo que ADC = BAE, calcule o valor de BAC.
Problema 14. (Austr
alia) Sejam ABC um tri
angulo e P um ponto em seu interior de modo
que P AC = P BC. Se L, M s
ao os pes das perpendiculares por P aos lados BC, AC,
respectivamente, e D e o ponto medio de AB, prove que DL = DM .
Problema 15. (Romenia) Sejam ABC um tri
angulo is
osceles com AB = AC, D o ponto
medio de BC, M o ponto medio de AD e N a projecao de D sobre BM . Prove que
AN C = 90 .
Problema 16. (Eslovenia) Seja ABCD um trapezio, com AB paralelo a CD. Sabendo que
a dist
ancia entre os pontos medios das bases e igual `a dist
ancia entre os pontos medios das
diagonais, prove que DAC e DBC s
ao angulos obtusos.
Problema 17. Em um tri
angulo is
osceles ABC, com AB = BC, sejam K, L pontos sobre
AB, BC, respectivamente, tais que AK + LC = KL. A reta paralela a BC passando pelo
ponto medio M de KL intersecta AC em N . Ache a medida de KN L.
Problema 18. Sejam ABC um tri
angulo e D, E, F os pontos medios de BC, CA, AB,
respectivamente. Prove que
DAC = ABE AF C = ADB.
5

POT 2012 - Geometria - Nvel 2 - Aula 5 - Prof. Ccero Thiago

Problema 19. Seja ABCD um trapezio com bases AB = a e CD = b. Sejam tambem M , N


os pontos medios dos lados AB, CD, respectivamente. Sabendo que DAB+ABC = 90 ,
determine o comprimento de M N .
Problema 20. (Cone Sul) Seja ABC um tri
angulo acut
angulo e sejam AN , BM e CP as
alturas relativas aos lados BC, CA e AB, respectivamente. Sejam R, S as projecoes de N
sobre os lados AB, CA, respectivamente, e Q, W as projecoes de N sobre as alturas BM ,
CP , respectivamente.
(a) Mostre que R, Q, W , S s
ao colineares.
(b) Mostre que M P = RS QW .
Problema 21. (TST Brasil) Sejam Q o ponto medio do lado AB de um quadril
atero inscritvel ABCD e S a intersec
ao das diagonais AC e BD. Sejam P , R as projecoes ortogonais
de S sobre AD, BC, respectivamente. Prove que P Q = QR.
Bibliografia
Lecture Notes on Mathematical Olympiad Courses
For Junior Section, vol. 1
Xu Jiagu

Polos Olmpicos de Treinamento


Aula

Curso de Geometria - Nvel 2

Prof. Ccero Thiago

Quadril
ateros Not
aveis
1. Paralelogramo: Um quadril
atero convexo e dito um paralelogramo quando possuir
lados opostos paralelos.
Teorema 1. Um quadril
atero convexo e paralelogramo se, e somente se:

a) Angulos
opostos s
ao iguais;
b) Lados opostos s
ao iguais;
c) Diagonais cortam - se em seus pontos medios;
Demonstrac
ao.
(a)
A

B E

Suponhamos inicialmente que ABCD e um paralelogramo e seja E um ponto no prolon facil perceber que DAB = CBE, pois s
gamento do lado AB. E
ao angulos correspondentes de retas paralelas. Por outro lado CBE = DCA, pois s
ao angulos alternos
internos. Portanto, DAB = DCA. Com o mesmo raciocnio podemos provar que
ADC = ABC.
Reciprocamente, seja ABCD um quadril
atero convexo tal que DAB = DCB e ADC =
ABC. Sabemos que DAB + DCB + ADC + ABC = 360 e com isso DAB +
ABC = 180 e DCB + ADC = 180 . Por outro lado, ABC + CBE = 180 Conclumos entao, que DAB = CBE e, com isso, AD k BC. Com o mesmo raciocnio
podemos provar que AB k CD. E com isso ABCD e um paralelogramo.
(b)

POT 2012 - Geometria - Nvel 2 - Aula 6 - Prof. Ccero Thiago


A
B

facil perceber, que DCA = BAC, pois s


Seja ABCD um paralelogramo. E
ao angulos
alternos internos. Da mesma forma, DAC = BCA. Com isso, DAC ABC, pelo
caso A.L.A. Portanto, AD = BC e AB = CD.

Reciprocamente, seja ABCD um quadril


atero convexo tal que AD = BC e AB = CD. E
facil perceber que, DAC ABC, pelo caso L.L.L. Portanto, ADC = ABC. De
maneira similar, podemos provar que DAB = DCB. Usando o fato provado no item
(a), podemos concluir que ABCD e um paralelogramo.
(c)
A

B
M

Seja ABCD um paralelogramo e seja M o ponto de encontro de suas diagonais. J


a sabemos, pelos itens anteriores, que os
angulos e lados opostos s
ao iguais. Por outro lado,
DAC = BCA, pois s
ao
angulos alternos internos. Pelo mesmo motivo ADB = CBD
e com isso ADM CBM , pelo caso A.L.A. Portanto, AM = M C e DM = M B.
Reciprocamente, seja ABCD um quadril
atero convexo tal que suas diagonais se intersec facil perceber, que
tam em seus pontos medios, ou seja, AM = M C e DM = M B. E
DM A = CM B, pois s
ao
angulos opostos pelo vertice. Entao, ADM CBM , pelo
caso L.A.L. Portanto, AD = BC. De maneira similar, podemos provar, que AB = CD.
Usando agora, o que foi provado no item (b), conclumos que ABCD e um paralelogramo.
2. Trap
ezio: Um quadril
atero convexo e trapezio se, e somente se, possui dois lados
paralelos. Um trapezio ser
a dito is
osceles se os lados n
ao paralelos forem iguais e ser
a dito
ret
angulo se um dos
angulos da base for reto.
Teorema 2. Os
angulos de cada base de um trapezio is
osceles s
ao congruentes e as diagonais tambem s
ao congruentes.
Demonstrac
ao. Sejam AE e BF alturas do trapezio. Como AB e CD s
ao paralelos entao AE = BF . Se AD = BC entao ADE BCF pelo caso especial para
2

POT 2012 - Geometria - Nvel 2 - Aula 6 - Prof. Ccero Thiago

tri
angulos ret
angulos cateto - hipotenusa. Com isso, ADC = BCD. Temos tambem
que ADC BCD pelo caso L.A.L, portanto AC = BD.
A

3. Losango: Paralelogramo com todos os lados iguais.


Teorema 3. As diagonais do losango s
ao perpendiculares.
Demonstrac
ao. Como o losango e um paralelogramo entao as diagonais cortam - se em
seus ponyos medios, ou seja, AM = M C e BM = M D. Com isso, AM B AM D,
pelo caso L.L.L, portanto AM B = AM D. Como AM B + AM D = 180 , entao
AM B = AM D = 90 .

A
b

B
b

4. Ret
angulo: Paralelogramo com quatro angulos retos.
Teorema 4. A diagonais de um ret
angulo s
ao iguais.

POT 2012 - Geometria - Nvel 2 - Aula 6 - Prof. Ccero Thiago


facil ver que ADC BCD pelo caso L.A.L. Portanto, AC = BD.
Demonstrac
ao. E
B

A
b

5. Quadrado: Ret
angulo com os quatro lados iguais.
Exerccios Resolvidos
1. Se dois segmentos s
ao iguais e paralelos, entao suas extremidades s
ao os vertices de
um paralelogramo.
Soluc
ao.
A
B
M
D

Sejam AD e BC os segmentos iguais e paralelos. Vamos entao construir os segmen facil perceber que DAC = BCA,
tos DB e AC, que se intersectam em M . E
pois s
ao
angulos alternos internos. Pelo mesmo motivo, ADC = CBD. Portanto,
ADM BCM , pelo caso A.L.A. Usando o resultado provado no item (c) do
teorema (1), provamos que ABCD e um paralelogramo.
2. Mostre que se por um ponto na base de um tri
angulo is
osceles tracamos retas paralelas aos lados congruentes, ent
ao se forma um paralelogramo cujo permetro e igual
a soma dos comprimentos dos lados congruentes.
Soluc
ao. Seja D um ponto da base do tri
angulo is
osceles ABC e sejam DE e DF

os segmentos paralelos aos lados iguais. E facil ver que AF DE e um paralelogramo


pois DE k AC e DF k AB. Portanto, AF = DE, AE = DF e os tri
angulos BDE
facil perceber que o tri
e CDF s
ao is
osceles assim BE = DE e DF = CF . E
angulo

POT 2012 - Geometria - Nvel 2 - Aula 6 - Prof. Ccero Thiago

ABC e o paralelogramo AF DE possuem o mesmo permetro.


b

E
b

3. (OCM) Sejam AB e CD as bases de um trapezio tal que a base menor CD e igual `a


soma dos lados n
ao paralelos do trapezio. Se E e um ponto de CD e EA e a bissetriz
do angulo A, mostre que EB e tambem bissetriz do angulo B.
Soluc
ao. Como AB k CD ent
ao BAE = DEA e, com isso, AD = BE. Como
CD = AD + BC ent
ao EC = CB. Assim, CEB = CBE. Mas AB k CD entao
CEB = EBA.

b
b

4. (Cone Sul) Sejam A, B e C tres pontos (n


ao colineares) e E(6= B) um ponto qualquer que n
ao pertence `
a reta AC. Construa paralelogramos ABCD (nesta ordem) e
AECF (tambem nesta ordem). Demonstre que BE k DF .
Soluc
ao. ABCD e AECF s
ao paralelogramos de diagonais AC, BD e AC, F E
respectivamente. Como as diagonais de um paralelogramo se cortam em seus pontos
5

POT 2012 - Geometria - Nvel 2 - Aula 6 - Prof. Ccero Thiago

medios e AC e uma diagonal comum, o ponto medio de AC e o ponto medio de BD


e de F E. Logo BEDF e um quadril
atero cujas diagonais BD e F E cortam - se em
seus pontos medios. Portanto BEDF e um paralelogramo e BE k DF .
B
b

A
b
b

F
b

5. (Torneio das Cidades) Em um quadrado ABCD, K e um ponto do lado BC e a


bissetriz do KAD intersecta o lado CD no ponto M . Prove que o comprimento do
segmento AK e igual `
a soma dos comprimentos dos segmentos DM e BK.
Soluc
ao. Seja L o ponto no prolongamento de BC tal que BL = DM . Como AB =
AD e ABL = 90 = ADM entao ABL ADM . Assim, BAL = DAM e
ALK = AM D. Por outro lado
KAL = BAL + KAB
= M AD + KAB
= M AK + KAB
= M AB
= AM D.
a u
ltima igualdade acontece porque AB e DC s
ao paralelos. Segue que KAL =
ALK e, portanto, AK = KL = KB + BL = KB + DM.

POT 2012 - Geometria - Nvel 2 - Aula 6 - Prof. Ccero Thiago

A
b

M
b

L
b

b
b

6. (Torneio das Cidades) ABCD e um paralelogramo. Um ponto M e escolhido sobre


o lado AB tal que M AD = AM O, onde O e o ponto de intersecao das diagonais
do paralelogramo. Prove que M D = M C.
Soluc
ao. Seja N o intersec
ao de M O e CD. Temos que M AD = AM N entao
AM N D e um trapezio is
osceles. Por simetria, AM = N C entao AM CN e um paralelogramo. Com isso, M DC = AN D = M CD e, portanto, M C = M D.
M

A
b

b
b

O
b

N
D

b
b

Exerccios Propostos

POT 2012 - Geometria - Nvel 2 - Aula 6 - Prof. Ccero Thiago

1. No quadrado ABCD consideram - se as diagonais AC e BD. Seja P um ponto qualquer pertencente a um dos lados. Demonstrar que a soma das dist
ancias de P `as
duas diagonais e constante.
2. (Maio) Num paralelogramo ABCD, BD e a diagonal maior. Ao fazer coincidir B
com D mediante uma dobra se forma um pentagono regular. Calcular as medidas
dos angulos que forma a diagonal BD com cada um dos lados do paralelogramo.
3. (Maio) No ret
angulo ABCD de lados AB, BC, CD e DA, seja P um ponto do lado
AD tal que BP C = 90 . A perpendicular a BP tracada por A corta BP em M e
a perpendicular a CP tracada por D corta CP em N . Demonstre que o centro do
ret
angulo est
a no segmento M N .
4. Sejam ABC e ABD tri
angulo com o lado AB comum. O tri
angulo ABC tem
BAC = 90 e AB = 2AC. O tri
angulo ABD tem ADB = 90 e AD = BD.
O segmento CD corta o segmento AB em O. Calcule a medida de BO sabendo que
AC = 4.
5. (OBM) O trapezio ABCD tem bases AB eCD. O lado DA mede x e o lado BC
mede 2x. A soma dos
angulos DAB e ABC e 120 . Determine o angulo DAB.
6. No quadril
atero convexo ABCD, sejam E e F os pontos medios dos lados AD e BC,
respectivamente. Os segmentos CE e DF se cortam em O. Demonstre que se as retas
AO e BO dividem o lado CD em tres partes iguais entao ABCD e um paralelogramo.
7. Seja ABCDEF um hexagono tal que seus lados opostos s
ao respectivamente paralelos, ou seja, AB k DE, BC k EF e CD k F A. Se AB = DE, demonstre que
BC = EF e CD = F A.
8. Seja ABCD um paralelogramo tal que M e o ponto medio de BC. Seja T a projecao
de D sobre M A. Prove que CT = CD.

B
b

POT 2012 - Geometria - Nvel 2 - Aula 6 - Prof. Ccero Thiago

9. Prove que o segmento que liga os pontos medios dos lados opostos de um quadril
atero
convexo passa pelo ponto medio do segmento que liga os pontos medios das diagonais.
10. Seja ABCD um paralelogramo. Pelo vertice A e tracada uma reta r e sejam E, F
e G as projec
oes de B, C e D sobre r, respectivamente. Prove que se r estiver no
exterior do paralelogramo, ent
ao CF = BE + DG e, se r estiver no interior, entao
CF = |BE DG|.
11. Sobre os lados AB e AC do tri
angulo ABC s
ao construdos no exterior tri
angulos
is
osceles semelhantes ABC e CAB . Prove que AB A C e um paralelogramo.
12. Os lados AB, BC, CD e DA de um quadril
atero ABCD s
ao divididos pelos pelos
E, F , G e H da seguinte forma:
CF
CG
DH
AE
=
=
=
.
EB
FB
GD
HA
Prove que EF GH e um paralelogramo.
13. Seja P1 P2 P3 P4 P5 um pent
agono convexo. Seja Qi o ponto de intersecao dos segmentos que unem os pontos medios dos lados opostos do quadril
atero Pi+1 Pi+2 Pi+3 Pi+4
onde Pk+5 = Pk , k N e i {1, 2, 3, 4, 5}. Prove que os pentagonos P1 P2 P3 P4 P5 e
Q1 Q2 Q3 Q4 Q5 s
ao semelhantes.
Sugest
oes
2. Seja O o ponto de intersec
ao das diagonais de um paralelogramo ABCD. Seja EF
um segmento que passa por O com extremidades E e F sobre os lados AB e CD,
respectivamente. Ent
ao, EO = EF .
3. Use o fato que as diagonais de um paralelogramo cortam - se em seus pontos medios.
8. Trace CP DT , com P em DT .
9. Use base media.
11. Use semelhanca de tri
angulos.
12. Use Teorema de Tales.

POT 2012 - Geometria - Nvel 2 - Aula 6 - Prof. Ccero Thiago

13. Os pontos medios dos lados de um quadril


atero s
ao vertices de um paralelogramo.
Bibliografia
1. Problemas 18 - Olimpada Matematica Argentina
Patrcia Fauring e Flora Gutierrez
Red Olmpica
2. Problemas 19 - Olimpada Matematica Argentina
Patrcia Fauring e Flora Gutierrez
Red Olmpica
3. Problemas 20 - Olimpada Matematica Argentina
Patrcia Fauring e Flora Gutierrez
Red Olmpica
4. Olimpadas de Mayo - I a VIII
Patrcia Fauring, Flora Gutierrez, Carlos Bosch e Mara Gaspar
Red Olmpica
5. Olimpadas de Mayo - IX a XVI
Patrcia Fauring, Flora Gutierrez, Carlos Bosch e Mara Gaspar
Red Olmpica
6. International Mathematics Tournament of Towns - 1997 - 2002
AM Storozhev
AMT
7. Colec
ao Elementos da Matematica, vol. 2 - Geometria Plana
Marcelo Rufino de Oliveira e Marcio Rodrigo da Rocha Pinheiro
8. Challenging Problems in Geometry
Alfred S. Posamentier e Charles T. Salkind
9. Problems and Solutions in Euclidean Geometry
M. N. Aref e William Wernick
10. Geometra
Radmila Bulajich Manfrino e Jose Antonio G
omez Ortega
Cuadernos de Olimpiadas de Matematicas

10

POT 2012 - Geometria - Nvel 2 - Aula 6 - Prof. Ccero Thiago

11. T
opicos de Matematica Elementar, vol. 2
Geometria Euclidiana Plana
Antonio Caminha Muniz Neto
SBM
12. Episodes in Nineteenth and Twentieth Euclidean Geometry
Ross Honsberger
MAA
13. Problems in Plane and Solid Geometry, vol. 1 - Plane Geometry
Viktor Prasolov
14. Advanced Euclidean Geometry
Alfred Posamentier
15. Lessons in Geometry
I. Plane Geometry
Jacques Hadamard
AMS
16. Hadamards Plane Geometry
A Readers Companion
Mark Saul
AMS
17. Olimpadas Cearenses de Matematica, Ensino Fundamental, 1981 - 2005
Emanuel Carneiro, Francisco Ant
onio M. de Paiva e Onofre Campos
18. Problemas de las Olimpiadas Matematicas del Cono Sur (I a IV)
Fauring - Wagner - Wykowski - Gutierrez - Pedraza - Moreira
Red Olmpica
19. Explorations in Geometry
Bruce Shawyer
World Scientific
20. Treinamento Cone Sul, vol.2.
Bruno Holanda, Ccero Magalhaes, Samuel Barbosa e Yuri Lima.

11

Polos Olmpicos de Treinamento


Curso de Geometria - Nvel 2

Aula

Prof. Ccero Thiago

Angulos
na circunfer
encia
Defini
c
ao 1: O
angulo inscrito relativo a uma circunferencia e um angulo que tem o vertice
na circunferencia e os lados s
ao secantes a ela.

A
b

Assim, AP B e o
angulo inscrito e AOB e o angulo central que e igual `a medida do arco,
que n
ao contem P , determinado na circunferencia pelos pontos A e B.
Teorema 1. Um
angulo inscrito e metade do angulo central correspondente.
Demonstrac
ao. A prova ser
a dividida em tres casos.

1 caso:
O tri
angulo OBC e is
osceles e, com isso, OBC = OCB . Entao, AOC = OBC +
OCB = 2OBC (propriedade do
angulo externo).

POT 2012 - Geometria - Nvel 2 - Aula 7 - Prof. Ccero Thiago

B
b

C
b

2
b

2 caso:
Pelo 1 caso temos que AOC = 2ABC e AOD = 2ABD. Portanto, COD =
2CBD.

A
b

3 caso:
Pelo 1 caso temos que EOD = 2ECD, entao 2 + 2 = 2 ( + ) = . Portanto,
AOD = 2ACD.

POT 2012 - Geometria - Nvel 2 - Aula 7 - Prof. Ccero Thiago


O
b
b

2
2
E

b
b

Defini
c
ao 2: Dizemos que uma reta e tangente a uma circunferencia se essa reta intersecta
a circunferencia em um u
nico ponto.
Teorema 2. Toda reta perpendicular a um raio na sua extremidade da circunferencia e
tangente `a circunferencia.
Demonstrac
ao.

r
A
b

B
b

Suponha que OA r mas r n


ao e tangente `a circunferencia, e seja B 6= A o segundo
ponto de intersec
ao. Isso e um absurdo pois o tri
agulo OAB seria is
osceles, pois OA = OB
(raio da circunferencia), com os
angulos da base iguais a 90 . Portanto, r e tangente `a
circunferencia.
Teorema 3. Toda tangente a uma circunferencia e perpendicular ao raio no ponto de
tangencia.
Demonstrac
ao.

POT 2012 - Geometria - Nvel 2 - Aula 7 - Prof. Ccero Thiago

A
b

Seja A o ponto de tangencia. Qualquer ponto de r est


a a uma dist
ancia maior do que A
do centro. Com isso, OA e a menor dist
ancia de O para a reta r. Portanto, OA r.
Defini
c
ao 3: Um
angulo de segmento relativo a uma circunferencia e um angulo que tem
o vertice na circunferencia, um lado secante e outro lado tangente `a circunferencia.

b
b

O angulo da figura e um
angulo de segmento.
Teorema 4. Um
angulo de segmento e a metade do angulo central correspondente.
Demonstrac
ao.
Calculando a soma dos
angulos internos no tri
angulo AOB temos 2+2 = 180 + =

90 . Mas, + = 90 . Portanto, = .

POT 2012 - Geometria - Nvel 2 - Aula 7 - Prof. Ccero Thiago

O
b

Defini
c
ao 4: Chamaremos de
angulo excentrico interior qualquer angulo formado por duas
cordas de uma circunferencia. Na figura abaixo, temos que BED e um angulo excentrico
AOC + BOD
interior que satisfaz BED =
, pois BED = ABC + DCB =
2
AOC
BOD
AOC + BOD
EBC + ECB =
+
=
.
2
2
2
A
b

E
b

C
b

Defini
c
ao 5: Chamaremos de
angulo excentrico exterior o angulo formado por duas
secantes a uma circunferencia tracadas por um ponto no exterior. Na figura abaixo,
BOD AOC
BP D e um
angulo excentrico exterior que satisfaz BP D =
, pois
2
BOD AOC
BOD AOC
BP D = BAD ADP =

=
.
2
2
2

POT 2012 - Geometria - Nvel 2 - Aula 7 - Prof. Ccero Thiago

B
b

A
b

O
b

C
b

Exerccios Resolvidos

1. (OBM) O tri
angulo ABC est
a inscrito na circunferencia S e AB < AC. A reta que
contem A e e perpendicular a BC encontra S em P (P 6= A). O ponto X situa-se sobre o segmento AC e a reta BX intersecta S em Q (Q 6= B). Mostre que BX = CX
se, e somente se, P Q e um di
ametro de S.
Soluc
ao. Vamos dividir o problema em duas partes:
(a) BX = CX P Q e um di
ametro de S. Seja ACB = . Assim, temos que
QBC = (ja que BX = CX) e P AC = 180 90 = 90 . Observe
que os
angulos P AC = P BC. Assim, vemos que P BC = P AC = 90
P BQ = 90 + = 90 P Q e di
ametro.
Q

A
b

90

X
b

90

POT 2012 - Geometria - Nvel 2 - Aula 7 - Prof. Ccero Thiago

(b) P Q e um di
ametro de S BX = CX. Se ACB = , P AC = P BC =
90 . Mas P Q e di
ametro, ou seja, P BQ = 90 90 + QBC = 90
QBC = BXC e is
osceles BX = XC.
2. Sobre um crculo de di
ametro AB s
ao escolhidos os pontos C, D e E em um semiplano
= BE
= 20 e
determinado por AB e F no outro semiplano, tais que
AC = CD
= 60 . Seja M a intersecc
BF
ao de BD e CE. Prove que F M = F E.
Soluc
ao.

D
b

C
b

M
b

Seja O o centro da circunferencia. Vamos provar que os tri


angulos OM F e BEF
s
ao congruentes. Como BOF = 60 entao o tri
angulo BOF e equilatero e
= CD
BE = CD e DCE = EBD = 60 ,
OF = BF . Alem disso, BE
ou seja, CDM EBM CM = BM OCM = OBM M OB = 80 .
Como OBE = OBM + EBD = ABD + 60 = 20 + 60 = 80 entao o trapezio
M OBE e is
osceles e, com isso, M O = EB. Finalmente, M OF = 80 +60 = 140 =
EBF . Isto prova que os tri
angulos OM F e BEF s
ao congruentes. Portanto,
F M = F E.
3. Sejam dois crculos C1 e C2 , com C2 tangente interno a C1 no ponto P . Seja s uma
reta tangente a C2 em um ponto B, e que corta C1 em A e C. Mostre que P B e
bissetriz do
angulo AP C.
Soluc
ao.

POT 2012 - Geometria - Nvel 2 - Aula 7 - Prof. Ccero Thiago

r
b

C
b

Seja r a reta tangente `


as duas circunferencias em P . Seja O o ponto de interseccao
de r com o prolongamento AC. Temos que
CP O = P AC =

CP
2

OP = OB P BO = BP C + CP O.
Mas P BO e
angulo externo do tri
angulo P BA P BO = P AC + BP A.
Portanto, BP C = BP A.
4. Seja O o centro da circunferencia circunscrita ao tri
angulo acut
angulo ABC e seja D
a projec
ao de A sobre BC. Prove que DAB = OAC.
Soluc
ao.

POT 2012 - Geometria - Nvel 2 - Aula 7 - Prof. Ccero Thiago

A
b

b
b

Seja AE um di
ametro. Alem disso, ABC = AEC. Portanto, BAD = EAC.
5. (Italia) Um tri
angulo ABC acut
angulo est
a inscrito em um crculo de centro O . Seja
D a intersecc
ao da bissetriz de A com BC e suponha que a perpendicular a AO por
D, corta a reta AC em um ponto P interior a AC. Mostre que AB = AP .
Soluc
ao.

POT 2012 - Geometria - Nvel 2 - Aula 7 - Prof. Ccero Thiago

A
b

O
b

E
b

Usando o resultado obtido no problema 4 temos que BAF = EAP . Como AD e


bissetriz do
angulo A, ent
ao F AD = DAE. Com isso, F AD DAE, pelo
caso A.L.A., assim AF = AE. Desse modo, AEP ABF , pelo caso A.L.A.
Finalmente, AB = AP .
6. (Ira) Em um tri
angulo ABC a bissetriz do angulo BAC intersecta o lado BC no
ponto D. Seja um crculo tangente a BC no ponto D e que passa pelo ponto A. Se
M e o segundo ponto de interseccao de AC com e se BM intersecta o crculo em
P , mostre que AP e uma mediana do tri
angulo ABD.
Soluc
ao.

10

POT 2012 - Geometria - Nvel 2 - Aula 7 - Prof. Ccero Thiago

A
b

b
b

Como BC e tangente `
a circunferencia , temos que N DP = DM P = P AD = .
Alem disso, M P D = M AD = BAD = . Sendo assim, N P D N AD,
ND
NP
=
N D 2 = N P N A. Temos tambem que N BP N AB,
entao
ND
NA
PN
NB
entao
=
N B 2 = N P N A. Portanto, N B 2 = N D 2 N B = N D.
NB
AN
Exerccios propostos

1. Na figura, a reta t e tangente ao crculo e paralela ao segmento DE. Se AD = 6,


AE = 5 e CE = 7, o valor da medida do segmento BD e:

11

POT 2012 - Geometria - Nvel 2 - Aula 7 - Prof. Ccero Thiago

t
A
b

D
b

(a) 3, 5 (b) 4 (c) 4, 5 (d) 5 (e) 5, 5


2. S
ao dadas duas circunferencias secantes, com pontos de interseccao C e D. Traca - se
por C uma secante `
a duas circunferencias, que intersecta uma delas em E e a outra
em F . Mostre que o
angulo EDF e constante.
3. As extremidades de uma corda ST , com comprimento constante, s
ao movidos ao
longo de um semicrculo com di
ametro AB. Seja M o ponto medio de ST e P o pe
da perpendicular de S sobre AB. Prove que a medida do angulo SP M independe
da posic
ao de ST .
dado um tri
4. E
angulo ABC. Sejam O o centro da circunferencia circunscrita ao
tri
angulo, I o centro da circunferencia inscrita no tri
angulo, D 6= A a interseccao da
reta AI com a circunferencia circunscrita. Prove que CD = BD = ID.
5. Se os lados AB e AC de um tri
angulo s
ao di
ametros de duas circunferencias, prove
que o outro ponto comum `
as circunferencias est
a em BC.
6. Sejam C1 e C2 duas circunferencia tangentes exteriores em T . Sejam A e B pontos
de C1 tais que a reta AB e tangente a C2 em P . Prove que T P e bissetriz externa

12

POT 2012 - Geometria - Nvel 2 - Aula 7 - Prof. Ccero Thiago

do tri
angulo AT B.
7. Na figura abaixo seja T o ponto de tangencia das circunferencias. Prove que M T P =
QT N .

M
b

P
b

Q
b

Sugest
oes/Solu
co
es
1. BD = 4. Use
angulo de segmento para concluir que ADE ABC.
2. Use que em uma circunferencia, a medida do angulo inscrito e metade da medida do
angulo central que subtende o mesmo arco.
3.

13

POT 2012 - Geometria - Nvel 2 - Aula 7 - Prof. Ccero Thiago

b
b

S
b

C
b

S1

facil
Seja C a circunferencia de di
ametro AB. Seja S1 o simetrico de S com relacao a AB. E
ver que P e o ponto medio de SS1 e seja M o ponto medio de ST . Temos que P M k S1 T .
1
s
Entao SP M = SS1 T = ST
. Por outro lado, ST
o depende do comprimento de ST .
2
Portanto, segue o resultado.
4. Use angulo externo e
angulos inscritos.
5. Use que todo
angulo inscrito em uma semicircunferencia mede 90 .
6. Use angulos de segmento.
7. Use angulos inscritos.
Bibliografia
1. Fundamentos de Matematica Elementar, vol.9.
Osvaldo Dolce e Jose Nicolau Pompeo.
2. Geometra - Una vision de la planimetra.
Lumbreras.
14

POT 2012 - Geometria - Nvel 2 - Aula 7 - Prof. Ccero Thiago

3. Challenging Problems in Geometry


Alfred S. Posamentier e Charles T. Salkind
4. Problems and Solutions in Euclidean Geometry
M. N. Aref e William Wernick
5. Geometra
Radmila Bulajich Manfrino e Jose Antonio G
omez Ortega
Cuadernos de Olimpiadas de Matematicas
6. T
opicos de Matematica Elementar, vol. 2
Geometria Euclidiana Plana
Antonio Caminha Muniz Neto
SBM

15

Polos Olmpicos de Treinamento


Aula

Curso de Geometria - Nvel 2

Prof. Ccero Thiago

Quadril
ateros inscritveis

Teorema 1. Um quadril
atero e inscritvel se, e somente se, a soma dos angulos opostos e
180 .

BCD
BAD
+
=
Demonstrac
ao. Seja ABCD um quadril
atero inscritvel. Temos que
2
2

360 , ou seja, 2A + 2C = 360 A + C = 180 . Como a soma dos angulos internos


de um quadril
atero convexo e 360 , entao B + D = 180 .
A
b

B
b

O
b

Seja ABCD um quadril


atero tal que A + C = B + D = 180 . Vamos admitir,
de maneira falsa, que ABCD n
ao e inscritvel. Seja E a interseccao de BC com a circunferencia circunscrita ao tri
angulo ABD. Sendo assim, A + E = 180 C = E, o
que e um absurdo pela propriedade do angulo externo. Portanto, ABCD e inscritvel.

POT 2012 - Geometria - Nvel 2 - Aula 8 - Prof. Ccero Thiago

A
b

B
b

O
b

C
b

b
b

Teorema 2. Um quadril
atero e inscritvel se, e somente se, o angulo entre um lado e uma
diagonal e igual ao
angulo entre o lado oposto e a outra diagonal.
Demonstrac
ao.
A
b

B
b

O
b

facil ver que DAC = DBC = DC .


Seja ABCD um quadril
atero inscritvel. E
2

POT 2012 - Geometria - Nvel 2 - Aula 8 - Prof. Ccero Thiago

A
b

B
b

O
b

C
b

b
b

Seja ABCD um quadril


atero tal que ADB = ACB. Vamos admitir, de maneira
falsa, que ABCD n
ao e inscritvel. Seja E a interseccao de BC com a circunferencia circunscrita ao tri
angulo ABD. Sendo assim, ADB = ACB = AEB, o que e um absurdo
pela propriedade do
angulo externo. Portanto, ABCD e inscritvel.
Exerccios Resolvidos

1. Em um tri
angulo ABC, BAC = 100 e AB = AC. Seja BD a bissetriz de ABC,
com D sobre o lado AC. Prove que AD + BD = BC.
Soluc
ao.

E facil ver que ABD = DBC = 20 . Seja E um ponto sobre BC tal que
BD = BE. Basta provar que EC = AD. Veja que BDE = BED = 80 .
Como BED = 80 e BCD = 40 , entao EDC = 40 , ou seja, ED = EC. Por
outro lado, ABED e um quadril
atero inscritvel pois BAD + BED = 180 , assim
EAD = EBD = 20 e AED = ABD = 20 . Portanto, AD = ED = EC e,
dessa forma, BC = AD + BD.

POT 2012 - Geometria - Nvel 2 - Aula 8 - Prof. Ccero Thiago

A
b

D
b

2. (Inglaterra) No tri
angulo acut
angulo ABC, CF e altura, com F em AB e BM e
mediana, com M em CA. Se BM = CF e M BC = F CA, prove que o tri
angulo
ABC e equilatero.
Soluc
ao.
A
b

M
b

Temos que F M = AM = M C e, com isso, M F C = F CM , ou seja, o quadril


atero

F BCM e inscritvel. Dessa forma, F CM = F BM e BM C = BF C = 90 . E


facil ver que BM C BM A, pelo caso A.L.A, entao AB = BC. Veja tambem
que BM C BF C, pelo caso cateto - hipotenusa, entao BCM = CBF e,
portanto, AC = AB. Finalmente, AB = AC = BC.

POT 2012 - Geometria - Nvel 2 - Aula 8 - Prof. Ccero Thiago

3. Seja M um ponto no interior de um quadril


atero convexo ABCD tal que ABM D e
um paralelogramo. Prove que se CBM = CDM , entao ACD = BCM .
Soluc
ao.

A
b

N
b

Seja N um ponto tal que BN k M C e N C k BM . Entao N A k CD, N CB =


CBM = CDM = N AB, ou seja, os pontos A, B, N e C s
ao concclicos. Entao,
ACD = N BC = BCM .
4. (Cone Sul) Seja ABCD um quadril
atero convexo tal que suas diagonais AC e BD
s
ao perpendiculares. Seja P a interseccao de AC e BD e seja M o ponto medio de
AB. Mostre que o quadril
atero ABCD e inscritvel se, e somente se, as retas P M e
CD s
ao perpendiculares.
Soluc
ao. Primeiramente vejamos quando P M e CD s
ao perpendiculares. Seja K a
intersecc
ao de P M e CD. Como no tri
angulo ABP , ret
angulo em P , M e o ponto
medio da hipotenusa AB P M = M A = M B. Assim, seja
ABD = M P B = AM P = 2 M P A = 90
CP K = AP M = 90 .

POT 2012 - Geometria - Nvel 2 - Aula 8 - Prof. Ccero Thiago

Como P KC = 90 P CD = . Logo, ABD = ACD = o quadril


atero
ABCD e inscritvel.
B
b

E
b

A
b

2
90
90

P
b

90

Vejamos agora o caso em que o quadril


atero ABCD e inscritvel. Do mesmo modo
como M e o ponto medio da hipotenusa AB do tri
angulo ret
angulo AP B entao
P M = M A = M B. Logo, se ABD = BAP = M P A = 90
CP K = 90 e como ABCD e inscritvel ACD = ABD = P KC =
180 (90 + ) = 90 M P CD. Portanto, ABCD e inscritvel se, e somente
se, P M CD.

POT 2012 - Geometria - Nvel 2 - Aula 8 - Prof. Ccero Thiago

B
b

E
b

A
b

2
90
90
P
b

90

b
b

5. Prove que as bissetrizes internas dos quatro angulos de um quadril


atero convexo determinam um quadril
atero inscritvel.
facil ver que = 180 e que = 180 . Dessa forma,
Soluc
ao. E
+ = 360 ( + + + )
= 360 180 = 180 .

POT 2012 - Geometria - Nvel 2 - Aula 8 - Prof. Ccero Thiago

B
b

A
b
b

H
b

Exerccios Propostos
1. (BAMO) Seja k um crculo no plano xy com centro sobre o eixo y e passando pelos
pontos A(0, a) e B(0, b) com 0 < a < b. Seja P um ponto qualquer do crculo, diferente de A e B. Seja Q a interseccao da reta que passa por P e A com o eixo x, e
seja O(0, 0). Prove que BQP = BOP .
2. (OBM) As diagonais de um quadril
atero inscritvel ABCD se intersectam em O. Os
crculos circunscritos aos tri
angulos AOB e COD intersectam as retas BC e AD,
pela segunda vez, nos pontos M , N , O e Q. Prove que o quadril
atero M N P Q est
a
inscrito em um crculo de centro O.
3. Um quadril
atero convexo est
a inscrito em um crculo de centro O. As diagonais AC
e BD intersectam - se em P . Os crculos circunscritos aos tri
angulos ABP e CDP
intersectam - se novamente em Q. Se O, P e Q s
ao tres pontos distintos, prove que
OQ e perpendicular a P Q.
4. (Ibero) Num tri
angulo escaleno ABC traca-se a bissetriz interna BD, com D sobre
AC. Sejam E e F , respectivamente, os pes das perpendiculares tracadas desde A e
C ate `a reta BD, e seja M o ponto sobre o lado BC tal que DM e perpendicular a
BC. Prove que EM D = DM F.

POT 2012 - Geometria - Nvel 2 - Aula 8 - Prof. Ccero Thiago

5. Seja M o ponto de intersec


ao das diagonais de um quadril
atero inscritvel ABCD,
em que AM B e agudo. O tri
angulo is
osceles BCK e construdo exteriormente ao
quadril
atero, com base a base sendo BC, tal que KBC + AM B = 90 . Prove que
KM e perpendicular a AD.
6. (Romenia) Seja ABC um tri
angulo acut
angulo, e seja T um ponto no interior tal que
AT B = BT C = CT A. Sejam M , N e P as projecoes de T sobre BC, CA, e
AB, respectivamente. O crculo circunscrito ao tri
angulo M N P intersecta os lados

BC, CA e AB, pela segunda vez, em M , N e P , respectivamente. Prove que o

tri
angulo M N P e equilatero.
7. (Cone Sul) Seja ABCD um quadrado (os vertices est
ao nomeados no sentido hor
ario)
e P um ponto qualquer pertencente ao interior do segmento BC. Constroi - se o quadrado AP RS (os vertices novamente nomeados no sentido hor
ario). Demonstrar que
a reta CR e tangente `
a circunferencia circuscrita ao tri
angulo ABC.
8. (IMO) Duas circunferencias 1 e 2 intersectam - se em M e N . Seja l a tangente
comum a 1 e 2 que est
a mais pr
oxima de M do que de N . A reta l e tangente a
1 em A e a 2 em B. A reta paralela a l que passa por M intersecta novamente a
circunferencia 1 em C e novamente a circunferencia 2 em D. As retas CA e DB
intersectam - se em E; as retas AN e CD intersectam - se em P ; as retas BN e CD
intersectam - se em Q. Mostre que EP = EQ.
9. Seja Q o ponto medio do lado AB de um quadril
atero inscritvel ABCD e S a intersecao de suas diagonais. Sejam P e R as projecoes ortogonais de S sobre AD e
BC, respectivamente. Prove que P Q = QR.
10. (Italia) Um tri
angulo ABC acut
angulo est
a inscrito em um crculo de centro O. Seja
D a intersec
ao da bissetriz de A com BC e suponha que a perpendicular a AO por
D, corta a reta AC em um ponto P , interior a AC. Mostre que AB = AP .
Bibliografia
1. A Decade of Berkeley Math Circle
Zvesdelina Stankova
2. Problems in plane and solid geometry
Viktor Prasolov
3. Episodes in nineteenth and twentieth century euclidean geometry
Ross Honsberger

Polos Olmpicos de Treinamento


Aula

Curso de Geometria - Nvel 2

Prof. Ccero Thiago

Teorema de Ptolomeu
Teorema 1. (Ptolomeu) O produto dos comprimentos das diagonais de um quadril
atero
inscritvel e igual a soma dos produtos dos comprimentos dos pares de lados opostos.
Demonstrac
ao.
b

b
b

Seja P o ponto sobre o prolongamento do lado CD tal que BAC = DAP . Como o
quadril
atero ABCD e inscritvel ent
ao ABC = ADP , assim ABC DAP . Com
isso,
AB
BC
AC
AD BC
=
=
DP =
.
AD
DP
AP
AB
AB
AC
Como BAD = CAP e
=
. Portanto, ABD ACP . Assim,
AD
AP
AB
AC BD
BD
=
CP =
.
CP
AC
AB
Mas CP = CD + DP , dessa forma
AC BD
AD BC
= CD +

AB
AB

POT 2012 - Geometria - Nvel 2 - Aula 9 - Prof. Ccero Thiago

AC BD = AB CD + AD BC.
Exerccios Resolvidos
que n
1. Seja ABC um tri
angulo equilatero e seja P um ponto sobre o arco BC,
ao contem
A, da circunferencia circunscrita ao tri
angulo ABC. Prove que P A = P B + P C.

Soluc
ao.
A
b

Como o tri
angulo ABC e equilatero entao AB = BC = CA = a. Aplicando o
teorema de Ptolomeu ao quadril
atero ABP C temos que
AB P C + AC P B = BC P A
a PC + a PB = a PA
P C + P B = P A.
2. (IME) Dado o quadril
atero ABCD, inscrito num crculo de raio r, conforme a figura
abaixo, prove que:
AB AD + BC CD
AC
=
.
BD
AB BC + CD AD

POT 2012 - Geometria - Nvel 2 - Aula 9 - Prof. Ccero Thiago

C
b

B
b

b
b

Soluc
ao.

b
b

b
b

Sejam M e N pontos sobre a circunferencia circunscrita ao quadril


atero ABCD tais

que AM = CD e DN = AB. Dessa forma AM = CD, DN = AB, BM = CN e


M C = BN = AD. Aplicando o teorema de Ptolomeu nos quadril
ateros M ABC e
N BCD temos
BM AC = AB M C + BC AM (I)
CN BD = DN BC + CD BN (II)
Fazendo (I) (II), temos
AB AD + BC CD
AC
=
.
BD
AB BC + CD AD
3

POT 2012 - Geometria - Nvel 2 - Aula 9 - Prof. Ccero Thiago

Este resultado e conhecido como Teorema de Hiparco.


3. (Seletiva do Brasil para a Cone Sul) Prove que as dist
ancias entre um ponto sobre
uma circunferencia e os quatro vertices de um quadrado nesta inscrita n
ao podem ser
todas n
umeros racionais.
Soluc
ao.

Como ABCD e um quadrado entao AB = BC = CD = DA = a. Pelo toerema de


Pit
agoras no tri
angulo ABC temos que AC 2 = AB 2 + BC 2 AC 2 = a2 + a2 =
2
2 a AC = 2 a. Aplicando o teorema de Ptolomeu no quadril
atero ABCP ,
temos

AC BP = AP BC + CP AB 2 a BP = AP a + CP a

2=

AP + CP
.
BP

Se todas as medidas fossem n


umeros racionais estaramos afirmando, de maneira falsa,

BP
= 2.
que 2 Q. Se P coincidir com um dos vertices, ou seja, P D, entao
CP
Assim, as medidas n
ao podem ser todas racionais.
4. (Ira) Seja ABC um tri
angulo com BC > CA > AB. Seja D um ponto sobre o
lado BC e seja E o ponto no prolongamento de BA, com A entre E e B, tal que
BD = BE = CA. Seja P o ponto sobre AC tal que E, B, D e P s
ao concclicos e
seja Q o segundo ponto de intersecao de BP com o crculo circunscrito ao tri
angulo
ABC. Prove que AQ + CQ = BP .

POT 2012 - Geometria - Nvel 2 - Aula 9 - Prof. Ccero Thiago

Soluc
ao.
b

A
b

Q
b

P
b

b
b

Veja que AQC EP D, pois CAQ = CBQ = DEP e AQC = 180


ABD = EP D. Por outro lado, pelo teorema de Ptolomeu, temos
BP DE = BE DP + BD EP.
Entao,
BP = BE

EP
CQ
AQ
DP
+ BD
= CA
+ CA
= AQ + CQ.
DE
DE
CA
CA

5. Seja A1 A2 A3 . . . An um polgono regular de n lados tal que


1
1
1
=
+
.
A1 A2
A1 A3 A1 A4
Determine n, ou seja, o n
umero de lados do polgono regular.
Soluc
ao. Temos que

1
1
1
=
+

A1 A2
A1 A3 A1 A4

A1 A2 A1 A3 + A1 A2 A1 A4 = A1 A3 A1 A4 . (I)
5

POT 2012 - Geometria - Nvel 2 - Aula 9 - Prof. Ccero Thiago

A5
A4
b
b

A3

An
b

A2

A1

Como o quadril
atero A1 A3 A4 A5 e inscritvel podemos aplicar o teorema de Ptolomeu,
assim
A4 A5 A1 A3 + A3 A4 A1 A5 = A3 A5 A1 A4 . (II)
Alem disso, como o polgono e regular, temos
A1 A2 = A4 A5 , A1 A2 = A3 A4 , A1 A3 = A3 A5 .
Comparando (I) e (II), encontramos
A1 A4 = A1 A5 .
Como as diagonais A1 A4 e A1 A5 s
ao iguais, segue que existe o mesmo n
umero de
vertices entre A1 e A4 e entre A1 e A5 . Dessa forma conclumos que n = 7.
Exerccios Propostos
1. (AHSME) Seja ABCD um quadril
atero e seja O o ponto de intersecao das diagonais
AC e BD. Se BO = 4, OD = 6, AO = 8, OC = 3 e AB = 6, determine a medida de
AD.
2. Seja ABCD um quadril
atero convexo tal que
AC BD = AB CD + AD BC.
Prove que ABCD e inscritvel.

POT 2012 - Geometria - Nvel 2 - Aula 9 - Prof. Ccero Thiago

3. Seja ABCD um quadrado. Determine o lugar geometrico dos pontos P , no mesmo


plano do quadrado ABCD, tais que
1
m
ax {P A, P C} = (P B + P D).
2
4. Uma circunferencia passa pelo vertice A de um paralelogramo ABCD intersectando
os lados AB e AD nos pontos P e R, respectivamente. Alem disso intersecta a diagonal AC no ponto Q. Prove que AQ AC = AP AB + AR AD.
5. Um ponto P e escolhido o interior do paralelogramo ABCD de tal forma que AP B+
CP D = 180 . Prove que AB AD = BP DP + AP CP .
6. Seja ABC um tri
angulo is
osceles, com AB = AC, inscrito em uma circunferencia .
que n
Seja P um ponto sobre o arco BC,
ao contem A, da circunferencia . Prove
PA
AC
que
=
.
PB + PC
BC
7. Seja ABCD um quadrado inscrito em uma circunferencia . Seja P um ponto sobre
que n
o arco BC,
ao contem A e D, da circunferencia . Prove que
PD
PA + PC
=
.
PB + PD
PA
8. A bissetriz do
angulo A do tri
angulo ABC intersecta o crculo circunscrito no ponto
D. Prove que AB + AC 2AD.
9. (IMO) Seja ABCDEF um hexagono convexo tal que AB = BC = CD, DE = EF =
F A e BCD = EF A = 60 . Sejam G e H pontos no interior do hexagono tais que
AGB = DHE = 120 . Prove que AG + GB + GH + DH + HE CF .

10.
(Mandelbrot) Um quadril
atero inscritvel e tal que seus lados medem 1, 7, 3 e
21, nesta ordem. Determine a dist
ancia entre os pontos medios das diagonais.
Bibliografia
1. Mandelbrot Morsels
Sam Vandervelde
2. Advanced Euclidean Geometry
Alfred Posamentier

POT 2012 - Geometria - Nvel 2 - Aula 9 - Prof. Ccero Thiago

3. Problem primer for the olympiad


C R Pranesachar, B J Venkatachala e C S Yogananda
4. 360 Problems for Mathematical Contests
Titu Andreescu e Dorin Andrica
5. Olimpadas de Matematica 97
Antonio Caminha, Onofre Campos e Paulo Rodrigues
6. Geometra - Una Vision de la planimetra
Lumbreras
7. the Art of Problem Solving, vol. 2: and Beyond
Richard Rusczyk e Sandor Lehoczky
8. Problems in plane and solid geometry, vol. 1 - Plane Geometry
Viktor Prasolov

Polos Olmpicos de Treinamento


Aula

Curso de Geometria - Nvel 2

10

Prof. Ccero Thiago

Relac
oes m
etricas no tri
angulo.
1. Seno, cosseno e tangente
C
b

seno de =
cosseno de =

cateto oposto
AC
AC
=
sin =
.
hipotenusa
BC
BC
AB
AB
cateto adjacente
=
cos =
.
hipotenusa
BC
BC

AC
AC
seno de
AC
tangente de =
= BC =
tan =
.
AB
cosseno de
AB
AB
BC

Como + = 90 e facil provar que sin = cos e cos = sin .


Teorema 1. (Pit
agoras) Em um tri
angulo ret
angulo a soma dos quadrados dos catetos
e igual ao quadrado da hipotenusa.
Demonstrac
ao.

POT 2012 - Geometria - Nvel 2 - Aula 10 - Prof. Ccero Thiago

A
b

b
b

am

1. ABD ABC
m
c
=
c2 = a m
a
c
2. ACD ABC
am
b
=
b2 = a (a m) = a2 a m
a
b
Entao,
c2 + b2 = a m + a2 a m
c2 + b2 = a2 .

2. Angulos
not
aveis
2.1. 45

45

a 2

45

POT 2012 - Geometria - Nvel 2 - Aula 10 - Prof. Ccero Thiago

2
a
sin 45 = =
2
a 2

a
2

cos 45 = =
2
a 2

2
tan 45 = 2 = 1.
2
2

2.2 30 e 60

30

a 3
2

60

a
2

a
1
sin 30 = cos 60 = 2 =
a
2

a 3

2
cos 30 = sin 60 =
=
a
2
1

2
tan 30 = =
3
3
2

2
tan 60 =
= 3.
1
2
3

POT 2012 - Geometria - Nvel 2 - Aula 10 - Prof. Ccero Thiago

Usaremos, sem demonstrac


ao, no restante deste material que sin x = sin(180 x) e

cos x = cos(180 x).


Teorema 2. (Lei dos Senos) Seja ABC um tri
angulo tal que BC = a, CA = b e AB = c.
Seja R o raio da circunferencia circunscrita. Entao
b
c
a
=
=
= 2R.
sin A
sin B
sin C
Demonstrac
ao.
A
b

b
O
b

B
b

facil ver que ABC = ADC. Assim, no tri


Seja AD um di
ametro. E
angulo ADC,
b
b
sin B =

= 2R. Analogamente,
2R
sin B
c
a
=
= 2R.
sin A
sin C
Finalmente,
b
c
a
=
=
= 2R.
sin A
sin B
sin C
Teorema 3. (Lei dos Cossenos) Seja ABC um tri
angulo tal que BC = a, CA = b e
AB = c. Ent
ao,
a2 = b2 + c2 2bc cos A,
b2 = a2 + c2 2ac cos B,
c2 = a2 + b2 2ab cos C.
4

POT 2012 - Geometria - Nvel 2 - Aula 10 - Prof. Ccero Thiago

Demonstrac
ao.

A
b

B
m

am

Vamos fazer o caso em que o tri


angulo e acut
angulo. O caso em que o tri
angulo e obtus
angulo fica como exerccio. Aplicando o teorema de Pit
agoras nos tri
angulos ABD e
ACD, temos:
c2 = m2 + H 2 e
b2 = (a m)2 + H 2

b2 = a2 2am + m2 + H 2 .
m
Assim, b2 = a2 + c2 2am. Por outro lado, cos B =
m = c cos B. Finalmente,
c
2
2
2
b = a + c 2ac cos B. Analogamente,
a2 = b2 + c2 2bc cos A e
c2 = a2 + b2 2ab cos C.
Teorema 4. (Stewart) Seja ABC um tri
angulo tal que BC = a, CA = b e AB = c. Seja
D um ponto sobre o lado BC tal que BD = x, CD = y e AD = z. Entao,
c2 y + b2 x z 2 a = axy.
Demonstrac
ao.

POT 2012 - Geometria - Nvel 2 - Aula 10 - Prof. Ccero Thiago

A
b

c
z

180
b

Aplicando a lei dos Cossenos nos tri


angulos ABD e ACD, temos
c2 = x2 + z 2 2xz cos(180 )
c2
z2
=x+
2z cos(180 ). (1)
x
x
E
b2 = y 2 + z 2 2yz cos

b2
z2
=y+
2z cos . (2)
y
y

Adicionando (1) e (2), encontramos


b2 c2
z2 z2
+
=x+y+
+

y
x
y
x
z2 z2
b2 c2
+
=a+
+

y
x
y
x
c2 y + b2 x z 2 a = axy.
Exerccios resolvidos

1. Num tri
angulo ABC s
ao dados A = 60 , B = 45 e BC = 4 cm. Determine a
medida de AC.
Soluc
ao.

POT 2012 - Geometria - Nvel 2 - Aula 10 - Prof. Ccero Thiago

C
b

60

45

Aplicando a lei dos senos temos


BC
AC
=

sin A
sin B
AC
4
=

sin 60
sin 45
4
AC
=
3
2
2
2

4 6
.
AC =
3
2. (OCM) Um observador estando a 25 m de um predio o visualiza sob um certo angulo.
Afastando - se, na direc
ao perpendicular ao predio mais 50 m o angulo de visualizacao
e a metade do anterior. Qual a altura do predio?
C
b

50 m

Soluc
ao.

25 m B

POT 2012 - Geometria - Nvel 2 - Aula 10 - Prof. Ccero Thiago


facil ver que ADC e is
E
osceles, ou seja, AD = CD = 50 m. Aplicando o teorema
de Pit
agoras no tri
angulo BDC temos

CD 2 = BD 2 + BC 2 502 = 252 + BC 2 BC = 25 3.
3. (China Western) Em um trapezio ABCD, AD//BC . Sejam E um ponto variando
sobre o lado AB, O1 e O2 os circuncentros dos tri
angulos AED e BEC, respectivamente. Prove que o comprimento de O1 O2 e fixo.
Soluc
ao.
A
b
b

E
b

O1

O2

facil ver que AEO1 = 90 ADE e BEO2 = 90 BCE. Entao,


E
O1 EO2 = ADE + ECB.
Como AD k BC, construa uma paralela a AD, por E. Dessa forma DEC =
ADE + BCE, ou seja, O1 EO2 = DEC. Usando lei dos senos, temos
DE
2O1 E sin A
O1 E
=
=
.
EC
2O2 E sin B
O2 E
Assim, DEC O1 EO2 . Portanto,
O1 E
O1 E
1
O1 O2
=
=
=
.
DC
DE
2O1 E sin A
2 sin A
Portanto, O1 O2 =

DC
, que e um valor fixo.
2 sin A

4. Seja ABCD um quadril


atero inscrito em uma circunferencia de di
ametro AD. Se
AB = BC = 1 e AD = 3, ache o comprimento da corda CD.
Soluc
ao.
8

POT 2012 - Geometria - Nvel 2 - Aula 10 - Prof. Ccero Thiago

C
b

D
b

Temos que AD = 3, AB = BC = 1. Aplicando o teorema de Pit


agoras no tri
angulo
ABD, temos

AD 2 = AB 2 + BD 2 32 = 11 + BD 2 BD = 2 2.

BD
2 2
Alem disso, cos =
=
. Aplicando a lei dos cossenos no tri
angulo BCD,
AD
3
temos
BC 2 = BD 2 + CD 2 2 BD CD cos

2 2
2
2

1 = 8 + CD 2 2 2 CD
3
7
CD = 3 ou .
3
Como o di
ametro mede 3, ent
ao CD =

7
.
3

5. (Teste de selec
ao do Brasil para a Cone Sul) Em um tri
angulo acut
angulo ABC,
A = 30 , H e seu ortocentro e M e o ponto medio de BC. Sobre a reta HM
tomemos um ponto T 6= H tal que HM = M T . Mostre que AT = 2BC.
Soluc
ao.

POT 2012 - Geometria - Nvel 2 - Aula 10 - Prof. Ccero Thiago

C
b

H
b

M
b

T
b

HBT C e um paralelogramo pois M e o ponto medio de BC e HM = M T . Alem


disso, BC AC e BH k AC, assim CT AC, ou seja, T CA = 90 . Com
isso, T pertence `
a circunferencia circunferencia circunscrita a ABC e AT e di
ametro.
Portanto,
BC
BC
=
= 2BC.
AT = 2R =
sin A
sin 30
Exerccios propostos
1. (OCM) Se as diagonais de um quadril
atero (convexo) s
ao perpendiculares, prove que
as somas dos quadrados dos lados opostos s
ao iguais.
2. (OCM) Dobra - se um pedaco de arame de 32 cm de comprimento formando um
tri
angulo is
osceles de 12 cm de base. Calcule a medida do comprimento da bissetriz
do angulo oposto `
a base.
3. (OBM) P e um ponto interior a um quadrado ABCD. As dist
ancias de P aos vertices
A e D e ao lado BC s
ao iguais a 10 cm. O lado do quadrado mede:
(a) 10 cm (b) 12 cm (c) 14 (d)16 cm (e) 18 cm
4. Um ponto P , interno de um
angulo de 60 , dista 6 m e 9 m dos lados desse angulo.
Qual a dist
ancia entre P e a bissetriz do angulo?
5. Seja ABC um tri
angulo tal que ABC = 45o . Seja D o ponto sobre o segmento BC
tal que 2BD = CD e DAB = 15o . Determine o angulo ACB.
6. (AIME) Seja ABC um tri
angulo tal que AB = 13, BC = 15 e CA = 14. Seja D o
ponto do segmento BC tal que CD = 6. Seja E o ponto de BC tal que CE > CD e
BAE = CAD. Determine BE.

10

POT 2012 - Geometria - Nvel 2 - Aula 10 - Prof. Ccero Thiago

7. No tri
angulo ABC, BAC = 20 e AB = AC. Os pontos M e N est
ao sobre os lados
AB e AC, respectivamente, e s
ao tais que BCM = 60 e CBN = 50 . Calcule a
medida do
angulo CM N .
8. Em um tri
angulo ABC, BAC = 100 e AB = AC. Seja BD a bissetriz de ABC,
com D sobre o lado AC. Prove que AD + BD = BC.
9. (Teste de selec
ao do Brasil para a IMO) Seja uma circunferencia de centro O tangente aos lados AB e AC do tri
angulo ABC nos pontos E e F . A reta perpendicular
ao lado BC por O intersecta EF no ponto D. Mostre que A, D e M (ponto medio
de BC) s
ao colineares.

Bibliografia
1. 103 Trigonometry Problems - From the training of the USA IMO team
Titu Andreescu
2. Precalculus
Richard Rusczyk
3. Olimpadas de Matematica 97
Antonio Caminha, Onofre Campos e Paulo Rodrigues
4. Olimpadas Cearenses de Matematica, Ensino Fundamental, 1981 - 1985
Emanuel Carneiro, Francisco Ant
onio M. de Paiva e Onofre Campos.
5. Olimpadas Cearenses de Matematica, Ensino Medio, 1981 - 1985
Emanuel Carneiro, Francisco Ant
onio M. de Paiva e Onofre Campos.

11

Polos Olmpicos de Treinamento


Aula

Curso de Geometria - Nvel 2

11

Prof. Ccero Thiago

Pot
encia de ponto e eixo radical
1. Defini
c
ao
Seja uma circunferencia de centro O e raio R. Seja P um ponto que est
a a uma dist
ancia
d de O, vamos definir a potencia do ponto P em relacao `a circunferencia por
PotP = d2 R2 .

facil ver que se P e um ponto no exterior de entao a potencia ser


E
a positiva, se P e um
ponto sobre a circunferencia ent
ao sua potencia ser
a zero e se P e um ponto no interior da
circunferencia ent
ao sua potencia ser
a negativa.
Teorema 1. Seja P um ponto e uma circunferencia. Se uma reta que passa por P
intersecta a circunferencia nos pontos A e B, entao o produto P A P B e constante.
Demonstrac
ao.

1 caso: P e um ponto no exterior.

m
m

M
b

POT 2012 - Geometria - Nvel 2 - Aula 11 - Prof. Ccero Thiago

Seja OM a mediatriz de AB. Ent


ao
P A P B = (P M m) (P M + m) = P M 2 m2 = P M 2 + OM 2 (OM 2 + m2 )
= P O 2 R2 = PotP .
Vamos analisar tambem o caso em que pelo ponto P e tracada uma tangente a .

T
b

O
b

Dessa forma pelo teorema de Pit


agoras temos que
P O 2 = P T 2 + R2 P T 2 = P O 2 R2 = PotP .
2 caso: P e um ponto no interior.

POT 2012 - Geometria - Nvel 2 - Aula 11 - Prof. Ccero Thiago

m
b

A
b

P
b

Seja OM a mediatriz de AB. Ent


ao
P A P B = (m P M ) (m + P M ) = m2 P M 2 = m2 + OM 2 (OM 2 + P M 2 )
= R2 P O 2 = PotP .
2. Eixo radical
Chamaremos de Eixo radical o lugar geometrico dos pontos que possuem a mesma
potencia com relac
ao a duas circunferencias dadas.
Teorema 2. O conjunto dos pontos que possuem a mesma potencia com relacao a duas
circunferencias dadas e uma reta perpendicular `a reta que contem os centros.
Demonstrac
ao.

POT 2012 - Geometria - Nvel 2 - Aula 11 - Prof. Ccero Thiago

P
b

b
b

O1

O2

Sejam 1 e 2 circunferencias com centros O1 e O2 e raios R1 e R2 , respectivamente. Alem


disso, seja P um ponto que possui a mesma potencia com relacao as duas circunferencias.
Assim,
PotP1 = PotP2
P O12 R12 = P O22 R22
P O12 P O22 = R12 R22 .

Seja M o ponto medio de O1 O2 , Q a projecao de P sobre O1 O2 e P M Q = . Aplicando


a lei dos cossenos nos tri
angulos P O1 M e P O2 M temos
P O12 = O1 M 2 + P M 2 2 O1 M P M cos(180 ) =
P O12 = O1 M 2 + P M 2 + 2 O1 M P M cos

P O22 = O2 M 2 + P M 2 2 O2 M P M cos .
Entao,
P O12 P O22 = 2 O1 O2 P M cos .

Por outro lado, cos =

MQ
M Q = P M cos , com isso
PM
R2 R22
MQ = 1
= Fixo.
2 O1 O2

Portanto, o lugar geometrico dos pontos P e a reta perpendicular a O1 O2 que passa por Q.

POT 2012 - Geometria - Nvel 2 - Aula 11 - Prof. Ccero Thiago

Por outro lado, seja P1 um ponto de P Q. Vamos provar que P1 possui a mesma potencia
com relacao `
as duas circunferencias. Assim, pelo toerema de Pit
agoras
P1 O12 = O1 Q2 + P1 Q2 ,
P1 O22 = O2 Q2 + P1 Q2 .
Entao,
P1 O12 P1 O22 = O1 Q2 O2 Q2 .
P
b

P1
b

O1

O2

Alem disso,
P O12 = O1 Q2 + P Q2 ,
P O22 = O2 Q2 + P Q2 .
Entao,
P O12 P O22 = R12 R22 = O1 Q2 O2 Q2 = P1 O12 P1 O22
P1 O12 R12 = P1 O22 R22
PotP11 = PotP21 .

Problema 1. Dois crculos 1 e 2 intersectam - se em P e Q. Uma reta passando por P


intersecta 1 e 2 novamente em A e B, respectivamente, se X e o ponto medio de AB e a
reta que passa por Q e X intersecta 1 e 2 novamente em Y e Z, respectivamente. Prove
que X e o ponto medio de Y Z.

POT 2012 - Geometria - Nvel 2 - Aula 11 - Prof. Ccero Thiago

Soluc
ao.
PotX
2 = XP XB = XZ XQ,
PotX
1 = XP XA = XY XQ.
Entao,
XP XB
XZ XQ
=

XP XA
XY XQ
XY = XZ.

1
b

2
P

X
A

b
b

O1
b

O2

Problema 2. (OCM) Duas tangentes OA e OB s


ao tracadas a um crculo de um ponto
externo O. Uma corda AC e construda paralela a OB e uma secante OC e desenhada
intersectando o crculo em E. Se K e o ponto de intersecao de OB com o prolongamento
de AE, prove que OK = KB.

POT 2012 - Geometria - Nvel 2 - Aula 11 - Prof. Ccero Thiago

Soluc
ao.
Temos que KOC = ECA pois OB k AC e ECA = EAO pois OA e tangente ao
crculo. Entao OKE AKO assim
OK
KE
=
OK 2 = KE KA.
KA
OK

Usando a potencia de K com relac


ao `
a circunferencia temos
KB 2 = KE KA.
Portanto, OK = KB.

A
b

E
b

Problema 3. Seja C uma semicircunferencia de centro O e di


ametro AB e D e o ponto
medio do arco AB. Sobre a reta OD toma - se o ponto E, do mesmo lado de D com relacao
a AB, tal que OE = BD. Se BE corta a semicircunferencia em F e P e o ponto de AB
AB
tal que F P e perpendicular a AB. Prove que BP =
.
3
Soluc
ao.

2 e
Sem perda
de
generalidade
fa
c
a
OA
=
OB
=
1.
Logo,
OD
=
1,
OE
=
BD
=

EB = 3. Utilizando a potencia de E com relacao `a circunferencia de di


ametro AB temos
EF EB = EO 2 R2 = EO2 1.
7

POT 2012 - Geometria - Nvel 2 - Aula 11 - Prof. Ccero Thiago

Assim,
EF

3 = ( 2)2 1 EF =

3
2 3
e FB =
.
3
3

Alem disso, BP F BOE ent


ao
BF
2
BP
=
BP = .
BO
BE
3
Portanto,

2
1
AB
BP
= 3 = BP =
.
AB
2
3
3
E
b

F
b

b
b

Problema 4. Considere tres crculos 1 , 2 e 3 tais que seus centros O1 , O2 e O3 , respectivamente, n


ao est
ao alinhados. Sejam r, s e t os eixos radicais de 1 e 2 , 1 e 3 e 2 e 3 ,
respectivamente. Prove que r, s e t s
ao concorrentes em um ponto chamado centro radical.
Soluc
ao.
8

POT 2012 - Geometria - Nvel 2 - Aula 11 - Prof. Ccero Thiago

Seja P um ponto sobre r s, ou seja, P possui a mesma potencia com relacao 1 , 2 e 3 .


Portanto, P est
a sobre a reta t.

O1
b

O2
b

P
b

O3

Exerccios propostos
1. Em um tri
angulo ABC, a bissetriz do angulo A e a mediana relativa a BC intersectam este lado em pontos distintos O e M , respectivamente. O crculo circunscrito ao
tri
angulo AOM intersecta os lados AB e AC em E e F , respectivamente. Prove que
BE = CF .
2. Seja BD a bissetriz do
angulo B do tri
angulo ABC. Se o crculo circunscrito ao
tri
angulo BDC intersecta AB em E e o crculo circunscrito ao tri
angulo ABD intersecta BC em F , prove que AE = CF .
3. Um tri
angulo acut
angulo ABC est
a inscrito numa circunferencia de centro O. As
alturas do tri
angulo s
ao AD, BE e CF . A reta EF intersecta a circunferencia em P
e Q.
9

POT 2012 - Geometria - Nvel 2 - Aula 11 - Prof. Ccero Thiago

(a) Prove que OA e perpendicular a P Q.


(b) Se M e o ponto medio de BC, prove que AP 2 = 2AD.OM .
4. Seja C um ponto sobre o semicrculo de di
ametro AB e seja D o ponto medio do arco
AC. Se E e a projec
ao de D sobre BC e F e a intersecao de AE com o semicrculo,
prove que BF bissecta o segmento DE.
5. Seja P um ponto no interior de um crculo tal que existem tres cordas que passam
por P e tem o mesmo comprimento. Prove que P e o centro do crculo.
6. Sejam 1 e 2 crculos concentricos, com 2 no interior de 1 . Partindo de um ponto
A pertencente a 1 , e desenhada uma tangente AB `a 2 (B 2 ). Seja C o segundo
ponto de intersec
ao de AB com 1 , e D o ponto medio de AB. Um reta passando
por A intersecta 2 em E e F de tal maneira que as mediatrizes de DE e CF se
AM
.
intersectam em um ponto M sobre AC. Determine a raz
ao
MC
7. (IMO) Seja ABC um tri
angulo com circuncentro O. Sejam P e Q pontos no interior
dos lados CA e AB, respectivamente. Sejam K, L e M os pontos medios dos segmentos BP , CQ e P Q, respectivamente, e seja o crculo que passa por K, L e M .
Se P Q e tangente a , prove que OP = OQ.
8. (IMO) Um crculo de centro O passa pelos vertices A e C de um tri
angulo ABC e
intersecta os segmentos AB e BC novamente em pontos distintos K e N , respectivamente. Os crculos circunscritos aos tri
angulos ABC e KBN se intersectam em
exatamente 2 pontos distintos B e M . Prove que OM B = 90 .

Bibliografia
1. Problemas de las olimpiadas matematicas del Cono Sur (I a a IV a )
Fauring - Wagner - Wykowski - Gutierrez - Pedraza - Moreira
2. Olimpadas Cearenses de Matematica - Ensino Fundamental - 1981 - 2005
Emanuel Carneiro, Francisco Ant
onio M. de Paiva e Onofre Campos
3. Potencia de um ponto em relac
ao a uma circunferencia
Eduardo Wagner
Revista do professor de matem
atica - N
umero 45

10

POT 2012 - Geometria - Nvel 2 - Aula 11 - Prof. Ccero Thiago

4. Mathematical Olympiad Challenges


Titu Andreescu e Razvan Gelca
5. Lecture Notes on Mathematical Olympiad Courses - For senior section - Vol. 1
Xu Jiagu
6. T
opicos de Matematica Elementar - Volume 2
Antonio Caminha Muniz Neto

11

Polos Olmpicos de Treinamento


Aula

Curso de Geometria - Nvel 2

12

Prof. Ccero Thiago

Relac
oes entre
areas I

Teorema 1. (F
ormula tradicional.)

A area do tri
angulo ABC pode ser calculada por [ABC] =

BC AD
.
2

Teorema 2. (Area
de um tri
angulo em fun
c
ao do raio da circunfer
encia inscrita.)
Sejam a, b e c as medidas dos lados BC, CA e AB do tri
angulo ABC, respectivamente, e
seja r a medida do raio da circunferencia inscrita. Entao, a area do tri
angulo ABC pode
ser calculada por
[ABC] = p r,
em que p =

a+b+c
.
2

Demonstrac
ao.

POT 2012 - Geometria - Nvel 2 - Aula 12 - Prof. Ccero Thiago

A
b

r
b

r
b
b

[ABC] = [BIC] + [CIA] + [AIB]


ar br cr
+
+

2
2
2

a+b+c
r
[ABC] =
2

[ABC] =

[ABC] = p r.
Teorema 3. (F
ormula trigonom
etrica da
area de um tri
angulo.)
Sejam a, b e c as medidas dos lados BC, CA e AB do tri
angulo ABC, respectivamente.
A area do tri
angulo ABC pode ser calculada por
[ABC] =

b c sinA
a c sinB
a b sinC
=
=
.
2
2
2

Demonstrac
ao. Vamos demonstrar uma das igualdades. As outras s
ao an
alogas.

POT 2012 - Geometria - Nvel 2 - Aula 12 - Prof. Ccero Thiago

B
b

Seja A = . Temos que

AC BD
aH
=
.
2
2
H
H = c sin , entao
Por outro lado, no tri
angulo ABD, temos sin =
c
[ABC] =

[ABC] =

a c sin
.
2

Teorema 4. (Area
de um tri
angulo em fun
c
ao do raio da circunfer
encia circunscrita.)
Sejam a, b e c as medidas dos lados BC, CA e AB do tri
angulo ABC, respectivamente,
e seja R o raio da circunferencia circunscrita. Entao, a area do tri
angulo [ABC] pode ser
calculada por
abc
[ABC] =
.
4R
Demonstrac
ao.

POT 2012 - Geometria - Nvel 2 - Aula 12 - Prof. Ccero Thiago

A
b

O
b

b
b

Sejam a, b e c as medidas dos lados BC, CA e AB do tri


angulo ABC, respectivamente.
Temos que
a c sin
[ABC] =
.
2
Por outro lado, seja AD um di
ametro entao, no ACD, temos que
sin =

b
.
2R

Portanto,
[ABC] =

abc
.
4R

Teorema 5. (F
ormula de Heron.)
Sejam a, b e c as medidas dos lados BC, CA e AB do tri
angulo ABC, respectivamente.
Entao, a area do tri
angulo ABC pode ser calculada por
[ABC] =
em que p =

a+b+c
.
2

p (p a) (p b) (p c),

Demonstrac
ao.

POT 2012 - Geometria - Nvel 2 - Aula 12 - Prof. Ccero Thiago

A
b

am

Aplicando o teorema de Pit


agoras nos tri
angulos ABD e ACD, temos:
1. c2 = m2 + h2 .
2. b2 = (a m)2 + h2 .
De (2), temos:
b2 = (a m)2 + h2
b2 = a2 2am + m2 + h2
b2 = a2 2am + c2
m=

a2 + c2 b2
.
2a

Substituindo em (1), temos:


2

c =

a2 + c2 b2
2a

h =c
2

h =
2

h =

+ h2

a2 + c2 b2
2a

a2 + c2 b2
c
2a

a2 + c2 b2
c+
2a

2ac + a2 + c2 b2
2a

2ac a2 c2 + b2
2a

4a2 h2 = [(a + c)2 b2 ] [(b2 (a c)2 ]


4a2 h2 = (a + c + b) (a + c b) (b + a c) (b + c a)
4a2 h2 = (a + b + c) (b + c a) (a + c b) (a + b c)
4a2 h2 = 2p (2p 2a) (2p 2b) (2p 2c)
a2 h2
= p (p a) (p b) (p c)
2
5

POT 2012 - Geometria - Nvel 2 - Aula 12 - Prof. Ccero Thiago

[ABC]2 = p (p a) (p b) (p c)
[ABC] =

p (p a) (p b) (p c).

Teorema 6. (Rela
c
ao entre as
areas de tri
angulos semelhantes.)
Sejam ABC e DEF dois tri
angulos semelhantes tais que
[ABC]
= k2 .
[DEF ]

AC
BC
AB
=
=
= k, entao
DE
DF
EF

Demonstrac
ao.
Se ABC DEF com

AC
BC
AG
AB
=
=
=
= k, entao
DE
DF
EF
DH

BC AG
[ABC]
BC AG
2
=
=

= k k = k2 .
EF DH
[DEF ]
EF DH
2
A
b

D
b

Teorema 7. Sejam r e s retas paralelas. Sejam A e B pontos distintos sobre a reta s e C1


e C2 pontos distintos sobre a reta r. Entao, [ABC1 ] = [ABC2 ].
Demonstrac
ao. O resultado e imediato pois [ABC1 ] = [ABC2 ] =

AB H
.
2

POT 2012 - Geometria - Nvel 2 - Aula 12 - Prof. Ccero Thiago

r
b

C1
b

C2

s
b

Teorema 8. (Usando
areas para calcular raz
ao de segmentos.)
Seja ABC um tri
angulo e D, E e F pontos sobre os lados BC, CA e AB tais que AD,
BE e CF s
ao concorrentes no ponto P . Defina K = [ABC], KA = [P BC], KB = [P CA]
e KC = [P AB]. Como K = KA + KB + KC , entao
(a)
KC CE
KA AF
KB
BD
=
,
=
e
=
.
DC
KB EA
KC F B
KA
(b)
AP
KB + KC BP
KA + KC CP
KA + KB
=
,
=
e
=
PD
KA
PE
KB
PF
KC
Demonstrac
ao.
A
b

F
b

H2

P
b

H1
b

(a) Temos que


BD
[ABD]
[BP D]
[ABD] [BP D]
[AP B]
KC
=
=
=
=
=
.
CD
[ACD]
[CP D]
[ACD] [CP D]
[ACP ]
KB
7

POT 2012 - Geometria - Nvel 2 - Aula 12 - Prof. Ccero Thiago

Da mesma maneira demonstra - se que

KA AF
KB
CE
=
=
e
.
EA
KC F B
KA

(b) Temos que


ADS P DR
H2
[ABC]
KA + KB + KC
AD
=
=
=

PD
H1
[BP C]
KA
AP
KB + KC
=
.
PD
KA
Da mesma maneira demonstra - se que

KA + KC CP
KA + KB
BP
=
=
e
.
PE
KB
PF
KC

Teorema 9. (Area
de quadril
atero convexo qualquer.)
Seja ABCD um quadril
atero convexo qualquer tal que e o menor angulo entre as diagoAC BD sin
nais. Entao, [ABCD] =
.
2
Demonstrac
ao.
Temos que
[ABCD] = [AP D] + [BP C] + [CP D] + [DP A]
P A P D sin P A P B sin P B P C sin P C P D sin
+
+
+

2
2
2
2
(P A P D + P A P B + P B P C + P C P D) sin

[ABCD] =
2
AC BD sin
(P A + P C)(P B + P D) sin
[ABCD] =
.
[ABCD] =
2
2
Exerccios Resolvidos
[ABCD] =

1. (Olimpada de Maio) ABC e um tri


angulo equilatero. N e o ponto do lado AC tal
que AC = 7AN , M e o ponto do lado AB tal que M N e paralelo a BC e P o ponto
area(M N P )
do lado BC tal que M P e paralelo a AC. Determine o valor de
.
area(ABC)
facil ver que CP M N e um paralelogramo e, com isso, area(M N P ) =
Soluc
ao. E
2
1
area(AM N )

1
area(BM P )
6
36
area(CP M N ). Alem disso,
=
e
=
.
=
2
area(ABC)
49
area(ABC)
7
49
Portanto,
36
1

1
area(P M N )

49 49 = 6 .
=
area(ABC)

2
49

POT 2012 - Geometria - Nvel 2 - Aula 12 - Prof. Ccero Thiago

2. S
ao dados 1000 pontos no plano n
ao colineares tais que se tres deles determinam um
tri
angulo ent
ao sua
area e menor ou igual a 1. Prove que todos os pontos est
ao em
um tri
angulo de
area menor ou igual a quatro.
Soluc
ao.
b

b
b

Y
D
b

b
b

Como existe um n
umero finito de tri
angulos que podem ser construdos usando os
1000 pontos ent
ao, escolhemos aquele de area m
axima que chamaremos de XY Z.
Seja ABC o tri
angulo tal que X, Y e Z s
ao os pontos medios de BC, CA e AB,
respectivamente, ent
ao [ABC] = 4[XY Z] 4. Seja D, um ponto no conjunto
dos 1000 pontos dados, no exterior do tri
angulo ABC entao [XY Z] < [XZD],
o que contradiz a escolha de ABC. Portanto, todos os pontos est
ao no interior do
tri
angulo ABC.
3. (Coreia) Seja ABCD um quadril
atero convexo e seja P o ponto de intersecao das
diagonais. Prove que
[P AB] + [P CD] = [P BC] + [P DA]
se, e somente se, P e o ponto medio de AC ou BD.
1
PA PB
4
P C P D sin P . Os n
umeros [P AB], [P CD] e [P BC], [P DA] tem a mesma
soma e o mesmo produto, ent
ao [P AB] = [P BC] e [P CD] = [P DA] ou
[P AB] = [P DA] e [P BC] = [P CD], ou seja, P e o ponto medio de AC ou
BD.
Soluc
ao. Observe que [P AB] [P CD] = [P BC] [P DA] =

4. (OCM) Os lados de um tri


angulo s
ao expressos, em cm, por tres inteiros consecutivos
2
e sua area, em cm , e dada por um inteiro. Prove que o menor lado do tri
angulo e
mpar.

POT 2012 - Geometria - Nvel 2 - Aula 12 - Prof. Ccero Thiago

Soluc
ao.
Sejam x1, x, x+1 os lados do tri
angulo. Pela formula de Heron, a area do tri
angulo
e

3x (x + 2) x (x 2)
[ABC] =


2
2
2
2
=

1 2 2
3x2 (x2 4)
=
3x (x 4).
16
4

Como [ABC] Z, devemos ter 3x2 (x2 4) par, o que nos diz que x deve ser par.
Portanto, o menor lado do tri
angulo, que e x 1, deve ser mpar.
5. (Hong Kong) Seja ABC um tri
angulo e sejam X, Y e Z pontos sobre os lados AB,
4 BY
6 CZ
8
AX
= ,
= e
= . Se a area do
BC e CA, respectivamente, tais que
XB
5 YC
7 ZA
9
tri
angulo ABC e 1989, determine a area do tri
angulo XY Z.
Soluc
ao.
[XY Z]
=1
1989

[AXZ] [BXY ] [CY Z]


+
+
1989
1989
1989

4 9
5 6
7 8
=1

+
+

9 17 9 13 13 17
1482
,
1
1989
Portanto, a
area do tri
angulo XY Z e 1989 1482 = 507.

Exerccios Propostos

1. No tri
angulo ABC, os pontos L, M e N est
ao sobre BC, CA e AB respectivamente,
e AL, BM e CN s
ao concorrentes no ponto P .
(a) Encontre o valor numerico de
PL PM
PN
+
+
AL BM
CN
(b) Encontre o valor numerico de
AP
BP
CP
+
+
AL BM
CN

10

POT 2012 - Geometria - Nvel 2 - Aula 12 - Prof. Ccero Thiago

2. (Ibero) Se AD, BE e CF s
ao tres cevianas concorrentes no circuncentro O do
tri
angulo ABC, demonstre que
1
1
2
1
+
+
= .
AD BE CF
R
3. (AIME) Num tri
angulo ABC, A1 , B1 e C1 est
ao sobre os lados BC, CA e AB,
respectivamente. Dado que AA1 , BB1 e CC1 s
ao concorrentes no ponto O, e que
AO
BO
CO
AO BO CO
+
+
= 92. Encontre o valor de

.
OA1 OB1 OC1
OA1 OB1 OC1
4. Em um ABC, AD, BE e CF s
ao concorrentes no ponto P tal que AP = P D = 6,
EP = 3, P B = 9 e CF = 20. Qual e a area do ABC?
5. Em um tri
angulo ABC, sejam S o ponto medio da mediana correspondente ao vertice
A e Q o ponto de intersec
ao de BS com o lado AC. Demonstrar que BS = 3QS.
6. Tres segmentos C1 A2 , C2 B1 e A1 B2 com extremos sobre os lados do tri
angulo ABC
s
ao paralelos aos lados e passam pelo ponto P . Prove que as areas dos tri
angulos
A1 B1 C1 e A2 B2 C2 s
ao iguais.
dado um quadril
7. (OBM) E
atero convexo ABCD. Sejam E, F, G e H os pontos
medios dos lados AB, BC, CD e DA, respectivamente. Determine a posicao de um
ponto P de forma que os quadril
ateros P HAE, P EBF , P F CG e P GDH tenham a
mesma
area.
8. Seja ABCDE um pent
agono convexo (n
ao necessariamente regular) tal que os tri
angulos
ABC, BCD, CDE, DEA e EAB tem area 1. Qual a area do pentagono?
9. Seja ABCD um quadril
atero convexo e EH, EI, EF e EG s
ao segmentos paralelos
e iguais a AB, BC, CD e DA, como mostra a figura abaixo. Determine a raz
ao entre
as areas dos tri
angulos HIF G e ABCD.
G
C
D

F
E

H
A

B
I

11

POT 2012 - Geometria - Nvel 2 - Aula 12 - Prof. Ccero Thiago

10. (AIME) Quadrados S1 e S2 s


ao inscritos em um tri
angulo ret
angulo ABC, como
mostrado na figura abaixo. Determine AC + CB se area(S1 ) = 441 e area(S2 ) = 440.
A

A
b

S2

S1
b
b

11. Seja P um ponto no interior de um tri


angulo equilatero ABC, e sejam D, E e F os
simetricos de P em relac
ao aos lados BC, CA e AB, respectivamente. Qual e maior,
a area do tri
angulo ABC ou a
area do tri
angulo DEF ?
C
E
D
P

B
F

12. (Portugal) Seja [ABC] um tri


angulo ret
angulo em A. Considere um ponto E sobre
a hipotenusa e traca - se a partir desse ponto uma paralela ao cateto AC. Seja a
intersec
ao desta paralela com o cateto AB. Prove que
BC 2
BD DE
+
=
,
DE BD
2S
sendo S a
area do tri
angulo ABC.
13. (Portugal) Os lados AB, BC e AC do tri
angulo representado na figura medem, respectivamente, 7, 11 e 8. Tracam - se W R, U P e V Q, perpendiculares aos lados.
12

POT 2012 - Geometria - Nvel 2 - Aula 12 - Prof. Ccero Thiago

Sabendo que U W mede 2, determine a raz


ao entre a area do tri
angulo U V W e a
area do tri
angulo ABC.
A
b

R
b

U
b

V
b

b
b

14. (OBM) ABCD e um quadril


atero convexo e inscritvel e M e um ponto sobre o lado
CD, tal que o tri
angulo ADM e o quadril
atero ABCM tem a mesma area e o mesmo
permetro. Prove que ABCD tem dois lados de comprimentos iguais.
15. Os pontos medios das diagonais AC, CE, EA, BD, DF e F B do hexagono convexo
ABCDEF s
ao vertices de um novo hexagono. Calcular a relacao entre as areas do
dois hexagonos.
16. (Mandelbrot) Seja ABCD um quadril
atero convexo tal que AB = 12, BC = 6 e
CD = 20. Suponha que ABCD possui uma circunferencia inscrita que e tangente ao
lado BC em seu ponto medio. Qual e a area do quadril
atero ABCD?
Bibliografia
1. Colec
ao Elementos da Matematica, vol.2 - Geometria Plana
Marcelo Rufino de Oliveira e Marcio Rodrigo da Rocha Pinheiro.
2. Olimpadas Cearenses de Matematica, Ensino Medio, 1981 - 1985
Emanuel Carneiro, Francisco Ant
onio M. de Paiva e Onofre Campos.
3. Olimpadas de Matematica, Categoria B, 10 , 11 e 12 anos, vol.1
Jorge Picado e Paulo Eduardo Oliveira.

13

POT 2012 - Geometria - Nvel 2 - Aula 12 - Prof. Ccero Thiago

4. T
opicos de Matematica Elementar, vol.2, Geometria Euclidiana Plana
Antonio Caminha Muniz Neto.
5. Area y Volumen, en la geometria elemental.
Jose Araujo, Guilermo Keilhauer, Norma Pietrocola e Valeri Vavilov.
6. Which Way did the Bicycle Go? And other intriguing mathematical mysteries
Joseph D. E. Konhauser, Dan Velleman e Stan Wagon.
7. 360 Problems for Mathematical Contests
Titu Andreescu e Dorin Andrica.

8. Areas
para achar raz
oes de segmentos
Ccero Thiago e Marcelo Mendes.
Revista Eureka 25
9. Mathematical Olympiad Treasures
Titu Andreescu e Bogdan Enescu
10. Mandelbrot Morsels
Sam Vandervelde.

14

Polos Olmpicos de Treinamento


Aula

Curso de Geometria - Nvel 2

13

Prof. Ccero Thiago

Revis
ao I
Problema 1. Na figura abaixo tem - se AD = DE, A = DEC e ADE = BDC.
Mostre que os tri
angulos ABD e EDC s
ao congruentes.
D
C

Soluc
ao. Como ADE = BDC ent
ao ADB = EDC. Portanto, ABD EDC
pelo caso ALA.
Problema 2. (OCM) Um tri
angulo ABC e tal que C = 2A e AC = 2BC. Prove que
este tri
angulo e ret
angulo.
Soluc
ao. Seja CD a bissetriz interna do angulo C. Entao, BCD = DCA = DAC e
AC
=
o tri
angulo CDA e is
osceles. Trace a altura DE deste tri
angulo. Observe que CE =
2
BC. Da, os tri
angulos BCD e ECD s
ao congruentes, pelo caso LAL, de onde conclumos
que CBD = CED = 90 , e o tri
angulo ABC e ret
angulo em B.
C
b

D
b

POT 2012 - Geometria - Nvel 2 - Aula 13 - Prof. Ccero Thiago

Problema 3. Seja ABC um tri


angulo is
osceles, com AB = BC e ABC = 82 . Seja M
um ponto no interior do tri
angulo tal que AM = AB e M AC = 11 . Ache a medida do
angulo M CB.
Soluc
ao.

B
b

M
b

b
b

facil ver que M AB = 38 . Como AM = AB, entao ABM = AM B = 71 . Dessa


E
forma, M BC = 11 . Seja D o ponto sobre o segmento AC tal que AD = BM . Assim,
M BC DAM pelo caso LAL. Portanto, M D = M C e BCM = AM D = . Pela
propriedade do
angulo externo temos que M DC = 11 + e, como M D = M C entao

11 + = 49 = M CB = 19 .
Problema 4. Seja ABC um tri
angulo is
osceles de base AC tal que B = 20 . Prove que
AB < 3AC.
Soluc
ao. Inicialmente construa os tri
angulos BCD e DBE congruentes ao tri
angulo ABC
como feito na figura abaixo. Dessa forma e facil ver que o tri
angulo ABE e equilatero.
facil verificar, usando a desigualdade triangular, que AC +CD+DE >
Entao, AE = AB. E
AE. Como os tri
angulos BCD, DBE e ABC s
ao congruentes entao AC = CD = DE.
Portanto, AB < 3AC.

POT 2012 - Geometria - Nvel 2 - Aula 13 - Prof. Ccero Thiago

B
b

20

20 20

80

E
b

80
80
b

80

80

80

Problemas propostos

1. Sobre os lados de um tri


angulo ABC constroem - se externamente os tri
angulos
equilateros BCD, CAE e ABF . Prove que os segmentos AD, BE e CF s
ao congruentes.
2. Mostre que a hipotenusa de um tri
angulo ret
angulo e maior que a semi - soma dos
catetos.
3. (Torneio das Cidades) Se a, b e c s
ao os comprimentos dos lados de um tri
angulo,
prove que a3 + b3 + 3abc > c3 .
4. Seja ABC um tri
angulo tal que ABC = 2BCA, ademais, seja D o ponto do lado
BC tal que AD e bissetriz do
angulo CAB e CD = AB. Calcule as medidas dos
angulos do tri
angulo ABC.
5. Seja ABC um tri
angulo is
osceles de base AC tal que B = 20 . Prove que AB >
3

POT 2012 - Geometria - Nvel 2 - Aula 13 - Prof. Ccero Thiago

2AC.
6. Seja ABC um tri
angulo tal que A = 20 . Sejam D e E pontos sobre os lados AC e
AB, respectivamente, tais que AED = 40 e ED = DC = BC, determine a medida
do angulo B.
7. (OBM) O canto de um quadrado de cartolina foi cortado com uma tesoura. A soma
dos comprimentos dos catetos do tri
angulo recortado e igual ao comprimento do lado
do quadrado. Qual o valor da soma dos angulos e marcados na figura abaixo?

27

8. Seja ABC um tri


angulo ret
angulo em C com AC < BC. Sejam D o ponto do lado AC
BK
e seja K o ponto do segmento BD tais que KAD = AKD = ABC. Calcule
.
CD

Polos Olmpicos de Treinamento


Aula

Curso de Geometria - Nvel 2

14

Prof. Ccero Thiago

Teorema de Ceva e Teorema de Menelaus.

Teorema 1. (Ceva) Sejam D, E e F pontos sobre os lados BC, AC e AB, respectivamente, do tri
angulo ABC. Os segmentos AD, BE e CF intersectam - se em um ponto
BD CE AF

= 1.
P se, e somente se,
CD EA F B
Demonstrac
ao.

A
b

F
b

b
b

Defina K = [ABC], KA = [P BC], KB = [P CA] e KC = [P AB].


Temos que
BD
[ABD]
[BP D]
[ABD] [BP D]
[AP B]
KC
=
=
=
=
=
.
CD
[ACD]
[CP D]
[ACD] [CP D]
[ACP ]
KB
KA AF
KB
BD CE AF
KC KA KB
CE
=
e
=
. Assim,

= 1.
EA
KC F B
KA
CD EA F B
KB KC KA
BD CE AF
Sejam D, E e F pontos sobre os lados BC, CA e AB tais que

=1
CD EA F B
mas AD, BE e CF n
ao s
ao concorrentes. Seja F1 sobre AB tal que AD, BE e CF1 s
ao
De maneira an
aloga,

POT 2012 - Geometria - Nvel 2 - Aula 14 - Prof. Ccero Thiago

concorrentes em P . Assim,

BD CE AF1
AF
AF1

= 1. Dessa forma,
=
F = F1 .
CD EA F1 B
FB
F1 B
A
b

F1
b

F
b

Exerccios resolvidos
1. Prove que as medianas de um tri
angulo s
ao concorrentes em um ponto que se chama
baricentro.
Soluc
ao.
Sejam M , N e R os pontos medios de AC, BC e BA, respectivamente. Entao
AM CN BR

= 1,
M C N B RA
ou seja, AN , BM e CR s
ao concorrentes.
2. Prove que as bissetrizes internas de um tri
angulo s
ao concorrentes em um ponto que
se chama incentro.
Soluc
ao.
Sejam X, Y e Z os pes das bissetrizes relativas aos lados BC, AC e AB, respectivamente. Pelo teorema das bissetrizes internas temos que
AY CX BZ
AB CA BC

= 1,
Y C XB ZA
BC AB CA
ou seja, AX, BY e CZ s
ao concorrentes.
3. Prove que as alturas de um tri
angulo s
ao concorrentes em um ponto que se chama
ortocentro.

POT 2012 - Geometria - Nvel 2 - Aula 14 - Prof. Ccero Thiago

Soluc
ao.

A
b

M
b

N
b

b
b

b
b

facil ver que


Sejam AL, BM e CN as alturas do tri
angulo ABC. E
AN C AM B

AC
AN
=
(I)
MA
AB

BL
AB
=
(II)
NB
BC
CM
BC
CM B CLA
=
(III).
LC
AC
BLA BN C

Multiplicando (I), (II) e (III) temos que


AN BL CM
AC AB BC

= 1,
M A N B LC
AB BC AC
ou seja, as alturas s
ao concorrentes.
4. Seja ABCDEF um hexagono convexo tal que cada uma das diagonais AD, BE e
CF dividem o hexagono em duas regi
oes de areas iguais. Prove que AD, BE e CF
s
ao concorrentes.
Soluc
ao.

POT 2012 - Geometria - Nvel 2 - Aula 14 - Prof. Ccero Thiago

F
b
b

A
Y
b

b
b

X
b
b

Sejam X a intersecc
ao de AD e CE, Y a interseccao de AE e CF e Z e a interseccao
de AC e BE. Denotaremos por [M N P ] a area do tri
angulo M N P , e seja K a area
facil ver que
do hexagono ABCDEF . E
K
[ABC]
[ACX]
[CDX]
[ACX] + [CDX]
[ACD]
CX
=
=
=
=
= 2
.
K
XE
[AXE]
[DEX]
[AXE] + [DEX]
[ADE]
[AEF ]
2
De maneira an
aloga,
K
[CDE]
EY
= 2
K
YA
[ABC]
2
e
K
[AF E]
AZ
.
= 2
K
ZC
[CDE]
2
Portanto,
K
K
K
[ABC]
[CDE]
[AF E]
CX EY AZ
2
2
= 1.

= 2
K
K
K
XE Y A ZC
[AEF ]
[ABC]
[CDE]
2
2
2
4

POT 2012 - Geometria - Nvel 2 - Aula 14 - Prof. Ccero Thiago

Pela recproca do teorema de Ceva no tri


angulo ACE temos que AX, CY e EZ
s
ao concorrentes e, com isso, AD, BE e CF s
ao concorrentes.
5. Seja ABC um tri
angulo e sejam P e Q pontos sobre os lados AB e AC, respectivamente, tais que P Q k BC. Prove que P C, QB e a mediana AM , com M em BC,
s
ao concorrentes.
Soluc
ao.
Como P Q k BC, ent
ao
AP
AQ
AP QC
=

= 1 (I).
PB
QC
P B AQ
Como AM e um mediana ent
ao BM = M C, assim
BM
= 1 (II).
MC
Multiplicando (I) e (II), temos
AP QC BM

= 1.
P B AQ M C
Pela recproca do teorema de Ceva temos que AM , QB e P C s
ao concorrentes.
b

P
b

b
b

Exerccios propostos

POT 2012 - Geometria - Nvel 2 - Aula 14 - Prof. Ccero Thiago

1. Sejam D, E e F os pontos de contato da circunferencia inscrita com os lados BC,


CA e AB, respectivamente, do tri
angulo ABC. Prove que AD, BE e CF s
ao concorrentes em um ponto que se chama Ponto de Gergonne.
2. Sejam l e l1 duas retas paralelas dadas no plano. Usando apenas regua encontre o
ponto medio do segmento AB que est
a na reta l.
3. (Coreia) Seja ABC um tri
angulo com AB 6= AC, seja V a interseccao da bissetriz
do angulo A com BC e seja D pe da altura relativa ao vertice A. Se E e F s
ao as
intersecc
oes dos crculos circunscritos aos tri
angulos AV D com CA e AB, respectivamente, mostre que AD, BE e CF s
ao concorrentes.
4. Seja P um ponto no interior de um tri
angulo. As bissetrizes de BP C, CP A e
AP B intersectam BC, CA e AB em X, Y e Z, respectivamente. Prove que AX,
BY e CZ s
ao concorrentes.

Teorema 2. Se uma reta intersecta as retas BC, CA e AB de um tri


angulo ABC nos
pontos L, M e N , respectivamente, entao
CL BN AM

= 1.
BL N A M C
Inversamente, se L, M e N s
ao pontos sobre os lados BC, CA e AB do tri
angulo ABC
CL BN AM
tais que

= 1, ent
ao L, M e N s
ao colineares.
BL N A M C
Demonstrac
ao.
A
b

Q
b

N
b
b

P
b

M
b

Sejam AP , BQ e CR as perpendiculares tracadas a partir de A, B e C, respectivamente,


facil ver que os tri
`a reta em que se encontram L, M e N . E
angulos ret
angulos AP N e
6

POT 2012 - Geometria - Nvel 2 - Aula 14 - Prof. Ccero Thiago

BQN s
ao semelhantes, assim como os tri
angulos ret
angulos QBL e RCL. Entao
BN
BQ CL
RC
=
e
=
.
AN
AP
BL
QB
Por outro lado, os tri
angulos ret
angulos AP M e CRM tambem s
ao semelhantes. De modo
que
AM
AP
=
.
CM
CR
Portanto,
BQ RC AP
BN CL AM

= 1.
AN BL CM
AP QB CR

A
b

N
b

N1
b

CL BN AM

= 1 e os pontos L, M e N n
ao s
ao
Suponha, de maneira falsa, que
BL N A M C
colineares. Prolongue LM ate intersectar AB em N1 . Pelo que foi provado acima temos
CL BN1 AM
que

= 1, assim
BL N1 A M C
BN1
BN
=
N = N1 .
N1 A
NA
Dessa forma, L, M e N s
ao colineares.
Exerccios resolvidos
1. Dadas tres circunferencias C1 , C2 e C3 de centros O1 , O2 e O3 e raios r1 , r2 e r3 ,
respectivamente. Seja X a interseccao das tangentes comuns externas de C1 e C2 ,
Y a intersecc
ao das tangentes comuns externas de C1 e C3 e, finalmente, Z a interseccao das tangentes comuns externas de C2 e C3 . Prove que X, Y e Z s
ao colineares.
facil verificar que X, O1 e O2 s
Soluc
ao. E
ao colineares. Assim, XO1 P1 XO2 P2
O1 P1
r1
O3 Y
r3 O2 Z
r2
O1 X
=
= . Analogamente,
=
e
= . Portanto,
e, com isso,
O2 X
O2 P2
r2
O1 Y
r1 O3 Z
r3
7

POT 2012 - Geometria - Nvel 2 - Aula 14 - Prof. Ccero Thiago

O1 X O3 Y O2 Z

= 1.
O2 X O1 Y O3 Z
Pela recproca do teorema de Menelaus conclumos que X, Y e Z s
ao colineares. Este
resultado e conhecido como teorema de Monge.
Z
b

Y
b

O2
O1

b
b

X
b

P1
b

P2

O3
b

2. Prove que as bissetrizes internas de dois angulos de um tri


angulo is
osceles e a bissetriz
externa do terceiro
angulo do tri
angulo intersectam os lados opostos em tres pontos
colineares.
Soluc
ao.

POT 2012 - Geometria - Nvel 2 - Aula 14 - Prof. Ccero Thiago

No tri
angulo ABC, BM e CN s
ao bissetrizes internas dos angulos B e C, respectivamente, e AL e a bissetriz externa do angulo A. Pelo teorema da bissetriz
interna temos que
AB BN
BC
AM
=
e
=
.
MC
BC N A
AC
Alem disso, pelo teorema da bissetriz externa temos que
CL
AC
=
.
BL
AB
Assim,
AM BN CL
AB BC AC

= 1.
M C N A BL
BC AC AB
Pela recproca do teorema de Menelaus temos que N , M e L s
ao colineares.

A
b

N
b

b
b

Exerccios propostos

1. Prove que as bissetrizes externas dos angulos de um tri


angulo, n
ao is
osceles, intersectam os lados opostos em tres pontos colineares.
2. O ortocentro de um tri
angulo ABC e o ponto medio da altura relativa ao vertice
C. Prove que cos C = cos A cos B, em que A, B e C s
ao os angulos do
tri
angulo ABC.
3. A bissetriz AD de um tri
angulo ABC divide o lado BC na raz
ao 2 : 1. Determine a
raz
ao em que a mediana CE divide a bissetriz.
4. (OBM) No tri
angulo ABC, D e ponto medio de AB e E ponto sobre o lado BC tal
que BE = 2 EC. Sabendo que ADC = BAE, calcule o valor de BAC.

POT 2012 - Geometria - Nvel 2 - Aula 14 - Prof. Ccero Thiago

5. (IMO) As diagonais AC e CE de um hexagono regular ABCDEF s


ao divididas interAM
CN
namente pelos pontos M e N , respectivamente, na raz
ao
=
= r. Determine
AC
CE
r se B, M e N s
ao colineares.
6. Seja ABC um tri
angulo e sejam E e D pontos sobre o lado BC tal que CE = ED =
DB. Seja F o ponto medio de AC e G o ponto medio de AB. Seja H a interseccao
EH
.
de EG e F D. Determine o valor de
HG
7. (Cone Sul) Seja C uma circunferencia de centro O, AB um di
ametro dela e R um
ponto qualquer em C distinto de A e de B. Seja P a interseccao da perpendicular
tracada por O a AR. Sobre a reta OP se marca o ponto Q, de maneira que QP e a
metade de P O e Q n
ao pertence ao segmento OP . Por Q tracamos a paralela a AB
que corta a reta AR em T . Chamamos de H o ponto de interseccao das retas AQ e
OT . Provar que H, R e B s
ao colineares.

Bibliografia
1. Leccture Notes on Mathematical Olympiad Courses
For senior Section, vol. 1
Xu Jiagu
2. Advanced Euclidean Geometry
Alfred Posamentier
3. III Olimpiada Nacional Escolar de Matematica 2006
Jorge Tipe, John Cuya, Claudio Espinoza e Sergio Vera.
4. Explorations in Geometry
Bruce Shawyer
5. Colecao Elementos de Matematica, vol.2
Marcelo Rufino de Oliveira
6. The theorem of Menelaus
B. Orach
Quantum - May/Jun 2001
7. Problemas de Geometra - Planimetria
I. Shariguin

10

Polos Olmpicos de Treinamento


Aula

Curso de Geometria - Nvel 2

15

Prof. Ccero Thiago

Pontos Not
aveis 1: Baricentro
Propriedade 1. As tres medianas de um tri
angulo intersectam - se num mesmo ponto,
chamado baricentro, que divide cada uma das medianas em duas partes tais que a parte
que contem o vertice e o dobro da outra.

Demonstrac
ao.
A

P
D
B

N
G1

E
C

Sejam N e P os pontos medios dos lados AC e AB, respectivamente, D e E os pontos


medios de BG1 e CG1 , respectivamente. Entao,
N P k BC e N P =

BC
2

POT 2012 - Geometria - Nvel 3 - Aula 15 - Prof. Ccero Thiago

BC
2
portanto, P DEN e uma paralelogramo. Com isso, BD = DG1 = G1 N , CE = EG1 =
G1 P , entao BG1 = 2G1 N e CG1 = 2G1 P . De maneira an
aloga, as medianas AM e BN
intersectam - se em um ponto G2 tal que AG2 = 2G2 M e BG2 = 2G2 N . Encontramos,
entao, dois pontos distintos G1 e G2 , no interior do segmento BN que o dividem na mesma
raz
ao, o que e uma contradic
ao logo, G1 = G2 = G. Portanto, as tres medianas intersectam
- se em um mesmo ponto G que chamaremos de baricentro.
DE k BC e DE =

Exerccios Resolvidos
1. (OBM) Seja N o ponto do lado AC do tri
angulo ABC tal que AN = 2N C e M o
ponto do lado AB tal que M N e perpendicular a AB . Sabendo que AC = 12 cm e que o
baricentro G do tri
angulo ABC pertence ao segmento M N , determine o comprimento do
segmento BG.
OBS: Baricentro e o ponto de intersecao das medianas do tri
angulo.
Soluc
ao.
Se BP e uma mediana do tri
angulo ent
ao AP = CP = 6 e P N = 2. Como G e o baricentro
1 PN
1
PG
= e
= , assim, pela recproca do teorema de Tales, GN e
do tri
angulo ent
ao
GB 2 N C
2
paralelo a BC e B = 90 . Como o tri
angulo ABC e ret
angulo entao AP = CP = BP = 6.
Com isso, BG = 4 e GP = 2.

M
b

b
b

2. (Bulgaria) Seja ABC um tri


angulo is
osceles (AC = BC) tal que A1 , B1 e C1 s
ao os
pontos medios de BC, AC e AB, respectivamente. Os pontos A2 e B2 s
ao os simetricos de
A1 e B1 com relac
ao ao lado AB. Seja M a intersecao de CA2 e A1 C1 e seja N a intersecao
de CB2 e B1 C1 . Seja P a intersec
ao de AN e BM , prove que AP = BP .
Soluc
ao. Como CC1 k A1 A2 e CC1 = A1 A2 , temos que CC1 A2 A1 e um paralelogramo.
Entao, A1 M = C1 M . Mas A1 B1 C1 B e tambem um paralelogramo e, portanto, a in2

POT 2012 - Geometria - Nvel 3 - Aula 15 - Prof. Ccero Thiago

tersecao BM e AC e B1 . Ent
ao, P est
a sobre a mediana BB1 . Analogamente, P est
a
sobre a mediana AA1 . No tri
angulo is
osceles ABC as medianas AA1 e BB1 possuem o
2
2
mesmo comprimento. Portanto, AP = AA1 = BB1 = BP .
3
3
Exerccios Propostos
Problema 1. Uma reta r passa pelo baricentro de um tri
angulo ABC deixando o vertice A
em um semiplano e os vertices B e C no outro semiplano determinado por r. As projecoes
de A, B e C sobre a reta r s
ao M , N e P , respectivamente. Prove que AM = BN + CP .
Problema 2. (OBM) Seja ABCD um quadril
atero convexo, onde N e o ponto medio de
DC, M e o ponto medio de BC, e O e a intersecao entre as diagonais AC e BD. Mostre
que O e o baricentro do tri
angulo AM N se, e somente se, ABCD e um paralelogramo.
Problema 3. (Portugal) No tri
angulo ABC as medianas dos lados AB e AC s
ao perpendiculares. Sabendo que AB = 6 e AC = 8, determine BC.
Problema 4. (Estonia) As medianas relativas aos vertices A e B do tri
angulo ABC s
ao
perpendiculares. Prove que AB e o menor lado do tri
angulo ABC.
Problema 5. (OCM) Seja ABC um tri
angulo tal que as medianas BM e CN , que se cortam em G, s
ao iguais. Prove que o tri
angulo ABC e is
osceles.
Problema 6. Prove que a soma dos quadrados das dist
ancias de um ponto P aos vertices
de um tri
angulo ABC e mnima quando P e o baricentro do tri
angulo.
Solu
co
es
1. Seja AD um mediana e Q o ponto medio de N P . Entao, DQ e a base media do trapezio
BN + CP
. Como G e o baricentro do tri
angulo ABC
N BCP assim DQ k BN e DQ =
2
facil ver que AM G GQD, entao AM = DQ. Portanto,
entao AG = 2GD. E
2
AM = BN + CP .

POT 2012 - Geometria - Nvel 3 - Aula 15 - Prof. Ccero Thiago

A
b

G
b

b
b

2. () Suponha que ABCD e um paralelogramo, entao AO = OC e BO = BD. Se M


BD
e N s
ao os pontos medios de BC e CD entao M N k BD e M N =
. E facil concluir
2
DO
BO
, N P k DO e N P =
.
que P e o ponto de medio de OC ent
ao M P k BO, M P =
2
2
Portanto, N P = P M e AO = 2OP , ou seja, O e o baricentro de AM N .
() Suponha que O e o baricentro do tri
angulo AM N entao N P = P M e AO = 2OP . Se
BD
. E facil concluir
M e N s
ao os pontos medios de BC e CD entao M N k BD e M N =
2
BO
, N P k DO e
que P e o ponto de medio de OC ent
ao OP = P C, M P k BO, M P =
2
DO
. Da, AO = OC e DO = OB, ou seja, ABCD e um paralelogramo.
NP =
2

POT 2012 - Geometria - Nvel 3 - Aula 15 - Prof. Ccero Thiago

A
b

b
b

O
b

P
b

b
b

3. Sejam M e N os pontos medios de AB e AC, respectivamente, e G o ponto de encontro


das medianas M C e N B. Aplicando o teorema de Pit
agoras BGM e CN G, temos:
GM 2 + 4GN 2 = GM 2 + GB 2 = BM 2 = 32 = 9
e
4GM 2 + GN 2 = GC 2 + GN 2 = CN 2 = 42 = 16.

Deste modo, 5GM 2 + 5GN 2 = 9 + 16 = 25, logo N M = 5. Portanto, BC = 2 5.

N
b

4. Seja M o baricentro do tri


angulo A1 B1 C1 . Seja A2 um ponto sobre a reta M A tal que
B1 A1 C1 A2 e um paralelogramo. Os pontos B2 e C2 s
ao construdos analogamente. Como
A1 C1 k A1 B1 k C1 B2 ent
ao os pontos A2 , C1 e B2 s
ao colineares e C1 e o ponto medio de
5

POT 2012 - Geometria - Nvel 3 - Aula 15 - Prof. Ccero Thiago

A2 B2 . O mesmo e verdade para os pontos A2 , B1 e C2 e C2 , A1 e B2 . Vamos mostrar que


A2 = A, B2 = B e C2 = C, o que resolve o problema. Assuma que A2 6= A e A est
a entre
A2 e M . Ent
ao C2 est
a entre C e M , B est
a entre B2 e M e consequentemente A2 est
a
entre A e M , que e uma contradic
ao.
5. As medianas intersectam - se no ponto M e a mediana que parte do vertice C intersecta
AB no ponto F . Ent
ao, F e o ponto medio da hipotenusa do tri
angulo ret
angulo ABM ,
ou seja, AB = 2F M . Como M divide a mediana CF na raz
ao 2 : 1, entao AB = CM .
O maior angulo do tri
angulo AM C e o angulo obtuso AM C, portanto AC e o maior lado
deste tri
angulo. Assim, AC > M C = AB. De maneira an
aloga BC > AB.
m
= GN e
6. Seja BM = CN = m. Como G e o baricentro de ABC, temos GM =
3
2m
BG =
= CG. Da, segue que os tri
angulos BGN e CGM s
ao congruentes (pelo caso
3
LAL), de modo que BN = CM . Logo, AB = 2 BN = 2 CM = AC, e o tri
angulo ABC
e is
osceles.

A
b

N
b

G
b

7. Seja ABC um tri


angulo com BC = a, AC = b e AB = c. Seja M o ponto medio de
BC, G o baricentro do tri
angulo ABC e P um ponto qualquer. Usando que, a soma dos
quadrados de dois dos lados de um tri
angulo e igual a duas vezes o quadrado da mediana
relativa ao terceiro lado mais a metade do quadrado do terceiro lado (a demonstracao desse
resultado usa lei dos Cossenos e ser
a provado na aula de relacoes metricas), no tri
angulo
6

POT 2012 - Geometria - Nvel 3 - Aula 15 - Prof. Ccero Thiago

P BC com mediana P M temos:


a2
. (I)
2
O baricentro G e tal que GA = 2GM . Faca GM = m; GA = 2m e tome H em AG tal que
GH = AH = m. Assim, o tri
angulo HP M , com mediana P G satisfaz
P B 2 + P C 2 = 2P M 2 +

1
P H 2 + P M 2 = 2P G2 + (2m)2 = 2P G2 + 2m2 (II)
2
e o tri
angulo AP G com mediana P H satisfaz
1
P A2 + P G2 = 2P H 2 + (2m)2 = 2P H 2 + 2m2 . (III)
2
Somando (I) e (III)
P A2 + P B 2 + P C 2 + P G2 = 2P M 2 +

a2
+ 2P H 2 + 2m2 =
2

a2
+ 2m2 = por (II)
2
a2
2(2P G2 + 2m2 ) +
+ 2m2 =
2
a2
4P G2 + 6m2 + .
2
2
a
Portanto, P A2 + P B 2 + P C 2 = 3P G2 + 6m2 + . (IV)
2
= 2(P M 2 + P H 2 ) +

Como o tri
angulo a e m s
ao constantes, P A2 + P B 2 + P C 2 e mnimo quando P G = 0, ou
seja, P = G e o baricentro do tri
angulo ABC.
A
b

H
b

b
b

POT 2012 - Geometria - Nvel 3 - Aula 15 - Prof. Ccero Thiago

Bibliografia
1. Lecture Notes on Mathematical Olympiad Courses
For Junior Section, vol. 1
Xu Jiagu
2. Puntos Notables - Teora - Demostraciones - Trazos Auxiliares
440 problemas resueltos e propuestos
Julio Orihuela Bastidas
Editorial Cuzcan
3. Geometra
Radmila Bulajich Manfrino e Jose Antonio G
omez Ortega
Cuadernos de Olimpiadas de Matematicas
4. T
opicos de Matematica Elementar, vol. 2
Geometria Euclidiana Plana
Antonio Caminha Muniz Neto
SBM
5. Episodes in Nineteenth and Twentieth Euclidean Geometry
Ross Honsberger
MAA
6. Problems in Plane and Solid Geometry, vol. 1 - Plane Geometry
Viktor Prasolov
7. Advanced Euclidean Geometry
Alfred Posamentier
8. Lessons in Geometry
I. Plane Geometry
Jacques Hadamard
AMS
9. Hadamards Plane Geometry
A Readers Companion
Mark Saul
AMS
10. Colecao Elementos da Matematica
Geometria Plana, vol. 2
Marcelo Rufino de Oliveira e Marcio Rodrigo da Rocha Pinheiro

POT 2012 - Geometria - Nvel 3 - Aula 15 - Prof. Ccero Thiago

11. Olimpadas Cearenses de Matematica, Ensino Medio, 1981 - 2005


Emanuel Carneiro, Francisco Ant
onio M. de Paiva e Onofre Campos
12. Problemas de las Olimpiadas Matematicas del Cono Sur (I a IV)
Fauring - Wagner - Wykowski - Gutierrez - Pedraza - Moreira
Red Olmpica
13. Fundamentos de Matematica Elementar, vol. 9 - Geometria Plana
Osvaldo Dolce e Jose Nicolau Pompeo
14. Olimpiada Matematica Espa
nola
15000 problemas de diferentes Olimpiadas de Matematica en el mundo

Polos Olmpicos de Treinamento


Aula

Curso de Geometria - Nvel 2

16

Prof. Ccero Thiago

Pontos Not
aveis 2: Incentro

Teorema 1. Seja XOY um


angulo dado e P um ponto em seu interior. Entao, a dist
ancia
de P a XO e igual `
a dist
ancia de P a Y O se, e somente se, o ponto P pertence `a bissetriz.
Demonstrac
ao.
X
b

M
b

b
b

N
Y
b

Suponhamos inicialmente que o ponto P pertence `a bissetriz. Entao XOP = Y OP .


Sejam M e N os pes das perpendiculares baixadas desde P sobre OX e OY , respectivamente. Podemos concluir, que M OP N OP , pelo caso L.A.A., pois OP e lado
comum, M OP = N OP e OM P = ON P = 90 . Portanto, P M = P N .
Reciprocamente, suponhamos agora que P M = P N . Pelo caso especial de congruencia
de tri
angulos, cateto-hipotenusa, os tri
angulos M OP e N OP s
ao congruentes. Portanto,
M OP = N OP , e assim, P pertence `a bissetriz.
Provemos agora que as tres bissetrizes de um tri
angulo ABC se intersectam num ponto
chamado incentro, que e equidistante dos lados do tri
angulo.

POT 2012 - Geometria - Nvel 3 - Aula 16 - Prof. Ccero Thiago

A
b

N1
N

P1
P

b
b

I
b

b
b

M1

Sejam BN e CP as bissetrizes relativas aos vertices B e C, respectivamente, e I o seu


ponto de intersec
ao. Como o ponto I pertence `as bissetrizes BN e CP , entao IM1 = IP1 e
IM1 = IN1 , em que M1 , N1 , P1 s
ao os pes das perpendiculares baixadas desde I sobre os
lados BC, CA e AB, respectivamente. Como IP1 = IN1 , entao, pela proposicao anterior,
I pertence `a bissetriz do
angulo A. Portanto, as tres bissetrizes passam por um mesmo
ponto chamado incentro que ser
a o centro da circunferencia inscrita no tri
angulo pois I
equidista dos lados do tri
angulo. Alem disso, M1 , N1 e P1 s
ao os pontos de tangencia do
crculo com os lados BC, CA e AB, respectivamente.

A
b

P1
P

N1
b

b
b

I
b

b
b

M1 M

Teorema 2. Seja ABC um tri


angulo tal que BC = a, CA = b e AB = c. Sejam M1 ,
N1 e P1 os pontos de tangencia com os lados BC, CA e AB, respectivamente. Entao,
a+b+c
.
AN1 = AP1 = p a, BM1 = BP1 = p b e CM1 = CN1 = p c, em que p =
2
2

POT 2012 - Geometria - Nvel 3 - Aula 16 - Prof. Ccero Thiago

Demonstrac
ao.
A
b

N1

P1
b

y
b

M1

Temos que y + z = a, x + z = b e x + y = c. Resolvendo o sistema encontramos x = p a,


y = p b e z = p c.
Teorema 3. (Bissetriz interna) A bissetriz interna AL do angulo A de um tri
angulo
AB
ABC divide internamente o lado oposto BC na raz
ao
, ou seja,
CA
AB
BL
=
LC
CA
em que L e o ponto de intersecc
ao da bissetriz interna com o lado BC.
Demonstrac
ao.
R
b

A
b

POT 2012 - Geometria - Nvel 3 - Aula 16 - Prof. Ccero Thiago


facil ver que
Seja R a intersecc
ao da paralela `
a bissetriz AL tracada pelo ponto C. E
BAL = CAL = ACR = ARC, com isso, AR = AC. Pelo teorema de Tales temos
que
BL
AB
=
.
AR
LC
Como AR = AC, ent
ao
AB
BL
=
.
AC
LC
Teorema 4. Seja ABC um tri
angulo tal que BC = a, CA = b, AB = c e seja AM a
ac
bissetriz relativa ao
angulo A, com M em BC. Entao, BM =
.
b+c
Demonstrac
ao.
A
b

b
b

am

Usando o teorema da bissetriz interna temos que


AC
c
b
ac
AB
=

=
m=
.
BM
CM
m
am
b+c
Teorema 5. Seja ABC um tri
angulo tal que BC = a, CA = b, AB = c, AM a bissetriz
AI
b+c
relativa ao angulo A, com M em BC, e seja I o incentro. Entao,
=
.
IM
a
Demonstrac
ao.
Aplicando o teorema da bissetriz interna no tri
angulo BAM temos que
AB
AI
b+c
AI
=

=
.
IM
BM
IM
a

POT 2012 - Geometria - Nvel 3 - Aula 16 - Prof. Ccero Thiago

A
b

c
b

b
b

ac
b+c

Teorema 6. Seja ABC um tri


angulo e I seu incentro. Seja E o ponto de intersecao de
AI com a circunferencia circunscrita ao tri
angulo ABC. Entao IE = IB = IC.
Demonstrac
ao.
A
b

I
b

facil ver que BAE = CAE = CBE = BCE e, portanto, BE = CE. Alem disso,
E
pela propriedade do
angulo externo, BIE = +. Portanto, BIE = IBE e BE = IE.
5

POT 2012 - Geometria - Nvel 3 - Aula 16 - Prof. Ccero Thiago

Observe agora uma parte da figura acima.


a
2

B
b

M
b
b

Temos que
a
BM
BE =
= CE = IE.
BE
2 cos
Teorema 7. Seja ABC um tri
angulo tal que BC = a, CA = b, AB = c e seja AM a
bissetriz relativa ao
angulo A, com M em BC. Alem disso, BAM = CAM = . Entao
2 b c cos
AM =
.
b+c
Demonstrac
ao.
cos =

A
b

b
b

facil ver que [ABC] = [BAM ] + [CAM ]. Entao,


E
c AM sin b AM sin
b c sin 2
=
+

2
2
2
AM =

2 b c cos
.
b+c

Teorema 8. (Area
de um tri
angulo em fun
c
ao do raio da circunfer
encia inscrita.)
Sejam a, b e c as medidas dos lados BC, CA e AB do tri
angulo ABC, respectivamente, e
seja r a medida do raio da circunferencia inscrita. Entao, a area do tri
angulo ABC pode
6

POT 2012 - Geometria - Nvel 3 - Aula 16 - Prof. Ccero Thiago

ser calculada por


[ABC] = p r,
em que p =

a+b+c
.
2

Demonstrac
ao.

A
b

r
b

r
b
b

[ABC] = [BIC] + [CIA] + [AIB]


ar br cr
+
+

2
2
2


a+b+c
[ABC] =
r
2

[ABC] =

[ABC] = p r.
Problema 1. (OBM) O tri
angulo ABC e ret
angulo em B. Sejam I o centro da circunferencia inscrita em ABC e O o ponto medio do lado AC. Se AOI = 45 , quanto mede,
em graus, o
angulo ACB?
Soluc
ao.
Como ABC e um tri
angulo ret
angulo, entao AO = BO = CO. Se ABI = AOI = 45
e BAI = OAI, ent
ao ABI AOI (ALA). Com isso, AB = AO = BO e, portanto,
tri
angulo ABO e equilatero. Assim, ACB = 30 .

POT 2012 - Geometria - Nvel 3 - Aula 16 - Prof. Ccero Thiago

B
b

I
b

Problema 2. Em um tri
angulo n
ao equilatero, a reta que passa pelo baricentro e pelo incentro e paralela a um dos lados do tri
angulo. Demonstre que os lados do tri
angulo est
ao
em progressao aritmetica.
Soluc
ao.
Como IG e paralelo a BC ent
ao podemos aplicar o teorema de Tales. Assim,
AG
b+c
2
AI
=

= b + c = 2a.
IE
GD
a
1
A
b

E
b
b

D
b

Exerccios propostos

POT 2012 - Geometria - Nvel 3 - Aula 16 - Prof. Ccero Thiago

1.
2. (IMO Shortlist) Seja ABC um tri
angulo tal que AB + BC = 3AC. Sejam I o seu
incentro e D e E os pontos de tangencia da circunferencia inscrita com os lados AB
e BC, respectivamente. Alem disso, sejam K e L os simetricos de D e E com relacao
ao incentro I. Prove que o quadril
atero ACKL e inscritvel.
3. (Teste de selec
ao do Brasil para IMO) Seja I o incentro do tri
angulo ABC e D o
ponto de intersec
ao de AI com o crculo circunscrito de ABC. Sejam E e F os pes
das perpendiculares baixadas a partir de I sobre BD e CD, respectivamente. SE
AD
, determine o
angulo BAC.
IE + IF =
2
4. (IMO) O prolongamento da bissetriz AL do tri
angulo acut
angulo ABC encontra o
crculo circunscrito em N . Por L tracam - se perpendiculares LK e LM aos lados
AB e AC, respectivamente. Prove que a area do tri
angulo ABC e igual `a area do
quadril
atero AKN M .
5. Num tri
angulo ABC tem - se AB = BC, e D e um ponto sobre a base AC tal que
o raio do crculo inscrito no tri
angulo ABD e igual ao raio do crculo tangente ao
segmento DC e aos prolongamentos das retas BD e BC. Prove que o raio deste
1
angulo ABC.
crculo e igual a da medida h de uma das alturas iguais do tri
4
6. Seja um quadril
atero ABCD inscrito num crculo de tal forma que os prolongamentos
dos lados AD e BC se encontram em Q e os prolongamentos de AB e CD, em P .
Prove que as bissetrizes dos
angulos DQC = AP D s
ao perpendiculares.
7. Do incentro de um tri
angulo ret
angulo, avista - se a metade da hipotenusa, isto e,
o segmento que une um vertice ao ponto medio da hipotenusa, segundo um angulo
m
e a frac
ao irredutvel que expressa a raz
ao entre as medidas dos catetos
reto. Se
n
deste tri
angulo, ent
ao m + n e igual a:
(a) 7 (b) 17 (c) 23 (d) 31 (e) 41
8. O crculo, de centro O, inscrito no tri
angulo ABC e cortado pela mediana AD nos
pontos X e Y . Sabendo que AC = AB + AD, determine a medida do angulo XOY .
9. (OCM) Seja ABC um tri
angulo cuja medida dos lados s
ao n
umeros inteiros e consecutivos. Alem disso, o maior
angulo A e o dobro do menor angulo. Determine a
medida dos lados deste tri
angulo.

Polos Olmpicos de Treinamento


Aula

Curso de Geometria - Nvel 2

17

Prof. Ccero Thiago

Pontos Not
aveis 3: Circuncentro e Ortocentro

Teorema 1. Sejam A, B e P tres pontos distintos no plano. Temos que P A = P B se, e


somente se, o ponto P pertence `
a mediatriz do segmento AB.
Demonstrac
ao.
r
P
b

b
b

Sejam M o ponto medio de AB e r a sua mediatriz. Suponha inicialmente que P pertence


facil ver que os tri
a` mediatriz. Com isso AM = M B e r e perpendicular `a AB. E
angulos
AM P e BM P s
ao congruentes pelo caso L.A.L. e, com isso, P A = P B.
Reciprocamente, suponha agora, que P A = P B, com isso ABP e is
osceles de base AB.
facil ver que os tri
Tracemos a mediana relativa ao lado AB. E
angulos AM P e BM P
s
ao congruentes pelo caso L.L.L. e, com isso, AM P BM P = 90 , ou seja, P est
a
sobre a mediatriz.
Teorema 2. As tres mediatrizes de um tri
angulo ABC se intersectam num ponto chamado
circuncentro que e o centro da circunferencia circunscrita.
Demonstrac
ao.

POT 2012 - Geometria - Nvel 2 - Aula 17 - Prof. Ccero Thiago

O
b

Sejam r e s as mediatrizes relativas aos lados BC e AB, respectivamente, e seja O o ponto


de intersecao das duas mediatrizes. Pelo teorema 1, temos que BO = CO e BO = AO.
Entao, CO = AO e, tambem pelo teorema 1, O deve estar sobre a mediatriz relativa ao
lado AC.
Alem disso o circuncentro e o centro da circunferencia circunscrita ao tri
angulo ABC pois
e equidistante dos tres vertices do tri
angulo.
Teorema 3. As tres alturas de um tri
angulo ABC se intersectam num ponto chamado
ortocentro.
Demonstrac
ao.

POT 2012 - Geometria - Nvel 2 - Aula 17 - Prof. Ccero Thiago

P
b

E
b

F
b

H
b

D
b

b
b

Inicialmente tracemos pelos vertices A, B e C, retas paralelas aos lados BC, CA e AB,
respectivamente, que determinam o tri
angulo M N P . J
a sabemos que as tres mediatrizes
facil perceber que A, B e C s
de um tri
angulo se intersectam em seu circuncentro. E
ao
os pontos medios dos segmentos N P , M P e M N , respectivamente, pois P ACB, N ABC e
ABM C s
ao paralelogramos e, portanto, os lados opostos de um paralelogramo s
ao iguais.
Tracemos as mediatrizes dos segmentos M P , M N e P N que ir
ao se intersectar no ponto
H. Mas as mediatrizes do tri
angulo M N P s
ao as alturas do tri
angulo ABC. Portanto,
provamos que as tres alturas de um tri
angulo ABC se intersectam em um ponto que ser
a
chamado de ortocentro.
Teorema 4. Seja O o centro da circunferencia circunscrita ao tri
angulo acut
angulo ABC
e seja D a projec
ao de A sobre BC ent
ao DAB = OAC.
Demonstrac
ao.

POT 2012 - Geometria - Nvel 2 - Aula 17 - Prof. Ccero Thiago

A
b

O
b

b
b

Seja AE um di
ametro. Alem disso, ABC = AEC. Portanto, BAD = EAC.
Teorema 5. O ortocentro, o baricentro e o circuncentro de um tri
angulo, n
ao equilatero,
s
ao colineares. A reta determinada por esses pontos e chamada de Reta de Euler.
Demonstrac
ao.
Sejam M e N os pontos medios de BC e AC, respectivamente. Entao, M N k AB e
AB
MN =
. O teorema 4 garante que BAD = OAC. Como O e o circuncentro
2
entao OA = OC e, com isso, OAC = OCA. O quadril
atero M CN O e inscritvel
entao OCA = N CO = OM N e M ON = 180 ACB. Alem disso, o quadril
atero DCEH tambem e inscritvel e, com isso, DHE = 180 ACB. Como
DHE = AHB conclumos que o tri
angulo AHB e semelhante ao tri
angulo M N O e, com
AH
AB
=
= 2. Temos que HAG = GM O pois AH e paralelo a OM e, como
isso,
MN
OM
AG
= 2. Portanto, o tri
angulo AHG e semelhante ao tri
angulo GM O
G e o baricentro,
GM
e, com isso, HGA = M GO provando entao que H, G e O est
ao alinhados e HG = 2GO.

POT 2012 - Geometria - Nvel 2 - Aula 17 - Prof. Ccero Thiago

A
b

b
b

H
b

G
b

b
b

b
b

Teorema 6. Os pes das alturas de um tri


angulo, os pontos medios do tres lados e os pontos
medios dos segmentos que ligam os vertices ao ortocentro est
ao sobre uma circunferencia
chamada Circunfer
encia dos 9 pontos.

Demonstrac
ao. Queremos provar que M , L, P , D, E, F , R, S e T s
ao concclicos. E
suficiente provar que R e D est
ao sobre a circunferencia circunscrita ao tri
angulo M LP ,
facil ver que
pois o restante e an
alogo. Considere a circunferencia de di
ametro RM . E
D pertence a . Por outro lado, RL k HC, LM k AB e HC AB, o que implica que
RLM = 90 . Portanto, L (e por simetria P ) pertence a .
A
b

E
b

F
b

H
b

P
b

L
b

N
T

S
b

b
b

Teorema 7. O centro da circunferencia dos 9 pontos e o ponto medio do segmento formado


pelo ortocentro e pelo circuncentro.
Demonstrac
ao.
Seja RM um di
ametro da circunferencia dos 9 pontos e seja N a intersecao de RM e OH.
5

POT 2012 - Geometria - Nvel 2 - Aula 17 - Prof. Ccero Thiago

Como R e ponto medio de AH ent


ao RH = OM . Alem disso, AH k OM . Portanto,
RHN N OM , RN = N M e HN = ON .
A
b

R
b
b

F
b

L
b

H
b

N
b
b

T
b

S
b

b
b

Problema 1. Seja ABC um tri


angulo e sejam H o ortocentro e o O o circuncentro do
tri
angulo. Se ABH = HBO = OBC e BH = BO determine a medida do angulo A.
Soluc
ao.
Como O e o circuncentro ent
ao OC = OB = BH. Alem disso, BH = OC = 2OM . Como
o tri
angulo M OC e ret
angulo ent
ao M OC = 60 . Assim, AOC = 120 e ABC = 60 .

POT 2012 - Geometria - Nvel 2 - Aula 17 - Prof. Ccero Thiago

B
b

O
H

b
b

Problema 2. Seja H o ortocentro de um tri


angulo ABC, tal que AC 6= BC. O segmento
que une os pontos medios de HC e AB intersecta a bissetriz de ACB no ponto N . Sabendo que o circuncentro do tri
angulo ABC pertence `a reta que passa pelos pontos H e
N , determine a medida do ACB.
Soluc
ao.
Seja M o ponto medio de AB, L o ponto medio de HC, O o circuncentro do tri
angulo ABC
bem sabido que OM = 1 HC = LC.
e R o raio do crculo circunscrito ao tri
angulo ABC. E
2
Como OM e paralelo a LC, ent
ao OM LC e um paralelogramo. Por outro lado, a bissetriz
do angulo ACB e bissetriz tambem do angulo OCH, da LN C = N CO = N CL e
N L = CL = LH, o que implica HN C = 90 , logo CN e altura e bissetriz do tri
angulo

HOC, assim HC = CO e, portanto, ACB = 60 .


Problemas propostos

1. Seja ABC um tri


angulo tal que ABC = 50 . Seja F um ponto qualquer sobre o
lado AC. Se M e N s
ao os ortocentros dos tri
angulos ABF e BF C, respectivamente,
determine a medida do
angulo M F N .

POT 2012 - Geometria - Nvel 2 - Aula 17 - Prof. Ccero Thiago

2. Seja ABC um tri


angulo tal que BAC = 40 e seja P um ponto sobre o lado AB tal
que o ortocentro de ABC coincide com o circuncentro de P BC. Determine a medida
do angulo P CB.
3. (ITA) Em um tri
angulo de vertices A, B e C, a altura, a bissetriz e a mediana, relativamente ao vertice C, dividem o angulo BCA em quatro angulos iguais. Se l e a
medida do lado oposto ao vertice C, calcule:
(a) A medida da mediana em funcao de l.
(b) Os
angulos CAB, ABC e BCA.
4. Seja ABC um tri
angulo que n
ao e is
osceles. Os pontos O e H s
ao, respectivamente,
o circuncentro e o ortocentro e M o ponto medio de OH.
(a) Se ABC e um tri
angulo acut
angulo e a bissetriz interna de BAC passa por M ,
determine a medida do
angulo BAC.
(b) Se ABC e um tri
angulo obtusangulo e a bissetriz externa do angulo BAC passa
por M , determine a medida do
angulo BAC.
5. (Torneio das cidades) AD, BE e CF s
ao alturas de um tri
angulo ABC. K, M e N
s
ao os ortocentros dos tri
angulos AEF , BF D e CDE. Prove que KM N e DEF s
ao
tri
angulos congruentes.
6. Seja ABC um tri
angulo. Sobre os lados AB e AC s
ao construdos no exterior do
tri
angulo os quadrados ABDE e ACF G. Prove que CD, BF e a altura relativa ao
vertice A s
ao concorrentes.
7. (OBM) Sejam H, I e O o ortocentro, o incentro e o circuncentro do tri
angulo ABC,
respectivamente. A reta CI corta o circuncrculo de ABC no ponto L, distinto de C.
Sabe-se que AB = IL e AH = OH. Determine os angulos do tri
angulo ABC.
8. (Ira) Em um tri
angulo ABC temos que A = 60 . Seja D um ponto que varia sobre
o lado BC. Sejam O1 o circuncentro de ABD e O2 o circuncentro de ACD. Seja M
a intersec
ao de BO1 e CO2 e N o circuncentro de DO1 O2 . Prove que M N passa por
um ponto fixo.

Polos Olmpicos de Treinamento


Aula

Curso de Geometria - Nvel 2

18

Prof. Ccero Thiago

Circunfer
encias ex - inscritas

Teorema 1. Seja XOY um


angulo dado e P um ponto em seu interior. Entao, a dist
ancia
de P a XO e igual a dist
ancia de P a Y O se, e somente se, o ponto P pertence a bissetriz.
Demonstrac
ao.
b

M
b

b
b

Suponhamos inicialmente que o ponto P pertence `a bissetriz. Entao, XOP = Y OP . Sejam M e N os pes das perpendiculares baixadas desde P sobre OX e OY , respectivamente.
Podemos concluir, que M OP N OP , pelo caso L.A.A.. Portanto, P M = P N .
Reciprocamente, suponhamos agora que P M = P N . Pelo caso especial de congruencia de
tri
angulos, cateto - hipotenusa, os tri
angulos M OP e N OP s
ao congruentes. Portanto,
M OP = N OP e, assim, P pertence `a bissetriz.
Teorema 2. As bissetrizes externas de quaisquer dois angulos de um tri
angulo s
ao concorrentes com a bissetriz interna do terceiro angulo.
Demonstrac
ao.

POT 2012 - Geometria - Nvel 2 - Aula 18 - Prof. Ccero Thiago

E
b

No tri
angulo ABC tracamos as bissetrizes externas dos angulos A e B os quais se
intersectam em P . Do teorema 1, como P pertence `a bissetriz externa do angulo A, entao
P E = P F . Alem disso, P pertence `
a bissetriz externa do angulo B, entao P F = P D.
Como P D = P E, pelo teorema 1, conclumos que P pertence `a bissetriz do angulo C.
Dessa forma, se P equidista dos tres lados do tri
angulo ABC e e um ponto no exterior do
tri
anglo entao P e o centro de uma das tres circunferencias ex - inscritas do tr
angulo ABC.
A circunferencia com centro Ia e raio ra e uma das tres circunferencias ex - inscritas que
representaremos apenas por (Ia , ra ). Analogamente s
ao definidas as circunferencias (Ib , rb )
e (Ic , rc ). Os pontos Ia , Ib e Ic s
ao os ex - incentros. Cada circunferencia ex - inscrita
toca um dos lados do tri
agulo internamente e os outros dois externamente, ou seja, toca no
prolongamento. Na figura a seguir, observe que pela propriedade de segmentos tangentes a
uma circunferencia, vulgarmente conhecido com Teorema do bico, temos que BL = BG,
alem disso
BL + BG = (BC + CL) + (AG + AB)
= BC + CE + AE + AB = a + b + c = 2p.
Portanto, as tangentes tracadas por B a` circunferencia (Ib , rb ) tem medida p. Dessa forma
e facil ver que
AJ = AK = BG = BL = CH = CM = p.
Alem disso, CL = BL BC = p a. Entao,
BM = BF = CL = CE = p a,
CK = CD = AH = AF = p b,
AG = AE = BJ = BD = p c.

POT 2012 - Geometria - Nvel 2 - Aula 18 - Prof. Ccero Thiago

G
b

Ib

H
b

A
b

Ic
b

I
b

E
b

L
b

b
b

K
b

Ia

Teorema 3. (Bissetriz externa) A bissetriz externa AL do angulo A de um tri


angulo
AB
ABC divide externamente o lado oposto BC na raz
ao
, ou seja,
CA
BL
AB
=
LC
CA
em que L e o ponto de intersecc
ao da bissetriz externa com o lado BC.
Demonstrac
ao.

POT 2012 - Geometria - Nvel 2 - Aula 18 - Prof. Ccero Thiago

R
b

facil ver que


Seja R a intersecc
ao da paralela `
a bissetriz AL tracada pelo ponto C. E
EAL = CAL = ACR = ARC, com isso, AR = AC. Pelo teorema de Tales temos
que
AB
BL
=
.
AR
LC
Como AR = AC, ent
ao
BL
AB
=
.
AC
LC

Teorema 4. (Area
de um tri
angulo em fun
c
ao do raio de uma circunfer
encia ex
- inscrita.)
Sejam a, b e c as medidas dos lados BC, CA e AB do tri
angulo ABC, respectivamente,
e sejam ra , rb e rc os raios das circunferencias ex - inscritas relativas aos lados a, b e c,
respectivamente. Ent
ao, a
area do tri
angulo ABC pode ser calculada por
[ABC] = ra (p a) = rb (p b) = rc (p c),
em que p =

a+b+c
.
2

Demonstrac
ao.

POT 2012 - Geometria - Nvel 2 - Aula 18 - Prof. Ccero Thiago

F
b

ax
ra

C
b

Ia
b

ax

ra
ra
D

x
b

Pela propriedade dos segmentos tangentes, temos que DB = BE = x e DC = CF = a x.


Entao,
[ABC] = [AIa E] + [AIa F ] 2[BCIa ]
(c + x) ra (b + a x) ra
a ra
+
2

2
2
2
ra
ra
[ABC] =
(a + b + c 2a) =
(2p 2a) = ra (p a).
2
2
Analogamente,
[ABC] =

[ABC] = rb (p b) = rc (p c),
Problema 1. Sejam ABC um tri
angulo, M o pe da bissetriz interna do angulo A e N o
pe da bissetriz interna do
angulo B. Suponha que M N seja bissetriz do angulo AM C.
Calcule a medida do
angulo A.
Soluc
ao.

E facil ver que N e um dos ex - incentros do tri


angulo ABC pois e a intersecao da bissetriz externa do
angulo AM B e da bissetriz interna do angulo B. Logo, AN e bissetriz
externa do angulo A. Portanto, A = 120 .

POT 2012 - Geometria - Nvel 2 - Aula 18 - Prof. Ccero Thiago

b
b

Problema 2. (OBM) Um tri


angulo ABC, de lados AB = c, AC = b e BC = a, tem
permetro 2p. Uma circunferencia tangencia o lado BC e os prolongamentos dos lados AB
e AC nos pontos P , Q e R, respectivamente. O comprimento AR e igual a:
(a) p a (b) p b (c) p c (d) p (e) 2p
Soluc
ao.
A
b

b
b

R
b

Q
b

IA

POT 2012 - Geometria - Nvel 2 - Aula 18 - Prof. Ccero Thiago

Pelo teorema 2 e facil ver que AR = AQ = p. Portanto, a resposta e o item (b).


Problema 3. No quadril
atero ABCD determine a medida do angulo AED.
C
b

D
b

60
60

E
b

70
40
b

Soluc
ao.
b

60
D
b

60
60

E
b

70

40
40

70

40

Na figura, F DC = 60 e GBC = 70 . Entao, BC e DC s


ao bissetrizes externas dos
angulos ABD e ADB. Dessa forma, AC e bissetriz interna do angulo BAD. Portanto,
DAE = BAE = 40 . Finalmente, AED = 80 .
Exerccios propostos
1. Num tri
angulo ABC tem - se AB = BC, e D e um ponto sobre a base AC tal que
o raio do crculo inscrito no tri
angulo ABD e igual ao raio do crculo tangente ao
segmento DC e aos prolongamentos das retas BD e BC. Prove que o raio deste
1
crculo e igual a da medida h de uma das alturas iguais do tri
angulo ABC.
4
2. Prove que os tres segmentos determinados por um vertice e pelo ponto de tangencia
7

POT 2012 - Geometria - Nvel 2 - Aula 18 - Prof. Ccero Thiago

da circunferencia ex - inscrita com o lado oposto a esse vertice s


ao concorrentes em
um ponto chamado ponto de Nagel.
3. (OBM) A medida do
angulo B de um tri
angulo ABC e 120 . Sejam M um ponto sobre o lado AC e K um ponto sobre o prolongamento do lado AB, tais que BM e a bissetriz interna do
angulo ABC e CK e a bissetriz externa correspondente ao angulo
ACB. O segmento M K intersecta BC no ponto P . Prove que AP M = 30 .
4. (Leningrado) Sejam AF , BG e CH as bissetrizes de um tri
angulo ABC que tem
angulo A medindo 120 . Prove que o angulo GF H mede 90 .
5. (Belarus) Seja O o centro do crculo ex - inscrito do tri
angulo ABC oposto ao vertice
A. Seja M o ponto medio de AC e seja P a intersecao das retas M O e BC. Prove
que se BAC = 2ACB, ent
ao AB = BP .
6. (IMO) Dado um tri
angulo ABC, o ponto J e o centro da circunferencia ex-inscrita
oposta ao vertice A. Esta circunferencia ex-inscrita e tangente ao lado BC em M ,
e `as retas AB e AC em K e L, respectivamente. As retas LM e BJ intersectam-se
em F , e as retas KM e CJ intersectam-se em G. Seja S o ponto de intersecao das
retas AF e BC, e seja T o ponto de intersecao das retas AG e BC. Prove que M e
o ponto medio de ST .
(A circunferencia ex-inscrita de ABC oposta ao vertice A e a circunferencia tangente
ao segmento BC, ao prolongamento do segmento AB no sentido de A para B e ao
prolongamento do segmento AC no sentido de A para C.)
Bibliografia
1. T
opicos de Matematica Elementar - Vol. 2
Antonio Caminha Muniz Neto
2. Geometria
Radmila Bulajich Manfrino e Jose Antonio G
omez Ortega

Programa Olmpico de Treinamento


Aula

Curso de Geometria - Nvel 2

Prof. Rodrigo Pinheiro

Problemas OBM - 1 Fase

Problema 1. Dois espelhos formam um angulo de 30 no ponto V . Um raio de luz, vindo


de uma fonte S, e emitido paralelamente a um dos espelhos e pe refletido pelo outro espelho
no ponto A, como mostra a figura. Depois de uma certa quantidade de reflexoes, o raio
retorna a S. Se AS e AV tem 1m de comprimento, a dist
ancia percorrida pelo raio de luz,
em metros, e:
a) 2

b) 2 + 3
p

c) 2(1 + 3)

d) 1 + 2 + 3

e) 5 3
b

S
b

30
b

Problema 2. Na figura, quanto vale x?


a) 6
b) 12
c) 18
d) 20
e) 24

POT 2012 - Geometria - Nvel 2 - Aula 5 - Prof. Rodrigo Pinheiro

5x

3x

2x
6x

4x

Problema 3. No tri
angulo P QR, a altura P F divide o lado QR em dois segmentos de
medidas QF = 9 e RF = 5. Se P R = 13, qual e a media de P Q?
a) 5
b) 10
c) 15
d) 20
e) 25
Problema 4. No desenho abaixo, o quadrilatero ABCD e um quadrado de lado 3cm e os
tri
angulos ABF e AED s
ao ambos equil
ateros. Qual a area do triangulo AEF?
a) 2cm2
b) 1, 5cm2
c) 3cm2
d) 4, 5cm2
e) 2, 5cm2

POT 2012 - Geometria - Nvel 2 - Aula 5 - Prof. Rodrigo Pinheiro

A
b

B
b

Problema 5. Eu planejava fazer um curral quadrado, com uma certa area, usando uma
certa quantidade de cerca de arame farpado. Descobri, porem, que tenho 10% a menos de
cerca do que esperava. Por esta razao, a area cercada sera:
a) 5% menor
b) 10% menor
c) 19% menor
d) 20% menor
e) 25% menor
Problema 6. O ret
angulo da figura a seguir esta dividido em 7 quadrados. Se a area do
menor quadrado e igual a 1, a
area do ret
angulo e igual a:
a) 42
b) 44
c) 45
d) 48
e) 49

POT 2012 - Geometria - Nvel 2 - Aula 5 - Prof. Rodrigo Pinheiro

Problema 7. Na organizacao retangular de pontos da figura abaixo, a dist


ancia entre pontos
vizinhos em uma mesma linha ou coluna e igual a 1cm. A area do pentagono ABCDE,
em cm2 , e igual a:
a) 9
b)

19
2

c) 10
d)

21
2

e) 11
b

A
b

Problema 8. A figura a seguir mostra um quadrado ABCD e um tri


angulo equilatero
BEF , ambos com lado de medida 1cm. Os pontos A, B e E s
ao colineares, assim como os
pontos A, G e F . A
area do tri
angulo BF G e, em cm2 :
a)

1
4

b)

1
3

d)

3
4

3
12

e)

3
10

c)

POT 2012 - Geometria - Nvel 2 - Aula 5 - Prof. Rodrigo Pinheiro

C
b

G
b

E
b

Problema 9. No tri
angulo ABC, AB = 20, AC = 21 e BC = 29. Os pontos D e E sobre
o lado BC s
ao tais que BD = 8 e EC = 9. A medida do angulo DAE, em graus, e igual
a:
a) 30
b) 40
c) 45
d) 60
e) 75
Problema 10. Seis ret
angulos identicos s
ao reunidos para formar um ret
angulo maior conforme indicado na figura com AB = 21cm. Qual e a area deste ret
angulo maior?
a) 210cm2
b) 280cm2
c) 430cm2
d) 504cm2
e) 588cm2
b

Problema 11. O desenho ao lado mostra um pedaco de papel


ao que sera dobrado e colado
nas bordas para formar uma caixa retangular. Os angulos nos cantos do papelao s
ao todos
retos. Qual sera o volume da caixa em cm2 ?
a) 1500
5

POT 2012 - Geometria - Nvel 2 - Aula 5 - Prof. Rodrigo Pinheiro

b) 3000
c) 4500
d) 6000
e) 12000
15cm
b

40cm

20cm

b
b

Problema 12. Na figura, os dois tri


angulos s
ao equil
ateros. Qual e o valor do angulo x?
a) 30
b) 40
c) 50
d) 60
e) 70

75

65

Problema 13. Na figura, todas as circunferencias menores tem o mesmo raio r e os centros
das circunferencias que tocam circunferencia maior s
ao vertices de um quadrado. Sejam a
e b as areas cinzas indicadas na figura. Entao a razao ab e igual a:
a)

1
2

b)

2
3

POT 2012 - Geometria - Nvel 2 - Aula 5 - Prof. Rodrigo Pinheiro

c) 1
d)

3
2

e) 2

Problema 14. Na figura, a reta P Q toca em N o crculo que passa por L, M e N . A reta
LM corta a reta P Q em R. Se LM = LN e a medida do angulo P N L e , < 60 , quanto
mede o angulo LRP ?
a) 3 180
b) 180 2
c) 180
d) 90

e)

L
b

b
b

Q
b

Problema 15. Sao dadas duas tiras retangulares de papel com 20cm de comprimento, uma
com 5cm de largura e outra com 11cm de largura. Uma delas foi colocada sobre a outra,
perpendicularmente, de modo a formar a figura ilustrada ao lado. O permetro dessa figura,
em centmetros e:
a) 50
b) 60
7

POT 2012 - Geometria - Nvel 2 - Aula 5 - Prof. Rodrigo Pinheiro

c) 80
d) 100
e) 120

Problema 16. Na figura, AB = AC, AE = AD e o angulo BAD mede 30 . Entao o angulo


x, mede:
a) 10
b) 20
c) 15
d) 30
e) 5
b

30
b

Problema 17. Tres quadrados s


ao colocados pelos seus vertices entre si e a dois bast
oes
verticais, como mostra a figura.

A medida do
angulo x e:
a) 39
8

POT 2012 - Geometria - Nvel 2 - Aula 5 - Prof. Rodrigo Pinheiro

b) 41
c) 43
d) 44
e) 46
Problema 18. Um tri
angulo equil
atero e um hex
agono regular tem o mesmo permetro. A
razao entre a
area do tri
angulo e a
area do hex
agono e:
a)

1
2

b) 1
c)

2
3

d)

3
2

e)

1
3

Problema 19. A figura a seguir representa um Tangram, quebra-cabecas chines formado


por 5 tri
angulos, 1 paralelogramo e 1 quadrado. Sabendo que a area do Tangram a seguir
e 64cm2 , qual e a
area, em cm2 , da regi
ao sombreada?

a) 7.6
b) 8
c) 10.6
d) 12
e) 21.3
Problema 20. Na figura a seguir, ABC e um tri
angulo qualquer e ACD e AEB s
ao
tri
angulos equil
ateros. Se F e G s
ao os pontos medios de EA e AC, respectivamente, a
e:
razao BD
FG

POT 2012 - Geometria - Nvel 2 - Aula 5 - Prof. Rodrigo Pinheiro

E
b

F
b

a)

1
2

b) 1
c)

3
2

d) 2
e) Depende das medidas dos lados de ABC.
Problema 21. Na figura, o lado AB do tri
angulo equil
atero ABC e paralelo ao lado DG
do quadrado DEF G. Qual e o valor do angulo x?
a) 80
b) 90
c) 100
d) 110
e) 120

Problema 22. O jardim da casa de Maria e formado por cinco quadrados de igual area e
tem a forma da figura abaixo. Se AB = 10m, entao a area do jardim em metros quadrados
e:
a) 200
10

POT 2012 - Geometria - Nvel 2 - Aula 5 - Prof. Rodrigo Pinheiro

b) 10 5
c) 100
d)

500
3

e)

100
3

Problema 23. A figura abaixo e formada por tres quadrados de lado 1cm e um ret
angulo
que os contorna. A
area do ret
angulo e:

a) 3 2

b) 4 2
c) 6

d) 6 2
e) 8
Problema 24. Qual o menorpermetro inteiro possvel de um tri
angulo que possui um dos
lados com medida igual a 5. 23 ?
a) 8
b) 9
c) 10
d) 11
e) 12

11

POT 2012 - Geometria - Nvel 2 - Aula 5 - Prof. Rodrigo Pinheiro

Problema 25. Uma mesa de bilhar tem dimens


oes de 3m por 6m e tem cacapas nos seus
quatro cantos P , Q, R e S. Quando uma bola atinge na borda da mesa, sua trajet
oria
forma um angulo igual ao que o
angulo anterior formava. Uma bola inicialmente a 1 metro
da cacapa P , e batida do lado SP em direcao ao lado P Q, como mostra a figura. A quantos
metros de P a bola acerta o lado P Q se a bola cai na cacapa S ap
os duas batidas na borda
da mesa?
a) 1
b)

6
7

c)

3
4

d)

2
3

e)

3
5

Gabarito
1) B
2) C
3) C
4) D
5) D
6) C
7) B
8) D
9) C
10) E
11) B
12) B
13) C
14) A
15) C
16) C
17) A
18) C
19) D
12

POT 2012 - Geometria - Nvel 2 - Aula 5 - Prof. Rodrigo Pinheiro

20)
21)
22)
23)
24)
25)

D
E
C
C
B
B

13

Você também pode gostar